Sunteți pe pagina 1din 87

RECREAII

MATEMATICE


REVI ST DE MATEMATI C PENTRU ELEVI I PROFESORI






















E d i t u r a Cr e n g u a G l d u
I AI , 2004
Iulie Decembrie 2004
Anul VI, Nr. 2
1 =
i
e


Semnificaia formulei de pe copert:
ntr-o form concis, formula 1 =
i
e leag cele patru ramuri fundamen-
tale ale matematicii:

ARITMETICA reprezentat de 1
GEOMETRIA reprezentat de
ALGEBRA reprezentat de i
ANALIZA MATEMATIC reprezentat de e



Redacia revistei :
Petru ASAFTEI , Temistocle BRSAN, Dan BRNZEI, Ctlin - Cristian BUDEANU,
Constantin CHIRIL, Eugenia COHAL, Adrian CORDUNEANU, Mihai CRCIUN
(Pacani), Paraschiva GALIA, Paul GEORGESCU, Dumitru GHERMAN (Pacani),
Gheorghe IUREA, Lucian Georges LDUNC, Mircea LUPAN, Dan tefan
MARINESCU (Hunedoara), Gabriel MRANU, Andrei NEDELCU, Gabriel POPA,
Dan POPESCU (Suceava), Florin POPOVICI (Braov), Maria RACU, Ioan ERDEAN
(Ortie), Dan TIBA (Bucureti), Lucian Tuescu (Craiova), Adrian ZANOSCHI.



Responsabili de numr :
Temistocle BRSAN, Gabriel POPA, Paul GEORGESCU, Gheorghe IUREA,
Lucian Georges LDUNC, Mircea LUPAN, Andrei NEDELCU





Adresa redaciei:
Catedra de Matematic Universitatea Tehnic Gh. Asachi Iai
Bd. Carol I, nr.11, 700506, Iai
Tel. 032 213737 / int. 123
E-mail: acord@math.tuiasi.ro



EDITURA CRENGUA GLDU
Toate drepturile rezervate
ISSN 1582 - 1765
Bd. N. Iorga, Bl. K2, ap. 4, IAI
Tel. / Fax: 032 - 230598




RECREAI I
MATEMATICE


REVI ST DE MATEMATI C PENTRU ELEVI I PROFESORI




















Apare cu sprijinul

FILIALEI IAI a SOCIETII de TIINE MATEMATICE
i INSPECTORATULUI COLAR al JUDEULUI IAI

IAI, 2004

Anul VI, Nr. 2
Iulie Decembrie 2004
1 =
i
e

C atre cititori
dup a cinci ani de aparitie a revistei
Fugit irreparabile tempus
Revista "Recrea tii matematice", cu dou a aparitii pe an (exceptnd primul an, 1999,
n care a fost publicat un singur num ar), a intrat n al saselea an al existentei sale.
Numele ei deriv a din cel al revistei "Recrea tii stiin tice", care a ap arut la Iasi n
perioada 1883-1888 si care este prima publicatie din tara noastr a destinat a tineretu-
lui studios. Revista "Recreatii stiintice" a publicat materiale din toate domeniile
stiintei, dar cu prec adere articole, note si probleme de matematic a. Este o cutezant a
faptul de a ne lega numele de aceast a veche si prestigioas a revist a si este de o mare
r aspundere ncercarea noastr a de a o continua prin asumarea obiectivelor acesteia
(v. RecMat - 1/2003, pp. 1-5), r amase actuale si acum, la mai mult de o sut a de ani.
Membrii fondatori ai revistei "Recreatii Matematice" sunt cei prezenti n redactia
primului num ar: Temistocle Brsan, C at alin Calistru, Alexandru C ar au su, Con-
stantin Cocea, Adrian Corduneanu si Gheorghe Iurea. Redactia revistei s-a modi-
cat si s-a l argit cu timpul; actuala redactie (prezent a pe copert a, interior) este for-
mat a din profesori, nv at atori si elevi, care fac din entuziasm si cu pasiune o munc a
necompensat a de vreo r asplat a material a. Nu-l vom uita pe Alin Spum a, exemplu
de abnegatie, pasiune si competent a.
n acesti primi ani de existent a, eforturile redactiei s-au dirijat spre cristalizarea
unei identit ati a revistei si dobndirea unui impact favorabil al acesteia cu publicul
interesat de matematica elementar a. ntr-adev ar, revista are n prezent o form a
grafic a de prezentare definitivat a si un continut structurat pe un num ar de rubrici
bine conturat. Pe de alt a parte, punctele de distributie a revistei sunt r aspndite pe
o arie ntins a, iar colaboratorii cu note si probleme originale provin din toat a tara.
Aceste preocup ari sunt n leg atur a strns a cu atingerea obiectivelor revistei, care
constituie n fapt ratiunea aparitiei sale.
Revista se adreseaz a elevilor de la cei mici, din clasele primare, pn a la absol-
ventii liceelor studentilor preocupati de viitoarea lor munc a la catedr a, profesorilor
si tuturor celor ce ndr agesc matematica elementar a.
Prima parte a fiec arui num ar ce ocup a aproape jum atate din spatiul ei este
destinat a articolelor de informare, studiilor si notelor originale, chestiunilor metodice
si din istoria matematicii. A fost publicat n acesti cinci ani un num ar de aproximativ
o sut a de articole de acest fel, care a oferit cititorilor un material variat, pentru toate
nivelurile de preg atire, n bun a parte accesibil si elevilor. Credem c a existenta acestui
spatiu larg de publicare a stimulat posibilit atile creative ale cititorilor nostri.
n scopul cultiv arii gustului pentru matematic a, au fost publicate un num ar de
portrete de matematicieni ilustri (Fermat, Abel, Kolmogorov etc.) sau au fost
prezentate unele probleme celebre (postulatul V al lui Euclid, marea teorem a a lui
Fermat, problema celor patru culori, trisec tia unghiului etc.). Au fost evocate perso-
nalit ati remarcabile ale matematicii romnesti (Spiru Haret, Gh. Vr anceanu etc.),
reviste cu un aport major n cultura stiintic a a t arii (Recrea tii stiin tice, Revista sti-
in tic a "V. Adamachi"), institutii cu un rol important n nv at amntul si cercetarea
romneasc a (Seminarul matematic "Al. Myller", Observatorul astronomic din Ia si ),
85
momente de vrf ale matematicii din tara noastr a (Al V-leaCongres interna tional al
matematicienilor romni ), ct si multe guri de matematicieni ieseni, disp aruti sau
n viat a (I. Creang a, Gh. Gheorghiev, R. Miron, C. Corduneanu etc) si guri
de prestigiu ale nv at amntului liceal (N. Colibaba).
Mention am n mod special rubrica "Nota elevului", care a devenit permanent a
ncepnd cu nr. 1/2001 al revistei si care a g azduit deja 11 articole. Elevii cei mai
buni au n aceast a rubric a un spatiu destinat ncerc arilor lor pe un anume subiect sau
pe o idee propice. La recomandarea redactiei revistei, Funda tia cultural a "Poiana"
(director d-l Dan Tiba) ofer a anual premii n bani tinerilor autori ai celor mai bune
astfel de note. Pn a n prezent au fost premiati un num ar de cinci elevi.
Partea a doua cea mai dinamic a parte a revistei este destinat a concursurilor,
problemelor propuse si solutiilor acestora si se ncheie cu o list a a rezolvitorilor.
Sunt publicate cu regularitate subiectele date la urm atoarele concursuri initiate si
organizate de ieseni: Concursul "Al. Myller" (concurs national, cl. VII-XII), Con-
cursul "F. T. Cmpan" (concurs interjudetean, cl. IV-VIII), Concursul "Recrea tii
matematice" (concurs n cadrul Taberei nationale de matematic a, cl. VII-XI), Con-
cursul "A. Haimovici" (concurs interjudetean pentru liceele economice, industriale
si agricole, cl. IX-XII). Tot cu scopul inform arii elevilor competitivi, sunt prezentate
si concursuri organizate n alte centre: Concursul "R. Miron" (Vaslui), Concursul
"Unirea" (Focsani), Concursul "O. Onicescu" (Botosani) toate ind interjudetene.
Pentru elevii din gimnaziu talentati au fost publicate enunturile si solutiile prob-
lemelor date la OBM (juniori), ct si problemele aflate n atentia juriului acesteia.
Subiectele date la examenul de admitere n cteva facult ati din universit atile
iesene ofer a o orientare candidatilor care opteaz a pentru facult atile respective.
Desigur, n centrul demersului nostru se afl a rubricile "Probleme propuse" si
"Solu tiile problemelor propuse" prin care se urm areste atragerea elevilor pentru
studiul matematicii si o bun a preg atire a celor care reusesc s a devin a rezolvitori
constanti si pasionati. n acesti cinci ani de aparitie, s-au publicat 73 probleme pen-
tru clasele primare si cte 50 probleme pentru fiecare clas a de gimnaziu sau de liceu.
Sunt rezolvate toate problemele propuse dup a un an de la publicarea lor, se dau mai
multe solutii (atunci cnd acestea apar) si se mentioneaz a autorii acestora.
Rubrica "Probleme pentru preg atirea concursurilor", deschis a n nr. 2/2001 al
revistei si mp artit a n dou a nivele (gimnazial si liceal), pune la dispozitia elevilor cu
nclinatii speciale probleme cu un grad de dificultate sporit.
Revista se ncheie cu "Pagina rezolvitorilor", o list a a elevilor rezolvitori de pro-
bleme menit a s a-i ncurajeze si s a-i ambitioneze. Orice elev mentionat de trei ori n
aceast a rubric a primeste din partea redactiei o diplom a si un premiu n c arti.
Printre note si probleme au fost pres arate si un num ar de "Recrea tii . . . mate-
matice" pentru a dovedi c a matematica poate lumina fetele celor pasionati nu numai
cu idei inspirate, ci si cu zmbete.
Dup a acesti cinci ani de aparitie, desprindem o concluzie: s a continu am cu aceeasi
pasiune pentru a ndrepta ceea ce am gresit, a completa ceea ce am f acut multumitor
si a tinde spre perfectiune n acele directii n care am reusit s a facem ceva bun.
Redactia revistei "Recreatii matematice"
86
Alexandru Myller, ctitorul scolii matematice iesene
(3 decembrie 1879 - 4 iulie 1965)
La 3 decembrie a. c. se mplinesc cinci
sferturi de veac de la nasterea lui Alexan-
dru Myller, savant de reputa tie inter-
na tional a si eminent profesor al Univer-
sit a tii din Ia si.
S-a n ascut n Bucuresti, unde a urmat
scoala primar a, liceul (1896) si Facultatea
de Stiinte (1900), avnd ca profesori pe
ilustrii matematicieni romni S. Haret, E.
Pangrati, N. Coculescu si D. Emmanuel.
Dup a un scurt stagiu ca profesor la
Liceul "V. Alecsandri" din Galati, pleac a,
n 1902, la studii la Gttingen, unde a avut
ca profesori pe celebrii matematicieni Felix
Klein si David Hilbert. Prelund creator
noua teorie a lui Hilbert asupra ecuatiilor
integrale, Myller public a un ciclu de lucr ari
printre care si teza de doctorat (1906) ela-
borat a sub ndrumarea lui Hilbert.
Lucr arile lui Myller n domeniul analizei
matematice, elaborate la Gttingen si con-
tinuate la Bucuresti, au marcat, prin ideile,
metodele si rezultatele obtinute, un moment important n dezvoltarea si armarea
matematicii romnesti pe plan national si international. Acestea, precum si titlul de
elev al lui Hilbert, l-au consacrat ca matematician de prim a m arime. La Iasi, Myller
si schimb a directia cercet arilor matematice, trecnd la geometrie, domeniu care l-a
fascinat nc a din tinerete.
A obtinut numeroase rezultate n domeniul teoriei ecuatiilor diferentiale si
integrale prin: extinderea unor rezultate ale lui Hilbert la cazul unor ecua tii diferen-
tiale de ordin arbitrar; ecua tii integrale cu nucleu antisimetric; probleme bilocale, la
limit a si de periodicitate pentru ecua tii diferen tiale ordinare si cu derivate par tiale;
utilizarea metodelor func tionale n rezolvarea unor probleme de fizic a matematic a.
n domeniul geometriei diferentiale contributiile sale se refer a la: geometrie
algebric a si geometrie riglat a; definirea no tiunii de concuren t a a vectorilor contra-
varian ti ca o generalizare a paralelismului Levi Civita; paralelismul ntr-un sistem
de plane, care a condus la no tiunea de congura tie Myller; dezvoltarea, mpreun a cu
O. Mayer a geometriei diferen tiale centroane. Aceste cercet ari au fost declansate si
impulsionate de aparitia teoriei relativit ati generale, unde se folosea n mod consis-
tent teoria conexiunilor afine si alte notiuni geometrice legate de conexiuni. Aceste
preocup ari au marcat momentul intr arii scolii de matematic a de la Ia si pe arena
mondial a a cercet arii stiin tice.
87
n domeniul istoriei matematicii a reusit s a repun a n valoare contributiile
originale ale unor precursori precum D. Asachi, St. Botez, E. Bacaloglu etc.
Numit n 1910 profesor titular la catedra de geometrie analitic a a Universit atii din
Iasi, Alexandru Myller pune bazele nv a t amntului matematic modern la aceast a
institu tie de nv a t amnt prin: fondarea vestitei biblioteci de specialitate (18. X. 1910)
ca fundament al cercet arilor originale; ncadrarea unui corp profesoral de mare va-
loare; atragerea unor tineri cu care creeaz a prima scoal a romneasc a de matematic a,
cunoscut a sub numele de Seminarul Matematic din Ia si; ini tierea n premier a
la Ia si a unor studii de istoria matematicii romne sti si universale; introducerea
unor cursuri libere de specialitate si a lucr arilor de licen t a; aporturi originale n
geometria diferen tial a, care au lansat scoala ie sean a n competi tie interna tional a.
Aceste evenimente s-au petrecut, prima oar a n 1922 cnd au fost publicate lucr arile
legate de paralelismul lui Levi Civita, a doua oar a, n 1933, cnd a ap arut memoriul
de geometrie diferential a centroafin a elaborat n colaborare cu O. Mayer.
Ca profesor, Al. Myller a fost un maestru n arta comunic arii cu studen tii. A
aplicat, pentru prima oar a n tara noastr a, metoda euristic a n predarea matematicii
la nivel universitar. Lectiile sale erau adev arate momente de creatie n care profesorul
ghida pe studenti s a descopere, mpreun a, adev arurile stiintei predate. A mp art asit
cu dragoste si generozitate fiec arei generatii de studenti tot ce a acumulat din punct
de vedere stiintific si metodic.
Sub ndrumarea sa, a fost obtinut n 1920 primul doctorat n matematici pure, la o
universitate romneasc a, de c atre Octav Mayer, iar n 1925 a fost obtinut doctoratul
n matematici de c atre Silvia Creang a, care devine prima femeie doctor n matematici
la o universitate romneasc a.
Dup a r azboi, a functionat ca rector n dicila perioad a 1944-1945, reusind s a
deschid a cursurile universitare n mai 1945, dup a refugiul n Transilvania. A fost
preocupat de reconstructia cl adirii, de revenirea la Iasi a laboratoarelor si bibliote-
cilor, precum si a profesorilor si studentilor.
n 1947 iese la pensie, dar r amne legat de activitatea Seminarului Matematic,
unde continu a cercet arile stiintice, se preocup a de instruirea tinerilor doctoranzi
si de bunul mers al bibliotecii urm arind cu asiduitate obtinerea unor publicatii mai
greu de procurat.
A mai desf asurat variate activit ati si la Institutul de Matematic a de la Filiala Iasi
a Academiei Romne. A continuat s a lucreze pn a la stingerea sa din viat a survenit a
la 4 iulie 1965.
Pentru meritele sale de exceptie, Academia Romn a l-a ales membru ti-
tular n 1938. n 1959 Universitatea Humbold din Berlin i-a decernat titlul
de Doctor Honoris Causa pentru str aduin ti deosebite de a creea o matematic a
romneasc a de sine st at atoare. A primit numeroase distinc tii si decora tii din partea
statului romn.
La mplinirea a 125 ani de la nasterea savantului Alexandru Myller, elevii si
elevii elevilor s ai l omagiaz a n semn de adnc a pretuire pentru opera sa nchinat a
dezvolt arii stiintei si nv at amntului modern n tara noastr a.
Prof. dr. Gheorghe BANTA S
Prof. dr. Vasile OPROIU
88
Henri Poincar
la 150 de ani de la nasterea sa
La pense nest quun clair,
au milieu dune longue nuit,
mais cest cet clair qui est tout.
H. P.
Este anevoie pentru un om obisnuit
s a compare colosii; adncimi neb anu-
ite tulbur a vederea detaliilor si ratiunea
nu reuseste s a completeze o imagine
de nteles. Exist a consensul c a Henri
Poincar a fost ultimul matematician
universal si s-a dovedit genial n tot ce
a ntreprins. Acum, la 150 de ani de la
nasterea sa, z abovim un pic gndind la
el.
Henri Poincar s-a n ascut la
Nancy, pe 29 aprilie 1854, ntr-o distins a
familie de intelectuali. Ambidextru si
miop, a suferit de o slab a coordonare
muscular a. Din scoala elementar a a ex-
celat n compozitii scrise. n 1862 este
nscris la Liceul din Nancy si timp de
11 ani s-a dovedit aici un elev str alu-
cit la toate materiile. Profesorul s au de
matematic a l considera un monstru al
matematicii. La concursurile generale ale liceelor din Franta a cstigat premiile n-
ti. n 1873 este admis la cole Polytechnique pe care o absolv a n 1875, dep asindu-
si clar colegii n domeniile matematicii. Citea mult si variat, realiznd conexiuni
neasteptate si dovedind o memorie vizual a excelent a. si completeaz a cu mult interes
studiile la cole des Mines si profeseaz a ca inginer n timp ce si elaboreaz a lucrarea
de doctorat, sub conducerea stiintic a a lui Charles Hermite, n domeniul ecuatiilor
diferentiale. Devine doctor n matematic a n 1879, convingnd dar si uimind comisia.
Pred a analiz a matematic a 2 ani la Caen, din 1881 primeste catedr a la Facul-
tatea de Stiinte din Paris, din 1886 trece la Sorbona pe Catedra de ecuatiile fizicii
matematice si probabilit ati, pred a si la cole Polytechnique. Lectiile sale acoper a
domenii variate, mereu n schimbare, dar nu sunt usor de urm arit de c atre studenti
din cauza abundentei de idei. Este ales n Academia de Stiin te n 1887; caz unic,
este ales n fiecare din cele cinci sec tii ; n 1906 devine secretar permanent al acesteia.
n 1908 este ales n Academia Francez a al c arei secretar permanent devine n anul
prematurului s au deces, la 17 iulie 1912.
Activitatea sa de creatie stiintic a se desf asura cu precizie: ntre 10 si 12 dimineata,
ntre 5 si 7 dup a amiaz a. Dup a ora 7 se informa. Pornea cte o lucrare f ar a un
89
plan prestabilit, f ar a calcule preliminare; dup a primii pasi urmau natural urm atorii.
Aborda si abandona subiecte din unghiuri de vedere diferite dar subconstientul s au
continua studiile, ntregind imaginea. Dicultatea de a opri cercetarea l determina
s a nu ntreprind a lucr ari importante dup a ora 7 seara, spre a nu-si tulbura somnul.
A dezvoltat studiul func tiilor automorfe dup a o idee ce sustine c a i-a venit f ar a
nici o preg atire prealabil a n momentul cnd urca ntr-un autobuz; folosea n context
complet diferit transform arile din geometria neeuclidian a. Prin lucrarea Analysis
situs, publicat a n 1895, ntemeiaz a domeniul topologiei algebrice n care clasica sa
conjectur a este nc a actual a. Este considerat fondatorul teoriei func tiilor analitice
de mai multe variabile complexe. A adus contribu tii esen tiale n geometria alge-
bric a, dezvoltnd metode ce au permis deduceri directe ale unor rezultate profunde
ce se bazau pe o idee de demonstratie gresit a. n 1901 a rezolvat o problem a cru-
cial a de teoria numerelor: c autarea punctelor de coordonate rationale pe o curb a
algebric a f (x, y, z) = 0 cu coecienti rationali. Geometria hiperbolic a, creat a de
c atre Lobacevski si Bolyai, a devenit de nteles pe modelul de univers imaginat de
c atre Poincar. Este considerat al aturi de Einstein (si Lorentz) fondator al teoriei
relativit a tii.
Retinem atentia cu cteva detalii referitoare la problema celor 3 corpuri. Se
presupun date la un moment initial 3 corpuri date prin trei puncte de mase invariabile
prin pozitia lor, viteze si acceleratii. Se presupune c a asupra lor nu intervine nici
o fort a exterioar a, dar c a ele evolueaz a respectnd legea atractiei universale. Se
cere s a se evalueze comportarea lor n timp. Problema abordat a de c atre Poincar
n 1889 era ct pe ce s a fie compromis a de o eroare comis a de editorul de la Acta
Mathematica. Un intens schimb de scrisori cu Mittag-Leler a l amurit chestiunea
si un memoriu al lui Poincar a ap arut n 1890, fiind considerat act de nastere a
teoriei haosului. Motivul este c a aceast a problem a a celor 3 corpuri revine la un
sistem de ecuatii diferentiale (cu necunoscutele cele 9 functii ce dau coordonatele
punctelor) care nu este stabil. Aceasta nseamn a c a modific ari infinitezimale ale
datelor problemei conduc la modific ari substantiale ale traiectoriilor.
H. Poincar a adus contributii esentiale n numeroase domenii ale matemati-
cilor aplicate: mecanic a cereasc a, cosmologie, mecanic a cuantic a, optic a, electrici-
tate, hidrodinamic a, telegrae, termodinamic a, teoria elasticit a tii, electromagnetism,
capilaritate.
n special, modul diferit de a concepe matematica la Poincar si Hilbert a condus
la filosofii distincte ale matematicii. n opozitie cu punctul de vedere logicist si
formalist, H. Poincar a sustinut un punct de vedere intui tionist: prin logic a
demonstr am, dar prin intui tie creem, logica r amne steril a f ar a a fi fertilizat a de
intui tie.
H. Poincar este un maestru al populariz arii stiintei; c artile sale: Stiin t a si
ipotez a (1902), Valoarea stiin tei (1905), Stiin t a si metod a (1908), Ultimele gnduri
(1908, postum) sunt disponibile si n limba romn a.
La funeraliile sale s-a spus: ... a fost matematician, geometru, filosof,
scriitor, poet al infinitului, bard al stiintelor.
Prof. dr. Dan BRNZEI
90
Trecerea planetei Venus prin fata Soarelui
Planeta Venus, a doua planet a dup a Mercur ca dep artare de Soare, cunoscut a si
sub numele de Luceaf arul de dimineat a sau de sear a, trece din cnd n cnd prin fata
Soarelui (se spune c a n acel moment planeta se afl a n conjunc tie inferioar a). Acest
"din cnd n cnd" reprezint a un interval de timp de cel putin 100 de ani. Pe data
de 8 iunie a acestui an fenomenul se va desf asura din nou, iar noi contemporanii
fenomenului vom avea prilejul s a-l vedem, dac a cerul va fi favorabil observatiilor.
Trecerea lui Venus zeita frumusetii si dragostei la romani, numit a la greci
Afrodita, ca ica lui Zeus si Dionei a prezentat o important a deosebit a n istoria
astronomiei si de ce nu si n istoria civilizatiei. Omul, aceast a creatie extraordinar a a
legilor lumii materiale, prin existenta si natura sa, s-a ntrebat de mult a vreme cum a
ap arut; s-a n ascut aici pe P amnt, a venit de undeva, a fost creat de cineva, care este
locul s au n lumea Universului observabil? Iat a ntreb ari, devenite fascinante timp
de secole, la care astronomia, desi a r aspuns doar partial, totusi a dat r aspunsuri mai
mult sau mai putin conving atoare.
Pozitia P amntului n Univers poate fi determinat a dac a se cunoaste distanta
Pamnt - Soare. Aceast a distant a, m asurat a prin procedee astronomice, este de
149 597 870 km. Ei bine, aceast a distant a a fost ameliorat a treptat prin observarea
timp de cteva secole si atunci cnd Venus trecea prin fata discului solar. Acum
exist a metode ce permit determinarea distantei P amnt - Soare cu precizie de cteva
zeci de kilometri, ns a pentru acele vremuri metoda trecerii a reprezentat un succes
remarcabil de la trecere la trecere.
Nici acum observarea fenomenului nu este lipsit a de interes dac a observatiile se
fac cu suficient a precizie (precizia n timp n determinarea fenomenului, trebuie s a
fie de cel putin o sutime de secund a; important este s a se cunoasc a la fel de precis
pozitia locului din care se fac observatiile).
Trecerea planetei Venus prin fa ta Soarelui are loc o dat a la 115 ani dup a care
urmeaz a o alt a trecere dup a 8 ani. Urm atoarea trecere urmnd s a aib a loc dup a 122
de ani, urmat a de o alt a dup a nc a 8 ani. A sa se succed aceste treceri. Dac a planul
traiectoriei lui Venus ar fi acelasi cu planul orbitei terestre, atunci trecerea lui Venus
ar avea loc dup a fiecare 584 de zile si 22 de ore. Acest interval de timp reprezint a
perioada sinodic a a lui Venus (intervalul dintre dou a pozitii asem an atoare ale planetei
n raport cu Soarele si P amntul).
ntr-o viat a de om, tr airea acestui eveniment nu poate fi dect de dou a ori, o
singur a dat a sau deloc, cu exceptia celor ce au o longevitate mai mare de 115 ani
sau 122 de ani. Penultima trecere vizibil a la noi a avut loc n anul 1874, fenomen
urm arit de profesorul Neculai Culianu, fost decan al Facult atii de Stiinte si apoi
rector ntre anii 1888 - 1898 al prestigioasei universit ati iesene. Ultima traversare a
putut fi observat a dup a 8 ani (1882).
Fenomenul se ncadreaz a n categoria eclipselor de Soare cnd Luna, Mercur sau
Venus si P amntul sunt n linie dreapt a. Spre deosebire de eclipsele de Soare, cnd
Luna interpunndu-se ntre Pamnt si Soare poate s a-l acopere complet, trecerea lui
91
Venus nu are aceast a calitate. Venus fiind mult dep artat a de P amnt (n acel moment
dep artarea va fi de 43 228 162 km) si mic a n raport cu Soarele, va l asa o umbr a ca
un mic disc ntunecat reprezentnd a 33-a parte din diametrul Soarelui.
Intrarea peste discul solar (primul contact exterior) va avea loc la ora 8 h 19 m
47 s ca dup a 19 m si 27 s Venus s a intre complet peste discul solar la ora 8 h 39 m
14 s (primul contact interior). Urm atoarele etape vor fi: ultimul contact interior la
ora 14 h 03 m 39 s, iar ultimul contact exterior va avea loc la ora 14 h 22 m 49 s.
Momentele caracteristice mentionate mai sus au fost calculate de c atre Institutul
de Mecanic a Cereasc a si Calculul Efemeridelor din Fran ta, pentru Bucuresti. Pe
teritoriul t arii noastre aceste momente vor diferi de la localitate la localitate, cu nu
mai mult de cteva secunde.
NASA a f acut calcule si pentru Ia si si iat a momentele caracteristice: 8 h 19 m
34 s primul contact exterior, 8 h 39 m 11 s primul contact interior, 14 h 03 m
15 s ultimul contact interior si 14 h 22 m 37 s ultimul contact exterior.
nceputul intr arii lui Venus va avea loc prin partea stnga (partea estic a) din
vecin atatea marginii inferioare a discului solar asa cum se poate vedea n desenul al a-
turat. Tot n desen se arat a traiectoria umbrei si momentele caracteristice mentionate
mai sus. Durata fenomenului, de la primul contact interior la ultimul contact interior
92
va fi de 5 h 24 m 25 s.
Pentru observarea fenomenului trebuiesc luate m asuri de protectie a ochilor. Re-
comand am ca atunci cnd se ndreapt a privirea spre Soare s a se foloseasc a o sticl a
afumat a sau o sticl a ce se utilizeaz a la ochelarii pentru sudur a electric a. Cel mai la
ndemn a ar fi folosirea foliei metalizate utilizat a drept ambalaj de c atre vnz atoarele
de flori. Aceast a ultim a protectie trebuie confectionat a din cel putin dou a sau trei
folii suprapuse, functie de sensibilitatea ochilor celor ce vor s a observe fenomenul.
Observatii se pot face si cu o lunet a dotat a cu un ocular prin care s a se proiecteze
Soarele pe un ecran situat la o distant a convenabil a. Imaginea Soarelui ar putea
ajunge la dimensiunile unui cerc cu raza 10 - 15 cm si chiar mai mult. Evident, luneta
trebuie instalat a pe un trepied pentru a-i asigura stablitate si urm arire comod a a
Soarelui. O urm arire asem an atoare a fenomenului poate fi f acut a proiectnd imaginea
Soarelui ca mai sus, folosind un binoclu.
Amatorii care vor s a aduc a contributii la observarea trecerii, avnd n vedere c a
unele observatoare specializate pot s a nu aib a vreme favorabil a observatiilor, urmeaz a
s a tin a seama de urm atoarele: 1) s a posede un ceas cu cronometru si pus la or a dup a
un post de radio national, 2) s a urm areasc a momentul primului contact exterior si
s a-l cronometreze si la fel s a procedeze pentru celelalte trei momente caracteristice
(primul contact interior, al doilea contact interior si al doilea contact exterior). n
plus fat a de aceste recomand ari, trebuie s a apeleze la o cunostint a ce posed a un
aparat numit GPS cu ajutorul c aruia s a se determine coordonatele locului unde a
f acut observatiile (longitudinea si latitudinea geografic a). Dac a m asur atorile vor fi
f acute cu o eroare n timp de plus sau minus 0,1 secunde, iar coordonatele locului vor
fi determinate cu o eroare de maximum 8 metri n jurul punctului unde s-au f acut
m asur atorile, atunci observatiile pot fi luate n consideratie.
Observatiile pot fi transmise n scris la Observatorul Astronomic din Ia si, Aleea
Sadoveanu nr. 5, textul urmnd s a contin a: mijloacele cu care s-au f acut observatiile,
momentele caracteristice m asurate si coordonatele locului.
Operatia, pe plan mondial, este coordonat a de c atre Observatorul European de
Sud cu sediul n Garching (Germania) iar calculele centralizate vor fi f acute la In-
stitutul de Mecanic a Cereasc a si Calculul Efemeridelor cu sediul la Paris.
Urm arirea fenomenului nu este lipsit a de interes dac a avem n vedere raritatea
lui, importanta stiintific a, ncrederea si seriozitatea ce se poate acorda calculelor
astronomice, spre deosebire de ideile avansate de prezic atori si astrologi, care dau
interpret ari apocaliptice unor astfel de fenomene. Din anul 3000 . H. si pn a n
prezent, au avut loc 64 de treceri si alte evenimente astronomice deosebite, dar nu s-a
constatat, n afara unor fenomene naturale izolate, uneori devastatoare (cutremure,
inundatii, r azboaie provocate de om etc.), s a se fi produs acel eveniment apocaliptic
mult proferat de prezic atori.
Iulian BREAHN

A
Membru al Uniunii Astronomice Internationale,
ex-director al Observatorului Astronomic din Iasi
93
Cteva noi aplicatii ale unei idei consacrate
Gabriel DOSPINESCU
1
Nu credem c a exager am spunnd c a 99% din inegalit atile care sunt propuse ca
probleme de concurs pot fi demonstrate direct, aplicnd alte inegalit ati deja con-
sacrate. Exist a ns a o categorie aparte de probleme foarte greu de rezolvat n acest
mod. Aceste probleme au solutii frumoase, bazate pe rationamente indirecte, dar
extrem de eciente.
Vom prezenta n continuare ideea comun a aflat a n spatele tuturor acestor solutii.
S a presupunem c a avem de demonstrat o inegalitate de forma g (x
1
) + g (x
2
) +
+ + g (x
n
) 1 n ipoteza c a variabilele x
1
, x
2
, . . . , x
n
veric a o relatie de tipul
f(x
1
) +f(x
2
) + +f(x
n
) = 1.
Presupunem prin reducere la absurd c a g (x
1
) +g (x
2
) + +g (x
n
) < 1. Notnd
S = g (x
1
) + g (x
2
) + + g (x
n
), obtinem c a S =
1
k
pentru un anumit k > 1.
Punem kg(x
i
) = a
i
, i = 1, n, rezolv am ecuatiile n x
i
si introducem aceste valori
ca functii de a
i
n relatia f(x
1
) + f(x
2
) + +f(x
n
) = 1. G asim c a variabilele a
i
,
i = 1, n satisfac de asemenea o relatie de forma h(x
1
) + h(x
2
) + + h(x
n
) = 0
precum si a
1
+a
2
+ +a
n
= 1. Problema se reduce acum la a dovedi c a egalitatea
h(x
1
) + h(x
2
) + + h(x
n
) = 0 este imposibil a, lucru care se realizeaz a deseori
demonstrnd c a h(x
1
) + h(x
2
) + +h(x
n
) > 0 (sau < 0) pentru orice numere a
1
,
a
2
, . . . , a
n
cu sum a 1.
Credem c a noua problem a este adesea mai usoar a dect cea initial a si pentru a
exemplifica acest lucru vom rezolva cteva probleme de concurs utiliznd strategia
de mai sus.
Prima dintre ele, Problema 3, OM China 2003, este faimoas a datorit a dificult atii
sale, fiind de asemenea nrudit a cu o alt a binecunoscut a problem a care va fi discutat a
n cele ce urmeaz a, Problema 2, OIM 2001.
Exemplul 1. Fie x
1
, x
2
, . . . , x
n
(0, /2) astfel ca tg x
1
tg x
2
. . . tg x
n
= 2
n/2
.
Determina ti cel mai mic k
n
pentru care inegalitatea cos x
1
+cos x
2
+ +cos x
n
k
n
este ntotdeauna adev arat a.
Solutie. Substituind tg
2
x
i
= 2a
i
, problema revine la a determina supremumul
expresiei
1

1 + 2a
1
+
1

1 + 2a
2
+ +
1

1 + 2a
n
pentru a
1
, a
2
, . . . , a
n
> 0 si cu
produsul egal cu 1.
Nu vom discuta cazul n = 1, acesta fiind trivial. Pentru n = 2, r amne de g asit
valoarea maxim a a lui
1

1 + 2x
+

x

x + 2
, x > 0. Studiind ceea ce se ntmpl a pentru
x = 1, x si x 0, deducem c a valoarea c autat a este
2

3
. Pentru a dovedi acest
lucru, este suficient s a se studieze monotonia functiei f (x) =
1

1 + 2x
+

x

x + 2
,
1
Student, Facultatea de Matematic a si Informatic a, Bucuresti
94
x > 0, cu ajutorul derivatei.
S a studiem ce se ntmpl a pentru n > 2. Dac a toti a
i
, i = 1, n, sunt egali cu 1,
valoarea expresiei de mai sus este
n

3
. Acum, dac a ncerc am s a "apropiem" ct mai
multi a
i
de 0, observ am c a putem "apropia" de 0 cel mult n1 dintre ei, caz n care
valoarea expresiei va tinde la n 1. Deoarece n 1 >
n

3
, este clar c a va trebui s a
dovedim c a
1

1 + 2a
1
+
1

1 + 2a
2
+ +
1

1 + 2a
n
n 1.
Este usor de v azut c a rationnd ca n problema precedent a ajungem repede ntr-
un impas. Observ am ns a c a este suficient s a demonstr am inegalitatea de mai sus
doar pentru n = 3. De ce acest lucru? Dac a n > 3 si a
1
a
2
a
n
= 1, atunci putem
alege a
i
, a
j
, a
k
al c aror produs este cel putin 1. Atunci
1

1 + 2a
1
+
1

1 + 2a
2
+ +
1

1 + 2a
n
< n3+
1

1 + 2a
i
+
1
p
1 + 2a
j
+
1

1 + 2a
k
.
Dar
1

1 + 2a
i
+
1
p
1 + 2a
j
+
1

1 + 2a
k

1

1 + 2a
i
+
1
p
1 + 2a
j
+
1
q
1 + 2
1
a
i
a
j
2,
presupunnd c a inegalitatea este adev arat a pentru n = 3, iar din relatia precedent a
rezult a c a ea este adev arat a si pentru n oarecare.
S a dovedim deci inegalitatea pentru n = 3. Presupunem c a ea nu este adev arat a
pentru o triplet a a
1
, a
2
, a
3
. Atunci
1

1 + 2a
1
+
1

1 + 2a
2
+
1

1 + 2a
3
=
2
p
, cu p < 1.
Notnd x
i
=
p
2

1 + 2a
i
, obtinem c a 1 =
3
Q
k=1

p
2
8x
2
k

1
2

<
3
Q
k=1

1
8x
2
k

1
2

. Este
deci suficient s a demonstr am c a pentru orice 0 < x, y, z <
1
2
cu x +y +z = 1 avem
c a

1
8x
2

1
2

1
8y
2

1
2

1
8z
2

1
2

1, inegalitate echivalent a cu
(1 2x)(1 2y)(1 2z)(1 + 2x)(1 + 2y)(1 + 2z) 8
3
x
2
y
2
z
2
.
O alt a schimbare de variabil a este acum necesar a. Desigur, aceasta este 12x = a,
1 2y = b, 1 2z = c. R amne deci s a demonstr am c a, pentru orice a, b, c > 0 cu
suma 1, avem
8(1 a)
2
(1 b)
2
(1 c)
2
abc(2 a)(2 b)(2 c).
Avem ns a c a
8(1 a)
2
(1 b)
2
(1 c)
2
=8(b +c)
2
(c +a)
2
(a +b)
2
8

8
9
(a +b +c)(ab +bc +ca)

2
=
=
512
81
(ab +bc +ca)
2

512
27
abc (a +b +c) =
512
27
abc.
Este suficient, deci, s a demonstr am c a (2a)(2b)(2c) <
512
27
, ceea ce este trivial,
deoarece (2 a)(2 b)(2 c) < 8, iar problema este acum rezolvat a.
95
A fost mentionat a anterior, n trecere, frumoasa Problem a 2, OIM 2001, propus a
de Hojoo Lee. Problema n cauz a fiind att de mult discutat a si popularizat a,
s-ar putea crede c a nu mai este nimic nou de spus despre ea. Totusi, credem c a
urm atoarea demonstratie a unei generaliz ari a problemei este nou a.
Exemplul 2. Demonstra ti c a dac a a
1
, a
2
, . . . , a
n
sunt numere reale strict pozi-
tive astfel nct a
1
a
2
a
n
= 1, iar k > 1 este un num ar natural, atunci
1
k
p
1 + (n
k
1) a
1
+
1
k
p
1 + (n
k
1) a
2
+ +
1
k
p
1 + (n
k
1) a
n
1.
Vasile Crtoaje
Solutie. Presupunem c a
1
k
p
1 + (n
k
1) a
1
+
1
k
p
1 + (n
k
1) a
2
+ +
1
k
p
1 + (n
k
1) a
n
=
1
p
,
pentru un anumit p > 1. Not am
p
k
p
1 + (n
k
1) a
i
= x
i
, i = 1, n si obtinem c a

n
k
1

n
=
n
Q
i=1

p
k
x
k
i
1

>
n
Q
i=1

1
x
k
i
1

, iar x
1
+x
2
+ +x
n
= 1.
Pentru a ajunge la o contradictie, vom dovedi c a
n
Q
i=1

1
x
k
i
1



n
k
1

n
pentru orice x
1
, x
2
, . . . , x
n
> 0 astfel nct x
1
+x
2
+ +x
n
= 1. Pentru demonstrarea
acestei inegalit ati, s a observ am c a
n
Y
i=1

1
x
k
i
1

=
n
Q
i=1

1 +x
i
+ +x
k1
i

n
Q
i=1
(x
1
+ +x
i1
+x
i+1
+ +x
n
)
(x
1
x
2
. . . x
n
)
k
(1)
si, din inegalitatea mediilor,
n
Y
i=1
(x
1
+x
2
+ +x
i1
+x
i+1
+ +x
n
) (n 1)
n
x
1
x
2
. . . x
n
. (2)
Desigur, pentru minorarea celuilalt produs din formula (1), aplicarea direct a a ine-
galit atii mediilor nu mai este la fel de ecient a. Conform inegalit atii mediilor, avem
n
X
j=1
x
i
j
1 +x
j
+ +x
k1
j

n
n
q
(x
1
x
2
. . . x
n
)
i
n
q
Q
n
j=1

1 +x
j
+ +x
k1
j

, i = 0, k 1.
Sumnd inegalit atile de mai sus, obtinem
n
Y
j=1

1 +x
j
+ +x
k1
j

1 +G+G
2
+ +G
k1

n
, (3)
unde G =
n

x
1
x
2
. . . x
n

1
n
. Din (1), (2) si (3) deducem c a
n
Y
i=1

1
x
k
i
1

(n 1)
n
G
n

1 +G+G
2
+ +G
k1

n
G
nk
,
96
iar pentru a naliza solutia problemei este sucient s a demonstr am c a
1 +G+G
2
+ +G
k1
G
k1
1 +n+ +n
k1
, ceea ce este trivial, deoarece G
1
n
.
Este acum momentul s a discut am o alt a problem a deosebit a, propus a la Concursul
anual al Gazetei Matematice de c atre Vasile Crtoaje. Solutia autorului se bazeaz a
pe aplicarea succesiv a a ctorva identit ati, fiind aproape imposibil de g asit n mod
independent. Credem c a solutia ce urmeaz a este mai natural a din acest punct de
vedere.
Exemplul 3. Demonstra ti c a pentru orice a, b, c, d>0 astfel ca a
2
+b
2
+c
2
+d
2
= 1
este satisf acut a inegalitatea (1 a)(1 b)(1 c)(1 d) abcd.
Solutie. S a presupunem c a
(1 a) (1 b) (1 c) (1 d)
abcd
= p
4
, cu p < 1. Fie
1 a
pa
= x,
1 b
pb
= y,
1 c
pc
= z,
1 d
pd
= t. Atunci 1 =
P 1
(1 +px)
2
>
P 1
(1 +x)
2
,
iar xyzt = 1. Problema se reduce, deci, la a demonstra c a
1
(1 +x)
2
+
1
(1 +y)
2
+
1
(1 +z)
2
+
1
(1 +t)
2
1
pentru orice x, y, z, t > 0 verificnd xyzt = 1.
Profitnd de faptul c a enuntul problemei utilizeaz a 4 numere, vom separa nu-
merele n 2 grupe si demonstra c a
1
(1 +x)
2
+
1
(1 +y)
2

1
1 +xy
si
1
(1 +z)
2
+
1
(1 +t)
2

1
1 +zt
.
Prima inegalitate se reduce la xy (x y)
2
+ (1 xy)
2
0, care este n mod
evident adev arat a, pentru a doua rationndu-se n mod analog. Prin sumarea celor
dou a inegalit ati si tinnd seama de faptul c a xyzt = 1, rezult a concluzia.
n ncheierea acestui articol vom discuta alte dou a probleme deosebite, c arora li se
pot da solutii rapide utiliznd ideile de mai sus. Prima dintre ele a fost publicat a n
revista "American Mathematical Monthly", fiind propus a de c atre Vasile Crtoaje.
Exemplul 4. Fie x
1
, x
2
, . . . , x
n
> 0 astfel nct x
1
+x
2
+ +x
n
=
1
x
1
+
1
x
2
+
+ +
1
x
n
. Demonstra ti c a
1
n 1 +x
1
+
1
n 1 +x
2
+ +
1
n 1 +x
n
1.
Solutie. Presupunem c a
1
n 1 +x
1
+
1
n 1 +x
2
+ +
1
n 1 +x
n
=
1
p
,
p < 1. Not am a
i
=
p
n 1 +x
i
, i = 1, n si deducem c a a
i
<
1
n 1
, i = 1, n, iar
a
1
+a
2
+ +a
n
= 1. Conditia din enunt se transcrie sub forma
n
X
k=1

p
a
k
n + 1

=
n
X
k=1
1
p
a
k
n + 1
.
Deoarece p < 1, putem scrie
n
X
k=1

1
a
k
n + 1

>
n
X
k=1

p
a
k
n + 1

=
n
X
k=1
1
p
a
k
n + 1
>
n
X
k=1
1
1
a
k
n + 1
.
97
R amne, deci, s a demonstr am c a pentru orice a
1
, a
2
, . . . , a
n
>0 cu a
1
+a
2
+ +a
n
= 1
are loc inegalitatea
n
P
k=1

1
a
k
n + 1


n
P
k=1
1
1
a
k
n + 1
, care este echivalent a cu
n
P
k=1
a
k
1 (n 1) a
k

n
P
k=1
1 (n 1) a
k
a
k
.
O alt a substitutie este acum necesar a, anume 1 (n 1)a
k
= b
k
, k = 1, n. Se
observ a c a avem de asemenea b
1
+ b
2
+ + b
n
= 1 si r amne deci s a dovedim
c a
n
P
k=1
1 b
k
b
k
(n 1)
2
n
P
k=1
b
k
1 b
k
, rezultat care se poate obtine prin sumarea a n
inegalit ati de forma
b
k
1 b
k
=
b
k
P
i6=k
b
i

1
(n 1)
2
P
i6=k
b
k
b
i
.
n final vom discuta o alt a frumoas a problem a de concurs, propus a la Barajul de
selec tie a lotului Romniei pentru OIM, 1999, de c atre Gheorghe Eckstein.
Exemplul 5. Demonstra ti c a dac a x
1
, x
2
, . . . , x
n
> 0 satisfac x
1
x
2
. . . x
n
= 1,
atunci
1
n 1 +x
1
+
1
n 1 +x
2
+ +
1
n 1 +x
n
1.
Solutie. Ca mai sus, problema se reduce la a demonstra c a pentru orice x
1
, x
2
, . . . ,
x
n
<
1
n 1
astfel nct x
1
+x
2
+ +x
n
= 1 are loc inegalitatea
n
Q
i=1

1
x
i
n + 1

1.
Cu substitutia b
i
= 1 (n 1) x
i
problema de demonstrat se reduce la
(1 b
1
) (1 b
2
) . . . (1 b
n
) (n 1)
n
b
1
b
2
. . . b
n
.
Conform inegalit atii mediilor,
n
Y
i=1
(1 b
i
) =
n
Y
i=1

X
j6=i
b
j

n
Y
i=1
h
(n 1) n1
s
Y
j6=i
b
j
i
= (n 1)
n
b
1
b
2
. . . b
n
,
ceea ce trebuia demonstrat.
n ncheiere, tin s a multumesc prof. Marian Tetiva pentru lectura acestui articol
si observatiile pretioase f acute, care au contribuit la mbun at atirea formei finale.
E R R A T A
n articolul "Combinatoric a . . . algebric a" de Gabriel Dospinescu, publicat n
nr. 2/2003 al revistei, forma corect a a Lemei de la pag. 19 este:
Fie n num ar natural prim si = cos
2
n
+i sin
2
n
. Are loc egalitatea
a
0
+a
1
+ +a
n1

n1
= 0, a
0
, a
1
, . . . , a
n1
Q,
dac a si numai dac a a
0
= a
1
= = a
n1
.
Diferentele ce apar ne-au fost semnalate de prof. Sergiu Romascu, Vaslui; aces-
tea nu afecteaz a cu nimic restul articolului. De aceast a neglijent a se face vinovat a
redactia si nu autorul articolului.
98
Cteva propriet ati ale medianelor
Temistocle BRSAN
1
1. Ne propunem s a indic am un num ar de propriet ati ale medianelor la nivelul
de ntelegere al unui elev bun de gimnaziu. Instrumentele principale de lucru vor fi
teorema lui Thales si teorema lui Menelaus.
Pentru o exprimare scurt a, vom folosi notiunile de puncte izotomice si puncte
conjugate armonic. Fie A, B, X, Y patru puncte coliniare. Punctele X si Y sunt
izotomice n raport cu A si B dac a sunt simetrice fat a de mijlocul segmentului [AB]
(evident, X si Y sunt fie interioare, fie exterioare acestui segment); aceast a conditie
revine la egalitatea AX = BY . Punctele X si Y sunt conjugate armonic n raport
cu A si B dac a mpart segmentul [AB] n rapoarte egale:
XA
XB
=
Y A
Y B
(evident, X
este interior segmentului si Y exterior sau invers).
Propozitia 1. Fie ABC un triunghi oarecare, A
0
, B
0
, C
0
mijloacele laturilor
[BC], [CA], respectiv [AB] si M, N BC dou a puncte izotomice n raport cu vr-
furile B si C, cu M diferit de mijlocul segmentului [A
0
C]. Atunci, sunt adev arate
arma tiile:
1

dreptele B
0
M si C
0
N se intersecteaz a ntr-un punct X AA
0
;
2

paralela prin M la AC si paralela prin N la AB se intersecteaz a ntr-un punct


X
0
AA
0
;
3

punctele X si X
0
sunt conjugate armonic n raport cu A si A
0
.
Demonstratie. Avem patru situatii distincte ilustrate n figurile de mai jos;
demonstratia este ns a aceeasi.
A A A A
B B B B C
C C C
A
A
A A
B B B B C C C C
X
X
X
X
X
X
X
X
N N
N
N M
M
M M
Deoarece M nu-i mijlocul lui [A
0
C] rezult a c a N nu-i mijlocul lui [A
0
B] si,
ca urmare, dreptele B
0
M si C
0
N intersecteaz a AA
0
. Fie {X} = AA
0
B
0
M si
{X
1
} = AA
0
C
0
N. Conform teoremei lui Menelaus, aplicat a la 4AA
0
C si transver-
sala B
0
M si la 4AA
0
B si C
0
N, avem
XA
XA
0

MA
0
MC

B
0
C
B
0
A
= 1 si
X
1
A
X
1
A
0

NA
0
NB

C
0
B
C
0
A
= 1,
1
Prof. dr., Catedra de matematic a, Univ. Tehnic a "Gh. Asachi", Iasi
99
de unde
XA
XA
0
=
MC
MA
0
si
X
1
A
X
1
A
0
=
NB
NA
0
. Punctele M, N fiind izotomice n raport cu
B si C, avem MA
0
= NA
0
si MC = NB. Rezult a c a
XA
XA
0
=
X
1
A
X
1
A
0
, deci punctul
X
1
coincide cu X. Afirmatia 1

este dovedit a.
Fie X
0
, X
0
1
intersectiile dreptei AA
0
cu paralela prin M la AC, respectiv paralela
prin N la AB. Conform teoremei lui Thales, au loc relatiile:
X
0
A
X
0
A
0
=
MC
MA
0
si
X
0
1
A
X
0
1
A
0
=
NB
NA
0
. Din acestea si din izotomia punctelor M si N fat a de B si C,
deducem egalitatea rapoartelor din membrii din stnga. Ca urmare, X
0
1
coincide cu
X
0
, deci 2

este demonstrat a.
Mai sus s-a ar atat c a
XA
XA
0
=
MC
MA
0
si
X
0
A
X
0
A
0
=
MC
MA
0
, deci X si X
0
mpart [AA
0
]
n acelasi raport, adic a X, X
0
sunt conjugate armonic fat a de A, A
0
. Afirmatia 3

,
deci si propozitia, este demonstrat a.
Observatie. S a urm arim deplasarea punctului X pe AA
0
, atunci cnd M par-
curge BC. Not am cu G centrul de greutate al triunghiului ABC si cu A

mijlocul
medianei [AA
0
]. Dac a M este n C, atunci X coincide cu A. Dac a M se ndep arteaz a
de C, la "dreapta" acestuia, atunci X (AA

). Dac a M se apropie de B, din


"stnga" acestuia, atunci X (A

G). Pentru M situat n B, X coincide cu G. Dac a


M (BA
0
], atunci X (GA
0
]; punctul M n pozitia A
0
coincide cu X. Pentru M
ntre A
0
si mijlocul segmentului [A
0
C], X parcurge semidreapta de origine A
0
ce nu
contine A. n sfrsit, dac a M este ntre mijlocul lui [A
0
C] si C, atunci punctul X
este situat pe semidreapta de origine A ce nu contine A
0
si se apropie de vrful A.
O pozitie particular a interesant a a punctului M este semnalat a n urm atorul
Corolar. Dac a sunt ndeplinite condi tiile din Propozi tia 1 si n plus MC =
= NB =
a
2
, atunci X este izotomicul punctului G n raport cu A si A
0
, iar X
0
este
simetricul lui A
0
fa t a de A.
Demonstratie. Avem X [AA
0
] si
XA
XA
0
=
MC
MA
0
=
a/2
a/2 +a/2
=
1
2
, adic a
XA
0
= 2XA sau XA =
1
3
AA
0
. Cum avem si GA
0
=
1
3
AA
0
, rezult a XA = GA
0
si
prima afirmatie este dovedit a. Pe de alt a parte, X
0
[AA
0
] si
X
0
A
X
0
A
0
=
XA
XA
0
=
1
2
,
deci 2X
0
A = X
0
A
0
, de unde X
0
A = AA
0
. Asadar X
0
si A
0
sunt simetrice fat a de A.
2. Aplicatii. Fie D, D
a
, D
b
, D
c
punctele de tangent a a cercurilor nscris, A-
exnscris, B-exnscris respectiv C-exnscris triunghiului ABC cu dreapta BC. Amin-
tim c a au loc egalit atile [1], p.30:
DB = D
a
C = p b si D
b
C = D
c
B = p a (2p = a +b +c);
primele spun c a punctele D si D
a
sunt izotomice fat a de B si C, iar ultimele c a
aceast a proprietate o au si punctele D
b
si D
c
. Putem presupune c a AB 6= AC
pentru a evita cazul trivial n care 4ABC ar fi isoscel cu vrful A.
100
Propozitia 2. Relativ la punctele D
b
, D
c
, sunt adev arate afirma tiile:
1

B
0
D
b
si C
0
D
c
se intersecteaz a ntr-un punct X (AA

);
2

B
0
D
c
si C
0
D
b
se intersecteaz a ntr-un punct Y (A

G);
3

XA
XA
0
+
Y A
Y A
0
= 2;
4

X este izotomicul lui G n raport cu A si A


0
dac a si numai dac a 2a = b +c.
Demonstratie. 1

si 2

decurg direct din Propozitia 1 si observatia de mai sus.


De asemenea, avem
XA
XA
0
+
Y A
Y A
0
=
D
b
C
D
b
A
0
+
D
c
C
D
c
A
0
=
p a
(p a) +a/2
+
p
p a/2
=
2p a
p a/2
= 2,
adic a are loc 3

. n sfrsit, avnd n vedere Corolarul, X este izotomicul lui G dac a


si numai dac a D
b
C =
a
2
, adic a p a =
a
2
sau 2a = b +c.
A
B
C
A
B C
V
X
D
Y
U
a
D
b
D
c
D
Observatie. Triunghiurile ce satisfac conditia
2a = b+c (o latur a este media aritmetic a a celorlaltor
dou a) sunt speciale, cu multe propriet ati cunoscute
(n [2], p. 242, sunt date opt propriet ati). Armatia
4

indic a o nou a proprietate caracteristic a lor.


Un rezultat similar se obtine dac a lu am punctele
izotomice D, D
a
n locul punctelor D
b
, D
c
.
Propozitia 3. Dac a 4ABC satisface condi tia
|b c| 6=
a
2
, atunci relativ la punctele D, D
a
avem:
1

B
0
D si C
0
D
a
se intersecteaz a n U AA
0
;
2

B
0
D
a
si C
0
D se intersecteaz a n V AA
0
;
3

UA
UA
0

V A
V A
0
= 2 (+ n cazul b > c si n cazul b < c).
Omitem demonstratia, ce urmeaz a pe cea din Propozitia 2, dar mention am c a
prin conditia |b c| 6=
a
2
se evit a ca D sau D
a
s a fie mijlocul segmentului [A
0
C].
Observatie. Punctele X
0
, Y
0
, U
0
, V
0
conjugatele armonic ale punctelor X, Y ,
U, V n raport cu A si A
0
sunt alte patru puncte pe dreapta AA
0
care pot fi puse n
conexiune cu punctele de tangent a D, D
a
, D
b
, D
c
, asa cum se indic a n Propozitia 1.
R amne n seama cititorului examinarea lor.
Bibliografie
1. T. Lalescu - Geometria triunghiului, Ed. Tineretului, Bucuresti, 1958.
2. V. Gh. Vod a - Vraja geometriei demodate, Ed. Albatros, Bucuresti, 1983.
101
O constructie geometric a a mediilor (II)
Claudiu - Stefan POPA
1
n [2] am prezentat o constructie geometric a a mediilor armonic a, geometric a,
aritmetic a, p atratic a si ponderat a a lungimilor bazelor unui trapez, ca segmente cu
capetele pe laturile neparalele ale trapezului si paralele cu bazele lui. Ne propunem
n continuare dezvoltarea acestor idei, fapt ce va conduce la o serie de consideratii
cu interesante aplicatii geometrice.
Propozitie. Fie ABCD un trapez cu AB k CD si punctele A
0
, B
0
, C
0
, D
0
astfel
nct A(A
0
B), B(AB
0
), C(C
0
D), D(CD
0
), iar AA
0
= BB
0
= CC
0
= DD
0
.
Not am {E} = A
0
C BD
0
, {F} = AC
0
B
0
D, {K} = EF AD, {L} = EF BC.
n aceste condi tii, EF k AB, EF = AA
0
, iar KE = FL.
D C
K
E F
L
A B A B
C D
K
Demonstratie. Deoarece AA
0
k CC
0
si
AA
0
= CC
0
, rezult a c a AC
0
CA
0
este paralelo-
gram, deci AC
0
k A
0
C, AC
0
= A
0
C. Analog,
BD
0
k B
0
D, AD k A
0
D
0
si BC k B
0
C
0
, cu egali-
t atile de segmente corespunz atoare. Unghiurile
\
D
0
A
0
E si
\
DAF au laturile respectiv paralele si vor
fi congruente; la fel,
\
A
0
D
0
E
\
ADF. Avem nc a
A
0
D
0
= AD, prin urmare 4A
0
D
0
E 4ADF,
deci A
0
E = AF. ns a AE
0
k AF si atunci AFEA
0
este paralelogram. Rezult a c a EF k AA
0
si EF = AA
0
, primele dou a armatii ale
concluziei.
Fie {K
0
} = EF A
0
D
0
. Deoarece A
0
C si BD
0
sunt diagonale n trapezul A
0
BCD
0
,
iar K
0
L este paralel a la baze prin punctul de intersectie a diagonalelor trapezului,
urmeaz a c a EL = EK
0
. ns a KK
0
= AA
0
+EF, deci ELEF = EK
0
KK
0
, adic a
FL = KE, ceea ce ncheie demonstratia.
Observatie. Concluzia se p astreaz a, cu demonstratie asem an atoare, n cazul n
care punctele A
0
, B
0
, C
0
, D
0
se afl a pe semidreptele [AB, [BA, [CD, respectiv [DC.
n cele ce urmeaz a vom folosi notatiile: AB = a, CD = b, AA
0
= x. Studiem pro-
blema variatiei lungimii segmentului [KL] functie de x; vom gndi lungimea segmen-
tului [AA
0
] ca ind pozitiv a n cazul n care A (A
0
B) si negativ a pentru A
0
(AB.
Pentru x = 0, avem KL =
2ab
a +b
(= m
h
), iar pentru x =

ab (= m
g
), avem
KL = m
g
, dup a cum s-a demonstrat n [2]. Desennd figurile pentru cteva valori
ale lui x > 0, observ am c a segmentul [KL] "coboar a" pe m asur a ce x creste, f ar a a
"atinge" ns a linia mijlocie a trapezului. n momentul n care vom demonstra riguros
acest lucru, vom avea o (probabil) nou a demonstratie pentru inegalitatea mediilor a
dou a numere reale pozitive.
n trapezul A
0
BCD
0
, segmentul [K
0
L] are ca lungime media armonic a a bazelor
A
0
B = a +x si CD
0
= b +x, deci K
0
L =
2 (a +x) (b +x)
a +b + 2x
.
1
Profesor, Scoala "Alecu Russo", Iasi
102
Atunci, avem
KL = K
0
LKK
0
=
2(a +x)(b +x)
a +b +ax
x =
x(a +b) + 2ab
2x + (a +b)
=
m
a
x +m
2
g
x +m
a
= m
a
x +m
h
x +m
a
.
O x
y
a
m
a
m
g
m
g
m
h
m
h
m
g
m
g
m
Suntem astfel condusi la studiul functiei
f : R\ {m
a
} R, f (x) = m
a

x +m
h
x +m
a
.
Aceasta este o functie omograc a, strict
cresc atoare pe (, m
a
) si pe (m
a
, +),
al c arei grafic este o hiperbol a de asimptote
y = m
a
si x = m
a
. Prin calcul, stabilim c a
valorile x = m
g
sunt puncte fixe ale functiei.
Deoarece f ([0, )) = [m
h
, m
a
), iar restrictia
lui f la [0, ) este strict cresc atoare, justi-
carea armatiilor anterioare este complet a.
Consideratiile precedente conduc la urm a-
toarele interpret ari geometrice:
1. Faptul c a f (m
h
) = 0 arat a c a, dac a ABCD este trapez cu AB k CD,
A
0
[AB, D
0
[DC sunt astfel nct AA
0
= DD
0
= m
h
, iar {E} = A
0
C BD
0
,
atunci paralela prin E la bazele trapezului dat trece prin punctul de concurent a al
prelungirilor laturilor neparalele ale acestuia.
2. Faptul c a f (m
g
) = m
g
se interpreteaz a geometric astfel: dac a ABCD este
un trapez cu AB k CD, iar A
0
[AB, D
0
[DC sunt astfel nct AA
0
= DD
0
= m
g
,
atunci A
0
C, BD
0
si AD sunt concurente ntr-un punct E, iar paralela prin E la baze
intersecteaz a BC n F astfel nct EF este media geometric a a lungimilor bazelor.
3. n sfrsit, s a observ am c a pentru x =
a +kb
1 +k
, obtinem f (x) =
a kb
1 k
(calculul se efectueaz a cu usurint a). Prin urmare, considernd segmentul [AA
0
] de
lungime egal a cu media ponderat a a bazelor cu ponderile 1 si k, segmentul [KL] va
reprezenta media ponderat a a bazelor, cu ponderile 1 si k. Acest fapt ne permite
s a construim cu rigla si compasul conjugatul armonic al unui punct; cititorul poate
dezvolta singur ideile.
Bibliografie
1. L. Constantinescu - O interpretare geometric a a inegalit a tii mediilor, R.M.T. -
1/1982, 30.
2. C. - St. Popa - O construc tie geometric a a unor medii, Rec. Mat. - 2/2003, 13-14.
103
O generalizare a teoremelor de baz a
ale calculului diferential
Florin POPOVICI
1
Nota de fat a si propune s a extind a teoremele de baz a ale calculului diferential prin
impunerea conditiei de derivabilitate bilateral a n locul conditiei clasice de deriva-
bilitate. Rezultatele obtinute se exprim a sub forma unor conditii de apartenent a,
care pot fi, ns a, interpretate geometric.
Teorema 1 (Teorema lui Fermat generalizat a). Fie f : [a, b] R o func tie dat a.
Dac a c (a, b) este un punct de extrem local al func tiei f si aceasta este derivabil a
bilateral (la stnga si la dreapta) n punctul c, atunci
0

min

f
0

(c) , f
0
+
(c)

, max

f
0

(c) , f
0
+
(c)

. (1)
Demonstratie. Fie c punct de maxim local. Asadar, > 0 astfel nct avem:
x (c , c) [a, b] f (x) f (c)
f (x) f (c)
x c
0 f
0

(c) 0,
x (c, c +) [a, b] f (x) f (c)
f (x) f (c)
x c
0 f
0
+
(c) 0
si, ca urmare, 0

f
0
+
(c) , f
0

(c)

, adic a are loc (1).


y
x
0
x
Exemplu. Functia f (x) = max

x, 1 x
2

, x 0, este
derivabil a cu exceptia punctului x
0
=

5 1
2
, pentru care
f
0

(x
0
) = 1

5 si f
0
+
(x
0
) = 1. Relatia (1) revine la 0

5, 1

, adic a semitangentele la grafic n M


0
(x
0
, x
0
) sunt
de p arti diferite fat a de paralela prin acest punct la axa Ox.
Teorema 2 (Teorema lui Rolle generalizat a). Dac a f : [a, b] R este o func tie
continu a pe [a, b], derivabil a bilateral pe (a, b) si f (a) = f (b), atunci exist a un punct
c (a, b) astfel nct are loc rela tia (1).
Demonstratie. Conform teoremei lui Weierstrass, functia f este m arginit a si si
atinge marginile, adic a c
1
, c
2
[a, b] astfel nct f (c
1
) f (x) f (c
2
), x [a, b].
Dac a {c
1
, c
2
} {a, b}, atunci rezult a c a f este functie constant a si (1) are loc
pentru orice c (a, b). Dac a {c
1
, c
2
} 6 {a, b}, fie c {c
1
, c
2
} \ {a, b}; conform
Teoremei 1 pentru acest punct c are loc (1).
Teorema 3 (Teorema lui Lagrange generalizat a). Dac a f : [a, b] R este
o func tie continu a pe [a, b] si derivabil a bilateral pe (a, b), atunci exist a un punct
c (a, b) astfel nct
f (b) f (a)
b a

min

f
0

(c) , f
0
+
(c)

, max

f
0

(c) , f
0
+
(c)

. (2)
Demonstratie. Se aplic a Teorema 2 functiei auxiliare g : [a, b] R definite prin
g (x) = f (x)
f (b) f (a)
b a
x, x [a, b].
Corolarul 1 (Teorema lui Lagrange pentru func tii convexe). Fie f : I R ( I
interval deschis) o func tie convex a. Atunci, pentru orice puncte a, b I, a < b,
exist a c (a, b) astfel nct
1
Profesor, Liceul Teoretic "N. Titulescu", Brasov
104
f
0

(c)
f (b) f (a)
b a
f
0
+
(c) .
Demonstratie. Deoarece f este convex a pe I (deschis), rezult a c a f este continu a
pe I, derivabil a bilateral pe I si f
0

(x) f
0
+
(x), x I. Prin aplicarea Teoremei 3
pe intervalul [a, b], obtinem rezultatul cerut.
Corolarul 2. Fie f : I R ( I interval deschis) o func tie continu a si deriva-
bil a bilateral pe I. Atunci func tia f este cresc atoare pe I dac a si numai dac a este
satisf acut a condi tia
min

f
0

(x) , f
0
+
(x)

0, x I. (3)
Demonstratie. Necesitatea conditiei este usor de dovedit. Pentru suficient a, fie
x
1
, x
2
I cu x
1
< x
2
. Conform Teoremei 3, aplicat a restrictiei functiei f la [x
1
, x
2
],
c (x
1
, x
2
) astfel nct
f (x
2
) f (x
1
)
x
2
x
1
min

f
0

(c) , f
0
+
(c)

.
De aici si din (3), deducem c a f (x
1
) f (x
2
); ca urmare, functia f este cresc atoare.
Teorema 4 (Teorema lui Cauchy generalizat a). Dac a f, g : [a, b] R sunt dou a
func tii continue pe [a, b], derivabile bilateral pe (a, b) si
0 /

min

g
0

(x) , g
0
+
(x)

, max

g
0

(x) , g
0
+
(x)

, x (a, b) , (4)
atunci g (a) 6= g (b) si exist a un punct c (a, b) astfel nct
f (b) f (a)
g (b) g (a)

min

f
0

(c)
g
0

(c)
,
f
0
+
(c)
g
0
+
(c)

, max

f
0

(c)
g
0

(c)
,
f
0
+
(c)
g
0
+
(c)

. (5)
Demonstratie. Dac a am avea g (a) = g (b), atunci, conform Teoremei 2, ar exista
un punct c (a, b) astfel nct 0

min

g
0

(c) , g
0
+
(c)

, max

g
0

(c) , g
0
+
(c)

, ceea
ce contrazice (4). Deci are loc g (a) 6= g (b).
Consider am acum functia h : [a, b] R definit a prin
h(x) = f (x)
f (b) f (a)
g (b) g (a)
g (x) , x [a, b] .
Observ am c a h este continu a pe [a, b], derivabil a bilateral pe (a, b) si avem
h(a) =
f (a) g (b) f (b) g (a)
g (b) g (a)
= h(b). Conform Teoremei 2, exist a c (a, b) astfel
nct 0

min

h
0

(x) , h
0
+
(x)

, max

h
0

(x) , h
0
+
(x)

, adic a avem
f
0

(c)
f (b) f (a)
g (b) g (a)
g
0

(c) 0 f
0
+
(c)
f (b) f (a)
g (b) g (a)
g
0
+
(c) (6)
sau
f
0
+
(c)
f (b) f (a)
g (b) g (a)
g
0
+
(c) 0 f
0

(c)
f (b) f (a)
g (b) g (a)
g
0

(c) . (7)
Presupunem c a are loc (6) (se procedeaz a analog, dac a ar avea loc (7)). Datorit a
ipotezei (4), putem scrie
min

g
0

(c) , g
0
+
(c)

> 0 (8) sau max

g
0

(c) , g
0
+
(c)

< 0 (9) .
Dac a are loc (8), atunci (6) ia forma
f
0

(c)
g
0

(c)

f (b) f (a)
g (b) g (a)

f
0
+
(c)
g
0
+
(c)
,
deci (5) este adev arat a. Dac a are loc (9), atunci (6) se scrie
f
0
+
(c)
g
0
+
(c)

f (b) f (a)
g (b) g (a)

f
0

(c)
g
0

(c)
,
deci (5) este adev arat a si n acest caz. Demonstratia este complet a.
105
Asupra unei inegalit ati
Alexandru NEGRESCU
1
La a V-a editie a Concursului interjude tean de matematic a "Radu Miron", noiem-
brie 2003, elevilor clasei a IX-a li s-a propus urm atoarea problem a:
Fie x
1
, x
2
, . . . , x
n
(2, +) astfel nct
1
x
1
1
+
1
x
2
1
+ +
1
x
n
1
= 1.
Demonstra ti c a x
1
x
2
. . . x
n
(n + 1)
n
.
Vom da 5 demonstratii acestei inegalit ati.
Solutia I. Not am x
i
1 = a
i
, i = 1, n; deci a
i
> 1. Aplic am inegalitatea lui
Huygens:
(1 +a
1
) (1 +a
2
) (1 +a
n
) (1 +
n

a
1
a
2
a
n
)
n
, a
1
, a
2
, . . . , a
n
0.
Obtinem
x
1
x
2
. . . x
n

1 +
n
p
(x
1
1) (x
2
1) (x
n
1)

n
. (1)
Dar, conform cu inegalitatea dintre mediile armonic a si geometric a, avem
n
1
x
1
1
+
1
x
2
1
+ +
1
x
n
1

n
p
(x
1
1) (x
2
1) (x
n
1)
sau, tinnd seama de conditia din enunt,
n
n
p
(x
1
1) (x
2
1) (x
n
1). (2)
Din (1) si (2) rezult a c a x
1
x
2
. . . x
n
(1 +n)
n
, q.e.d.
Solutia II. Scriem
x
i
= 1 + (x
i
1) = 1 +
x
i
1
n
+
x
i
1
n
+ +
x
i
1
n
(n + 1)
n+1
s

x
i
1
n

n
.
Ca urmare,
x
1
x
2
. . . x
n
(n + 1)
n
n+1
r
(x
1
1)
n
(x
2
1)
n
(x
n
1)
n
n
n
2
. (3)
Din (2), avem (x
1
1) (x
2
1) (x
n
1) n
n
, deci
n+1
r
(x
1
1)
n
(x
2
1)
n
(x
n
1)
n
n
n
2
1. (4)
Combinnd (3) si (4), obtinem inegalitatea cerut a.
Solutia III. Not am
1
x
i
1
= y
i
, i = 1, n; deci y
i
(0, 1) si y
1
+y
2
+ +y
n
= 1.
Rezult a c a x
i
=
y
i
+ 1
y
i
, 1, n si avem:
x
1
x
2
. . . x
n
=
y
1
+ 1
y
1

y
2
+ 1
y
2
. . .
y
n
+ 1
y
n
=
1
Elev, cl. a IX-a, Colegiul National "A. T. Laurian", Botosani
106
=
2y
1
+y
2
+ +y
n
y
1

y
1
+ 2y
2
+ +y
n
y
2
. . .
y
1
+y
2
+ + 2y
n
y
n

(n + 1)
n n+1
q
(y
1
y
2
y
n
)
n+1
y
1
y
2
y
n
= (n + 1)
n
.
Solutia IV. Apel am la metoda lui Sturm. Deoarece x
i
=

1 +
1
x
i
1

1
x
i
1
,
i = 1, n, inegalitatea de demonstrat se scrie
"
n
Y
i=1

1 +
1
x
i
1

n
Y
i=1
1
x
i
1
(n + 1)
n
. (5)
S a analiz am comportarea produsului
1 +a
a

1 +b
b
, cu a, b > 0 si a +b este con-
stant a sububitar a, atunci cnd a si b "se apropie". Presupunem a < b si nlocuim
numerele a si b cu a +t si respectiv b t, unde 0 < t < b a. Atunci
1 +a +t
a +t

1 +b t
b t

1 +a
a

1 +b
b
=
t (1 +a +b) (a b +t)
ab (a +t) (b t)
< 0,
ceea ce arat a c a apropiind numerele a si b produsul
1 +a
a

1 +b
b
descreste.
Dac a printre numerele
1
x
1
1
,
1
x
2
1
, . . . ,
1
x
n
1
exist a dou a inegale, atunci
unul este strict mai mic ca
1
n
si cel alalt este strict mai mare ca
1
n
; fie
1
x
1
1
<
1
n
si
1
x
2
1
>
1
n
. nlocuim
1
x
1
1
si
1
x
2
1
prin
1
n
si respectiv
1
x
1
1
+
1
x
2
1

1
n
.
Suma numerelor
1
n
,
1
x
1
1
+
1
x
2
1

1
n
,
1
x
3
1
, . . . ,
1
x
n
1
r amne aceeasi,
dar membrul stng n (5), obtinut prin aceast a nlocuire, este mai mic. n noul set
de numere avem unul egal cu
1
n
, iar, dac a printre celelalte exist a dou a inegale, se
procedeaz a la fel pn a cnd se obtine un set de numere egale cu
1
n
. n acest caz
membrul stng are valoare minim a, anume,

1 +
1
n

.
1
n

n
= (n + 1)
n
.
Solutia V. Vom dovedi mai nti rezultatul urm ator:
Lem a. Dac a a
i
, b
i
> 0, i = 1, n, atunci
n
v
u
u
t
n
Y
i=1
(a
i
+b
i
)
n
v
u
u
t
n
Y
i=1
a
i
+
n
v
u
u
t
n
Y
i=1
b
i
. (6)
Demonstra tie. Inegalitatea se poate scrie astfel:
1
n
r
a
1
a
1
+b
1
. . .
a
n
a
n
+b
n
+
n
r
b
1
a
1
+b
1
. . .
b
n
a
n
+b
n
107
si rezult a aplicnd inegalitatea mediilor:
1 =
1
n

a
1
a
1
+b
1
+ +
a
n
a
n
+b
n

b
1
a
1
+b
1
+ +
b
n
a
n
+b
n

n
r
a
1
a
1
+b
1
. . .
a
n
a
n
+b
n
+
n
r
b
1
a
1
+b
1
. . .
b
n
a
n
+b
n
.
Lund n (6) a
i
= x
i
1 si b
i
= 1, i = 1, n, obtinem
n

x
1
x
2
. . . x
n

n
p
(x
1
1) (x
2
2) (x
n
1) + 1
(2)
n + 1,
de unde rezult a inegalitatea dorit a.
Observatie. Aceast a problem a poate fi usor generalizat a astfel:
Fie x
1
, x
2
, . . . , x
n
(2, +) astfel nct
1
x
1
1
+
1
x
2
1
+ +
1
x
n
1
= k.
Demonstra ti c a x
1
x
2
. . . x
n

k +n
k

n
.
Bibliografie
1. Gh. Andrei si colab. - Exerci tii si probleme de algebr a pentru concursuri si olim-
piade scolare, Partea I, Constanta, 1990.
2. M. Ganga - Manual pentru clasa a IX-a, Profil M1,M2, Ed. Mathpress, Ploiesti,
2003.
3. M. Ganga - Probleme elementare de matematic a, v. II, Ed. Mathpress, Ploiesti,
2003.
4. D. St. Marinescu, V. Cornea - Dou a inegalit a ti si unele aplica tii ale acestora,
Gazeta Matematic a, CVI (2001), nr. 3, 102-104.
5. I. Nedelcu - Probleme de matematic a pentru liceu, Ed. Mathpress, Ploiesti, 2003.
6. L. Panaitopol, M. Lascu, V. B andil a - Inegalit a ti, Ed. GIL, Zal au, 1996.
1 (Problema de cnt arire a lui Bachet). Care este cel mai mic num ar de
greut ati care pot fi folosite pentru a cnt ari cu o balant a ori num ar ntreg de kilograme
de la 1 la 40?
2. G asiti dou a numere care se scriu n baza 10 numai cu ajutorul cifrei 1 si care
au suma egal a cu produsul lor.
3. Un urs pleac a din brlogul s au 1 km spre sud, se ntoarce si parcurge 1 km
spre est, apoi 1 km spre nord, revenind astfel n punctul de plecare. Ce culoare are
ursul?
Not a. R aspunsurile la aceste probleme se g asesc la p. 110 si la p. 123
108
Asupra problemei VII.41 din RecMat - 2/2003
n nr. 2/2003 al revistei Recrea tii matematice este publicat a urm atoarea problem a,
propus a de elevul Alexandru Negrescu din Botosani:
VII.41. Rezolva ti n N
2
ecua tia
a
b + 1
+
b
a + 1
= 1.
Aceast a problem a a fost apoi propus a elevilor de cl. a VII-a n cadrul Concursului
"Recrea tii matematice", editia a III-a, 2003, Iasi.
Aceste mprejur ari, ct si accesibilitatea problemei, au f acut ca aceasta s a se
bucure de atentia elevilor. Ca rezultat, au fost date mai multe solutii distincte
sau variante ale lor. Nota de fat a colecteaz a aceste solutii. Cititorul va observa
entuziasmul si pasiunea cu care elevii au atacat problema VII.41.
Solutia I (Alexandra Ciofu, elev a, Hrl au). Dac a perechea (a, b) este solutie
a ecuatiei, atunci
a
b + 1
1 si
b
a + 1
1, de unde a b + 1 si b a + 1.
I Dac a a b, rezult a c a 0 a b 1, deci a b {0, 1}. n cazul a b = 0,
urmeaz a a = b si, deoarece (a, b) este solutie,
2a
a + 1
= 1. De aici, obtinem a = 1
si, deci, perechea (1, 1) va fi solutie a ecuatiei date. n cazul a b = 1, nlocuind n
ecuatia dat a pe a cu b + 1 obtinem 1 +
b
b + 2
= 1, deci b = 0. Asadar, n acest caz
obtinem solutia (1, 0).
II Dac a a b, rolurile numerelor a si b se schimb a si (a, b) va fi (1, 1) sau (0, 1).
Rezumnd, multimea solutiilor ecuatiei date este {(0, 1) , (1, 0) , (1, 1)}.
Solutia II (Maria Cr aciun, elev a, Hunedoara). Ecuatia dat a este echivalent a
cu a (a + 1) +b (b + 1) = (a + 1) (b + 1), deci cu a
2
ab +b
2
= 1. Aceasta din urm a
poate fi scris a sub forma a (a b) b (a b) = 1 ab sau (a b)
2
= 1 ab. Asadar,
num arul 1 ab este un p atrat perfect cel mult egal cu 1, dac a perechea (a, b) este
solutie a ecuatiei. Ca urmare, 1 ab {0, 1}, adic a ab {0, 1}.
Dac a ab = 1, atunci perechea (a, b) va fi (1, 1) si aceasta verific a ecuatia.
Dac a ab = 0, atunci a = 0 sau b = 0. Dac a a = 0, nlocuind n ecuatie obtinem
b = 1; perechea (0, 1) verific a ecuatia din enunt. Dac a b = 0, obtinem n mod
asem an ator solutia (1, 0). Dac a a = 0 si b = 0, verific am c a perechea (0, 0) nu-i
solutie.
n concluzie, multimea solutiilor ecuatiei este {(0, 1) , (1, 0) , (1, 1)}.
Solutia III (Bogdan-Alexandru Burican, elev, Hrl au). Ecuatia este echiva-
lent a cu a
2
ab +b
2
= 1 si apoi cu a
2
+b
2
+ (a b)
2
= 2. Rezult a c a a
2
2, deci
a {0, 1}.
Dac a a = 0, din ecuatia dat a obtinem b = 1.
Dac a a = 1, din relatia a
2
ab + b
2
= 1 obtinem b
2
b = 0, de unde b = 0 sau
b = 1.
Prin urmare, perechile (0, 1), (1, 0) si (1, 1) sunt solutiile ecuatiei date.
Solutia IV (Diana Prodan, elev a, Iasi). Ecuatia a
2
ab +b
2
= 1, echivalent a
109
cu ecuatia din enuntul problemei, se pune n forma 3a
2
+ (a 2b)
2
= 4. De aici,
avem 3a
2
4, deci a {0, 1}. Se continu a ca n Solutia III.
Solutia V (Adrian Hamciuc, elev, Iasi). Ca mai sus, obtinem ecuatia
a
2
ab + b
2
= 1. Cum a
2
+ b
2
2ab, rezult a c a ab 1, adic a ab {0, 1}. Se
ncheie ca n Solutia II.
Solutia VI (Diana Timofte, elev a, Iasi). Fie (a, b) o solutie a ecuatiei date, deci
si a ecuatiei a
2
ab +b
2
= 1. Ca urmare, ecuatia b
2
ab

a
2
1

= 0, considerat a
n b, are solutii reale si atunci = a
2
4

a
2
1

0. Rezult a c a 3a
2
4, deci
a {0, 1}. Se continu a ca n Solutia III.
Solutia VII (Alexandru Negrescu, elev, Botosani). Conform inegalit atii
Cauchy-Buniakovski-Schwarz, avem
[(b + 1) + (a + 1)]

a
b + 1
+
b
a + 1

a +

2
,
de unde

a +

2
(b + 1) +(a + 1), adic a a+b+2

ab a+b+2 sau

ab 1.
De aici ab {0, 1} etc.
Observatie. Solutiile prezentate mai sus au comun faptul c a n prima parte se
obtine, cu tehnici diverse de calcul, una dintre relatiile a {0, 1}, a b {0, 1},
ab {0, 1}. n partea a doua sunt utilizate aceste relatii (una dintre ele) pentru
determinarea multimii solutiilor ecuatiei.
Solutia Problemei de cnt arire a lui Bachet (p. 108)
Dac a greut atile se pun pe un singur taler, sunt necesare sase greut ati: 1 kg, 2 kg,
4 kg, 8 kg, 16 kg, 32 kg. ntr-adev ar, orice greutate de la 1 la 40 poate fi atins a
astfel:
1 = 1, 6 = 4 + 2,
2 = 2, . . . . . . . . . . . . . . .
3 = 2 + 1, . . . . . . . . . . . . . . .
4 = 4, 39 = 32 + 4 + 2 + 1,
5 = 4 + 1, 40 = 32 + 8.
Dac a greut atile pot fi puse pe ambele talere, sunt necesare numai patru greut ati:
1 kg, 3 kg, 9 kg, 27 kg. ntr-adev ar, avem:
1 = 1, 6 = 9 3,
2 = 3 1, . . . . . . . . . . . . . . .
3 = 3, . . . . . . . . . . . . . . .
4 = 3 + 1, 39 = 27 + 9 + 3,
5 = 9 3 1, 40 = 27 + 9 + 3 + 1.
110
Asupra unei probleme de concurs
Dumitru MIHALACHE, Marian TETIVA
1
n aceast a not a ne propunem s a prezent am dou a modalit ati de abordare a unei
probleme de geometrie si s a obtinem o generalizare a sa (rostul ghilimelelor se va
vedea la vremea potrivit a). Problema a fost propus a de C. Apostol la Concursul
Na tional de Matematic a Lauren tiu Duican (ajuns, iat a, la cea de a XI-a editie:
felicit ari si succes n continuare!) si enuntul ei poate fi citit n [1]; iat a acest enunt
(modificat pentru a retine doar esentialul):
Problema 1. n patrulaterul convex ABCD avem m(
b
A) = 80

, m(
b
B) = 120

,
m(
b
C) = 50

, m(
b
D) = 110

. S a se arate c a, dac a (BD este bisectoarea unghiului


b
B,
atunci (AC este bisectoarea unghiului
b
A.
ntr-o prim a faz a, neg asind nici o alt a idee, am ncercat o rezolvare a problemei
bazat a pe calcule trigonometrice. Rezultatul acestei c aut ari este
M A B
P
D
N
C
Metoda I de rezolvare a problemei. Din
ipotez a rezult a c a m(
\
DBA) = m(
\
DBC) =
= 60

si, apoi, c a m(
\
ADB) = 40

, iar
m(
\
BDC) = 70

. S a consider am punctele
M AB (A (MB)) si N (BC) ast-
fel nct triunghiurile MBD si BND s a fie
echilaterale (prin urmare m(
\
MDA) = 20

si
m(
\
NDC) = 10

). Fie a lungimea laturilor


acestor triunghiuri. Cu teorema sinusurilor n
triunghiul MAD obtinem
MA
sin20

=
a
sin100

MA =
a sin20

sin100

=
a sin20

sin80

,
iar din triunghiul NCD,
NC
sin10

=
a
sin50

NC =
a sin10

sin50

.
Atunci
AB = MB MA = a
sin80

sin20

sin80

= a
2 sin30

cos 50

sin80

= a
cos 50

sin80

si
BC = BN +NC = a
sin50

+ sin10

sin50

= a
cos 20

sin50

,
prin urmare
BC
AB
=
cos 20

sin80

sin50

cos 50

=
2 cos 20

sin80

sin100

= 2 cos 20

=
sin40

sin20

(este evident c a am aranjat acest raport, stiind unde vrem s a ajungem).


1
Profesori, Colegiul National "Gh. Rosca Codreanu", Brlad
111
Acum, fie x = m(
\
BAC), deci m(
\
BCA) = 60

x; tot cu teorema sinusurilor


(acum n triunghiul ABC) si tinnd seama de calculul anterior, avem
sinx
sin(60

x)
=
sin40

sin20

sinxsin20

= sin(60

x) sin40

;
transform am produsele n sume si avem
cos (x 20

) cos (x + 20

) = cos (20

x) cos (100

x) ,
deci
cos (100

x) cos (x + 20

) = 0 sin60

sin(40

x) = 0.
Cum, evident, 0

< x < 80

, de aici rezult a x = 40

si problema este rezolvat a.


Desigur, asemenea calcule nu sunt pentru cl. VII-a (iar problema a fost propus a
elevilor acestei clase); s a vedem asadar si o solutie a problemei la acest nivel.
Metoda a II-a se bazeaz a pe observatia urm atoare: dac a P (BC) este punctul
pentru care m(
\
PDC) = 50

, atunci triunghiul PDC este isoscel cu PD = PC


(evident), iar triunghiul ADP este echilateral. ntr-adev ar, unghiul
\
BPD, se vede
imediat, are m asura de 100

, de aceea patrulaterul ABPD este inscriptibil (avnd


dou a unghiuri opuse suplementare). Atunci m(
\
APD) = m(
\
DBA) = 60

, iar, pe de
alt a parte, m(
\
ADP) = m(
\
ADC) m(
\
PDC) = 110

50

= 60

, deci triunghiul
ADP are dou a unghiuri de m asur a 60

.
Rezult a c a avem AP = DP = PC si triunghiul PAC este tot isoscel; prin urmare
m(
[
PAC) = m(
[
PCA) =
180

m(
[
APC)
2
= 20

,
de unde rezult a imediat m(
\
BAC) = 40

si demonstratia se ncheie.
F ar a ndoial a, aceast a a doua variant a de solutionare a problemei este mult mai
simpl a dect prima; totusi, ideea de a alege punctul P nu vine prea usor. n schimb,
rolul s au n problem a este fundamental; de fapt, noi credem c a problema a fost
construit a pornind de la cele dou a triunghiuri isoscele lipite, ADP si CDP. C a
triunghiul ADP este chiar echilateral nu este esential si de asta v a puteti convinge
rezolvnd urm atoarea generalizare a primei probleme.
Problema 1
0
. Fie ABCD un patrulater n care m(
b
A) = +, m(
b
B) = 2+,
m(
b
C) = , m(
b
D) = +, m(
\
DBC) = , unde , , > 0

si 2 + + = 180

.
Atunci:
a) 90

> > .
b) (AC este bisectoarea unghiului
b
A.
Solutie. a) Deoarece m(
b
B) > m(
\
DBC), avem 2+ > si 2 < 2++ = 180

< 90

. Apoi, m(
\
ABD) = 2+ si m(
\
ADB) = 180

(2 + + +) =
= , deci > .
b) Alegem un punct P (BC) astfel nct
m(
\
PDC) = m(
\
PDB) = .
Acest punct exist a pe segmentul (BC), deoarece m(
\
BDC) = + . Mai departe
demonstratia decurge ca mai sus: observati c a triunghiul PCD este isoscel, apoi c a
112
ABPD este patrulater inscriptibil si c a triunghiul PAD este isoscel (cu m(
\
PAD) =
= m(
\
PDA) = ). Rezult a triunghiul PAC isoscel, de unde se va putea calcula
m asura lui
[
PAC si apoi se ajunge la m(
\
BAC) =
+
2
, ceea ce ncheie rezolvarea.
Nu e o generalizare efectiv a, totul se bazeaz a pe aceeasi idee (Problema 1 se
reg aseste pentru = 50

, = 60

si = 20

): iat a de ce am pus cuvntul ntre


ghilimele.
Iar pentru a ncheia l as am ca tem a o problem a nrudit a cu cele de mai sus
(eventual ncercati si o generalizare a ei).
Problema 2. Fie ABC un triunghi cu m(
b
B) = 110

si m(
b
C) = 50

. Conside-
r am punctele M (AC) si N (AB) astfel nct m(
\
MBC) = 70

si m(
\
NCB) = 30

.
S a se calculeze m(
\
AMN). (R aspuns: 60

.)
Bibliografie
1. F. Diac - A XI-a edi tie a Concursului Na tional de Matematic a Lauren tiu Duican,
Brasov, 2003, G. M. 11/2003.
LISTA MEMBRILOR FILIALEI IA SI a S. S. M.
continuare din nr. 1/2000, 1/2001, 1/2002, 1/2003 si 1/2004
126. DIMITRIU Gabriel I. M. F., Iasi
127. R

ADUCANU Petru Liceul "D. Cantemir", Iasi


128. IUREA Gheorghe Liceul "D. Cantemir", Iasi
129. LAZ

AR Cristian Colegiul National, Iasi


130. PETCU Alina Emilia Liceul Energetic, Iasi
131. POPA Gabriel Colegiul National, Iasi
132. V

AT

AM

ANU T

A Laura Scoala Waldorf, Iasi


133. NEDELCU Andrei Liceul "Gr. Moisil", Iasi
134. C

AR

AU SU Alexandru Univ. Tehnic a "Gh. Asachi", Iasi


135. ROMAN Neculai Scoala "V. Alecsandri", Mircesti (Iasi)
136. C

ALIN Ionela Liceul "D. Mangeron", Iasi


137. GOL

AE S Angelica Gr. sc. ind. usoar a "Victoria", Iasi


138. BEJAN Tinuta Scoala "Al. Vlahut a", Iasi
139. BUZAC Gabriela - Tamara Liceul Economic nr. 1, Iasi
140. P

ADURARU Adriana Scoala "B. P. Hasdeu", Iasi


141. LUCHIAN Dorel Liceul "M. Costin", Iasi
142. COZLAC Magda Liceul ind. nr. 7, Iasi
143. LUCA TUDORACHE Rodica Univ. Tehnic a "Gh. Asachi", Iasi
144. POPA Antoaneta Scoala Mnz atesti (Iasi)
145. ARBONE Dorina Scoala "Mircea cel B atrn", Iasi
146. IONESCU Mihaela Scoala "I. Ghica", Iasi
147. SAVA Radu Colegiul "C. Negruzzi", Iasi
(continuare la p. 128)
113
Exponentul num arului natural a n produsul n!
Mihai CR

ACIUN
1
n cele ce urmeaz a, date ind a, n N

, vom indica o formul a de calcul a expo-


nentului lui a n n!, notat exp
n!
(a), mpreun a cu cteva aplicatii.
Teorema 1. Dac a p este prim, atunci exp
n!
(p) =

n
p

n
p
2

n
p
3

+ .
Demonstratie. Mai nti, se observ a c a suma din enunt este finit a si se continu a
pn a cnd termenul curent are partea ntreag a egal a cu 0.
Dintre factorii produsului n!, un num ar de

n
p

vor fi multipli de p. Dintre ei, un


num ar de

n
p
2

vor fi multipli de p
2
, iar dintre acestia din urm a, un num ar de

n
p
3

vor fi multipli ai lui p


3
etc. Suma numerelor indicate va fi exponentul cerut, deoarece
fiecare factor al produsului n! care este multiplu al lui p
m
f ar a a fi si multiplu al lui
p
m+1
se socoteste n modul indicat de m ori, ca multiplu al lui p, p
2
, . . . , p
m
.
Exemplul 1. exp
50!
(3) =

50
3

50
9

50
27

50
81

= 16 + 5 + 1 + 0 = 22.
Teorema 2. Dac a p este prim iar N

, atunci exp
n!
(p

) =

exp
n!
(p)

.
Demonstratia este evident a.
Exemplul 2. exp
50!

3
7

=

exp
50!
3
7

=

22
7

= 3.
Teorema 3 (Legendre). Dac a a N

este descompus n factor primi sub forma


a = p
1
1
p
2
2
p

k
k
, atunci exp
n!
(a) = min{exp
n!
(p
1
1
) , exp
n!
(p
2
2
) , . . . , exp
n!
(p

k
k
)} .
Demonstratia este imediat a.
Exemplul 3.
exp
253!
(108) = min

exp
253!

2
2

, exp
253!

3
3

= min

exp
253!
2
2

exp
253!
3
3

.
Deoarece exp
253!
2 = 246 si exp
253!
3 = 125, urmeaz a c a exp
253!
(108) = 41.
Problema 1. Determina ti cu cte zerouri se termin a 1958!
Solutie. Num arul de zerouri este dat de exp
1958!
(10). Se observ a c a
exp
1958!
(10) = min{exp
1958!
2, exp
1958!
5} = exp
1958!
(5) .
Atunci
exp
1958!
(5) =

1958
5

1958
5
2

1958
5
3

1958
5
4

1958
5
5

= 467,
deci 1958! se termin a cu 467 zerouri.
Problema 2. Demonstra ti c a n! nu se divide cu 2
n
, n N.
Solutie. Fie n N

si fie k N astfel ca 2
k
n < 2
k+1
. Atunci
exp
n!
(2) =
h
n
2
i
+
h
n
2
2
i
+ +
h
n
2
k+1
i

n
2
+
n
2
2
+ +
n
2
k+1
= n

1
1
2k + 1

< n
1
Profesor, Liceul "M. Sadoveanu", Pascani
114
deci exp
n!
(2) < n si n! nu se divide cu 2
n
.
Problema 3. Determinati cu cte zerouri se termin a

5
n
1
4

!, unde n N

.
Solutie. S a not am N =
5
n
1
4
. Ca mai sus, num arul de zerouri este dat de
exp
N!
5. n plus
exp
N!
5 =

5
n
1
4 5

5
n
1
4 5
2

+ +

5
n
1
4 5
k

+ =
=

5
n
5 + 5 1
4 5

5
n
5
2
+ 5
2
1
4 5
2

+ +

5
n
5
k
+ 5
k
1
4 5
k

+ =
=
5
n1
1
4
+
5
n2
1
4
+ +
5
nn
1
4
=
5
n
4n 1
16
,
deoarece
5
k
1
4
N pentru k N

, iar
5
k
1
4 5
k
[0, 1). De aici, N! se termin a cu
5
n
4n 1
16
zerouri.
Problema 4. Factorialul c aror numere se termin a exact n 1000 zerouri?
Solutie. Fie n N un num ar al c arui factorial se termin a exact n 1000 zerouri
si fie k N astfel ca 5
k
n < 5
k+1
. Atunci
h
n
5
i
+
h
n
5
2
i
+ +
h
n
5
k+1
i
= 1000, deci
n
5
+
n
5
2
+ +
n
5
k
=
n
4

1
1
5
k

> 1000 si n > 1000. n plus,


h
n
5
i
< 1000, deci
n < 5005, ceea ce implic a k = 5. Se observ a c a pentru n 4004, n! se termin a n cel
mult

4004
5

4004
5
2

4004
5
3

4004
5
4

4004
5
5

= 999 zerouri, n vreme ce pentru


n4010, n! se termin a n cel putin

4010
5

4010
5
2

4010
5
3

4010
5
4

4010
5
5

=1001
zerouri. Numerele cerute sunt 4005, 4006, 4007, 4008, 4009.
Problema 5. Demonstra ti c a
(2m)! (2n)!
m!n! (m+n)!
N, unde m, n N.
Solutie. Mai nti, se poate demonstra inegalitatea
[x] + [y] + [x +y] [2x] + [2y] , x, y R. (1)
Exponentul unui factor prim p din descompunerea canonic a a num ar atorului este
s
1
=
X
k

n
p
k

m
p
k

m+n
p
k

, n vreme ce exponentul lui p din descom-


punerea canonic a a num ar atorului este s
2
=
X
k

2m
p
k

2n
p
k

. Folosind (1)
obtinem c a s
1
s
2
si deci
(2m)! (2n)!
m!n! (m+n)!
N.
Probleme propuse.
1. Cu cte zerouri se termin a num arul
101001!
2004!
?
2. S a se demonstreze c a numerele A = 111! 222! 333! 444! si B = 555! 666!
777! 888! sunt divizibile cu 10
268
, respectiv 10
715
.
3. Factorialul c aror numere se termin a cu exact 2004 zerouri?
4. Aflati exponentul lui k n

1 +k +k
2
+ +k
n

!, unde k, n N

, k 3.
115
Concursul de matematic a Al. Myller
Editia a II-a, Iasi, martie 2004
Not a (pentru clasele IV-VI ). Toate subiectele sunt obligatorii. Timp efectiv de lucru
90 min. Se acord a din oficiu 30 puncte, cte 6 puncte pentru problemele 1-5, cte 8 puncte
pentru problemele 6-10 si cte 10 puncte pentru problemele 11-15.
Clasa a IV-a
1. Un elev rezolv a fiecare dintre primele 5 probleme ale acestui test n cte 3
minute, iar pe fiecare dintre urm atoarele 5 n cte 5 minute. Cte minute i trebuie
pentru a rezolva una dintre ultimele 5 probleme, presupunnd c a fiecare din ele i
solicit a acelasi timp? (timpul total de lucru este 1h 30 min.)
2. Calculeaz a (100 99) + (98 97) + (96 95) + + (2 1).
3. Calculeaz a (5 + 55 + 555 + 5555 + 55555) : (1 + 1 + 111 + 1111 + 11111) : 5.
4. Cei 41 de elevi ai unei clase urc a n sir pe munte. Mircea observ a c a n fata
lui sunt un sfert dintre colegii s ai. Al ct alea n sir este Mircea?
5. Delia calculeaz a suma cifrelor pe care le afiseaz a ceasul ei digital (de exemplu
la ora 14:28 ea obtine 1 + 4 + 2 + 8 = 15). Care este suma maxim a pe care o poate
obtine?
6. Care sunt ultimele trei cifre ale num arului 1 2 3 4 5 . . . 2004 + 12?
7. Aflati suma dintre demp artit, mp artitor, ct si rest, stiind c a restul este cu
18 mai mic dect ctul, ctul este 25, demp artitul este impar, iar mp artitorul are o
singur a cifr a.
8. Un num ar se mparte la 3 si d a restul 2. Ctul se mparte din nou la 3,
obtinnd restul 2. Noul ct se mparte iar la 3 si g asim ctul 2 si restul 2. Care a
fost num arul initial?
9. La un magazin se aduc 301 kg de mere n l azi de 25 kg si 21 kg. Cte l azi se
folosesc n total?
10. 58 de elevi sunt asezati pe 4 rnduri, fiecare rnd avnd cu 3 elevi mai putin
dect rndul din fata sa. Cti elevi sunt pe ultimul rnd?
11. Dan vrea s a cumpere mingi. Dac a ar cump ara 5, i-ar mai r amne 100 000 lei,
iar dac a ar dori s a cumpere 7, ar mai avea nevoie de 220 000 lei. Ct cost a o minge?
12. Marinarii de pe un vapor au hran a pentru 60 de zile. Ei g asesc pe o insul a
30 de naufragiati si astfel hrana le va ajunge tuturor doar 50 de zile. Cti marinari
erau pe vapor?
13. 12 b aeti si 8 fete sunt membri ai cercului de matematic a. n fiecare s apt amn a,
nc a 2 fete si 1 b aiat sunt acceptati ca membri ai cercului. Cti membri va avea cercul
de matematic a atunci cnd num arul b aietilor va fi egal cu num arul fetelor?
14. Cte numere naturale de patru cifre au ultima cifr a 3?
15. Pe o insul a locuiesc numai arici, serpi si vulpi. Fiecare animal m annc a o
dat a pe zi, astfel nct orice arici m annc a la micul dejun cte un sarpe, orice vulpe
m annc a la prnz cte un arici, iar orice sarpe m annc a la cin a cte o vulpe. La
sfrsitul zilei de miercuri, pe insul a a r amas un singur animal. Cte animale existau
pe insul a luni, nainte de micul dejun?
116
Clasa a V-a
1. S a se calculeze suma 4 + 8 + 12 + + 2000.
2. Determinati numerele naturale x astfel nct multimile A = {2x; 6x + 4; 3x + 5}
si B = {2x 1; 2x + 1; 5x + 6} s a aib a un singur element comun.
3. Care sunt numerele prime a, b, c pentru care a + 10 b + 12 c = 82?
4. Aflati suma cifrelor num arului A = 10
2004
1.
5. S a se determine perechile de numere naturale (x, y) pentru care fractia
15
(x + 1) (y 4)
este echiunitar a.
6. S a se determine x N astfel nct num arul a = 5
x
+ 5
x+1
+ 5
x+2
+ 5
x+3
s a
aib a exact 48 divizori.
7. S a se calculeze suma S =
1
2
+
1
6
+
1
12
+
1
20
+
1
30
+ +
1
360
.
8. Aflati numerele a N pentru care
3a + 16
2a 5
N.
9. Care este suma ultimelor trei cifre ale produsului 1 2 3 . . . 25?
10. S a se afle cel mai mic num ar natural de 2 cifre pentru care suma dintre p atratul
si cubul lui este p atrat perfect.
11. Asezati n ordine cresc atoare numerele a = 2
50
, b = 2
47
+ 2
24
, c = 2
45
+ 2
44
+
+2
23
+ 2
22
, d = 2
48
+ 2
23
.
12. n pester a erau dragoni rosii si dragoni verzi. Fiecare dragon rosu avea 6
capete, 8 picioare si 2 cozi. Fiecare dragon verde avea 8 capete, 6 picioare si 4 cozi.
n total dragonii aveau 44 cozi. Sunt, de asemenea, cu 6 picioare verzi mai putin
dect capete rosii. Cti dragoni rosii sunt n pester a?
13. Dan spal a o masin a n 40 de minute, iar Ionut spal a o masin a n 2 ore. n ct
timp vor sp ala mpreun a 3 masini?
14. Dintre cei 101 de dalmatieni, 56 au o pat a neagr a pe urechea stng a, 25 au
pat a neagr a pe urechea dreapt a, iar 29 au ambele urechi albe. Cti dalmatieni au
pete negre pe ambele urechi?
15. Fie a = 2
214
+ 3
143
si b = 31
43
. Care dintre numere este mai mare?
Clasa a VI-a
1. Dac a
7a 2b
5a + 4b
=
2
15
, atunci
b
a
este ..............................
2. Num arul triunghiurilor din gur a este ..............................
3. Diferenta dintre m asurile suplementului si complemen-
tului aceluiasi unghi este ..............................
4. Rezultatul calculului
1
1 3
+
1
3 5
+
1
5 7
+ +
1
2003 2005
este .........................
5. Dou a unghiuri complementare au raportul m asurilor lor egal cu
0, 2 (4)
0, 4 (2)
; atunci
m asura unghiului mai mare este ..............................
6. Rezultatul calculului

ab0 ba

: 99 este ..............................
7. n cte moduri putem aseza patru persoane ntr-un rnd?
8. Ultima cifr a a num arului 2
2004
+ 3
2004
+ + 9
2004
este ..............................
9. Avem la dispozitie timbre de 4000 lei si de 9000 lei. Pentru a expedia o
scrisoare sunt necesare timbre n valoare de 35000 lei. Num arul total de timbre
117
este ..............................
10. Suplementul unui unghi si complementul s au au m asurile invers proportionale
cu 2 si 5. Suma m asurilor lor este ..............................
11. Num arul maxim de unghiuri n jurul unui punct cu m asuri numere naturale
diferite este ..............................
12. Un produs se scumpeste cu 10% si apoi cu 20%. Acelasi produs se scumpeste
cu 20% si apoi cu 10%. n ce caz pretul final este mai mare?
13. Solutia ecuatiei 1, (1x) + 2, (2x) + + 9, (9x) = 50 este ..............................
14. Cte cifre de 0 are la sfrsit num arul 1 2 3 4 . . . 2004?
15. Pe o tabl a de sah 4 4 se trage o linie dreapt a. Cel mai mare num ar de
p atr atele 1 1 care pot fi t aiate n dou a p arti este ..............................
Not a (pentru clasele VII-XII ). Toate subiectele sunt obligatorii. Timp efectiv de lucru
3 ore. Pentru fiecare subiect se acord a 7 puncte.
Clasa a VII-a
1. a) Demonstrati c a dac a m si n sunt numere naturale, atunci 25
n
7
m
este
divizibil cu 3.
b) Determinati cel mai mic num ar de forma |25
n
7
m
3
m
|, unde m si n sunt
numere naturale nenule.
Marius Ghergu, Slatina
2. Fie a, b dou a numere reale avnd modulele cel putin 2. Demonstrati c a
(a
2
+ 1)(b
2
+ 1) (a +b)(ab + 1) + 5. Cnd are loc egalitatea?
Marius Durea, Iasi
3. n triunghiul ABC se consider a n altimea [CM], M AB, iar N este sime-
tricul punctului M fat a de BC. Paralela prin punctul N la CM intersecteaz a BC
n P si AC n Q.
a) Demonstrati c a MQ AP dac a si numai dac a [AB] [AC].
b) Ar atati cum pot fi obtinute pozitiile punctelor A, B, C atunci cnd cunoastem
doar pozitiile punctelor M, N, P.
Petru R aducanu, Iasi
4. Se consider a un triunghi ABC, n puncte distincte A
1
, A
2
, . . . , A
n
pe latura
(BC), n puncte distincte B
1
, B
2
, . . . , B
n
pe latura (AC) si n puncte distincte C
1
,
C
2
, . . . , C
n
pe latura (AB). Fie M multimea punctelor care se obtin la intersectia
a cel putin dou a din segmentele (AA
i
), (BB
j
), (CC
k
). Determinati num arul minim
si num arul maxim de elemente pe care le poate avea multimea M.
Dan Brnzei, Iasi
Clasa a VIII-a
1. Determinati numerele ntregi a, b pentru care, oricare ar fi x real,
(2a a
2
)x
4
+ (2a + 2b 4)x
3
+ (3b 3)x
2
+ (2b
2
2b)x +b + 2 0.
Gheorghe Iurea, Iasi
118
2. Un zar este un cub de latur a 1, pe fetele c aruia sunt impri-
mate puncte ca n gur a, astfel nct suma numerelor de puncte
de pe fetele opuse s a fie 7. Din opt zaruri alc atuim un cub de
latur a 2.
a) Ce valori poate avea num arul punctelor care sunt vizibile
pe fetele cubului de latur a 2?
b) Este posibil s a asez am zarurile astfel nct oricare dou a fete care sunt n contact
s a aib a un num ar egal de puncte?
***
3. Fie ABCD o piramid a n care AC = BC = 1 si AB = AD = BD = CD =

2.
Determinati distanta de la punctul A la planul (BCD).
***
4. Fie cubul ABCDA
0
B
0
C
0
D
0
de latur a a si puncte M (AB), N (CC
0
),
P (D
0
A
0
), astfel ca AM = CN = D
0
P = x.
a) Calculati MP.
b) Ar atati c a triunghiul MNP are centrul de greutate pe segmentul [B
0
D].
Dan Brnzei
Clasa a IX-a
1. Fie n un num ar natural si numerele reale a, b, c astfel nct a
n
= a + b;
b
n
= b +c si c
n
= c +a. S a se arate c a a = b = c.
Gheorghe Iurea, Iasi
2. Pe laturile [AB], [BC], [CD], [DA] ale unui patrulater convex ABCD se
consider a punctele M, N, P, respectiv Qastfel nct
MA
MB
=
NB
NC
=
PD
PC
=
QA
QD
= k,
unde k 6= 1. S a se arate c a S
[ABCD]
= 2S
[MNPQ]
dac a si numai dac a S
[ABD]
=
= S
[BCD]
.
Petre Asaftei, Iasi
3. Fie ABC un triunghi dreptunghic n A si D un punct apartinnd laturii
[BC]. Bisectoarele unghiurilor ADB si ADC intersecteaz a laturile AB si AC n
punctele M si N. S a se arate c a unghiul dintre dreptele BC si MN are m asura
1
2

m(
b
B) m(
b
C)

dac a si numai dac a D este piciorul perpendicularei din A.


Bogdan Enescu, Buz au
4. S a se determine numerele reale x, x > 1 pentru care
n
p
[x
n
]este ntreg, oricare
ar fi n 2. (Se noteaz a cu [a] partea ntreag a a num arului real a).
Mihai Piticari, C-lung Moldovenesc, Dan Popescu, Suceava
Clasa a X-a
1. Fie A = {1, 2, 3, 4, 5}. S a se determine num arul functiilor f : A A cu
proprietatea c a nu exist a numere distincte a, b, c A astfel nct f(a) = f(b) = f(c).
Adrian Zanoschi, Iasi
2. Se consider a tetraedrul ABCD n care medianele din A n triunghiurile ABC,
ABD si ACD sunt perpendiculare dou a cte dou a. S a se arate c a muchiile din A
sunt egale.
Dinu Serb anescu, Bucuresti
3. Se consider a numerele reale x, y, z cu proprietatea c a cos x+cos y+cos z = 0 si
119
cos 3x+cos 3y +cos 3z = 0. S a se demonstreze inegalitatea cos 2x cos 2y cos 2z 0.
Bogdan Enescu, Buz au
4. Fie a 2 un num ar natural. Consider am multimea A = {

a,
3

a,
4

a,
5

a, . . .}.
a) S a se arate c a A nu contine o progresie geometric a infinit a neconstant a.
b) S a se arate c a pentru orice n 3, exist a n elemente din A care sunt n progresie
geometric a.
Bogdan Enescu, Buz au
Clasa a XI-a
1. a) Fie (x
n
) un sir de numere reale, cu proprietatea |x
n+1
x
n
|
1
2
n
, n N.
Ar atati c a sirul (x
n
) este convergent.
b) S a se construiasc a un sir de numere reale (y
n
), care s a aib a simultan propri-
et atile:
(i) lim
n
(y
n+1
y
n
) = 0; (ii) (y
n
) este m arginit; (iii) (y
n
) este divergent.
Eugen Popa, Iasi
2. Se d a paralelogramul ABCD, cu laturi inegale. Vrful B se proiecteaz a
pe AC n punctul E. Perpendiculara n E pe BD intersecteaz a dreptele BC si
AB n punctele F, respectiv G. S a se arate c a EF = EG dac a si numai dac a
m(
\
ABC) = 90

.
Mircea Becheanu, Bucuresti
3. Fie matricele A, B M
2
(Z) astfel nct AB = BA si det B = 1. S a se arate
c a dac a det(A
3
+B
3
) = 1, atunci A
2
= O.
Mircea Becheanu, Bucuresti
4. Fie f : R R o functie cu proprietatea lui Darboux. S a se arate c a dac a f
este cresc atoare pe R\ Q, atunci f este continu a.
Mihai Piticari, C-lung Moldovenesc
Clasa a XII-a
1. Fie p un num ar prim, p > 3. S a se arate c a ecuatia (x+y)
1
= x
1
+y
1
are
solutii n corpul Z
p
dac a si numai dac a 3 divide p 1.
Mihai Piticari, C-lung Moldovenesc
2. S a se calculeze limita
lim
n
X
1i<jn
ln

1 +
i
n

ln

1 +
j
n

p
n
4
+i
2
+j
2
.
Gabriel Mrsanu si Andrei Nedelcu, Iasi
3. Fie n 3 un num ar impar si A un inel comutativ cu 3n elemente. S a se arate
c a num arul elementelor nilpotente ale lui A este cel mult n. (Elementul a A se
numeste nilpotent dac a a
n
= 0, pentru n > 0 convenabil).
***
4. S a se arate c a, pentru orice num ar natural p,
lim
n
n
p+1
Z
1
0
e
nx
ln(1 +x
p
) dx = p!.
Gheorghe Iurea, Iasi
120
Concursul de matematic a Florica T. Cmpan
Etapa interjudetean a, 8 mai 2004
Not a. Toate subiectele sunt obligatorii. Timp de lucru: cl. a IV-a 90 de minute,
cl. V-VIII 2 ore.
Clasa a IV-a
1. Precizeaz a regula de formare a numerelor urm atoare si identific a cifrele care
lipsesc n ultimul num ar: 2798, 5783, 3574, 7862, 54 .
2. ntr-o urn a sunt bile albe, galbene si rosii. Dac a mp artim num arul bilelor albe
la num arul bilelor galbene obtinem ctul 2 si restul 2. mp artind num arul bilelor rosii
la suma celorlalte obtinem ctul 3 si restul 3. Aflati cte bile sunt n urn a, stiind c a
diferenta dintre num arul bilelor rosii si dublul sumei celorlalte este 17.
3. n timpul unui campionat de sah, doi participanti care jucaser a acelasi num ar
de partide s-au mboln avit si s-au retras, iar ceilalti au continuat turneul pn a la
sfrsit. Este adev arat c a cei doi participanti au ajuns s a joace ntre ei, dac a se stie c a
n total s-au jucat 23 de partide? (Turneul s-a jucat n sistemul "fiecare cu fiecare"
cte o singur a partid a.) Justificati r aspunsul.
Clasa a V-a
1. Aflati vrstele tat alui, ului si nepotului, stiind c a sunt exprimate prin trei nu-
mere prime, iar peste cinci ani vrstele lor vor fi exprimate prin trei numere naturale
p atrate perfecte.
2. Determinati toate numerele de forma abbc, cifrele a, b, c fiind distincte, stiind
c a sunt ndeplinite conditiile:
a) ba se divide cu 13; b) ab este num ar prim; c) c este p atratul unui num ar natural.
Cte solutii are problema?
3. Pe trei jetoane asezate cu fata n jos sunt scrise trei numere naturale nenule
si distincte a c aror sum a este 13. Jetoanele sunt asezate n ordine cresc atoare de
la stnga la dreapta. Ana ridic a, prima, jetonul din stnga, apoi Dan pe cel din
dreapta, iar ultimul, Stefan, pe cel din mijloc, declarnd fiecare, n aceast a ordine,
c a nu are suficiente informatii pentru a descoperi celelalte numere. Dar tu, acum,
poti spune ce num ar a v azut Stefan?
Mihaela Cianga, Iasi
Clasa a VI-a
1. Se consider a multimea A = {1, 2, 3, . . . , 2004}. Determinati:
a) probabilitatea ca, alegnd la ntmplare un element din A, acesta s a fie divizibil
cu 167.
b) probabilitatea ca, alegnd la ntmplare o submultime B 6= a lui A, produsul
tuturor elementelor multimii B s a fie egal cu produsul tuturor elementelor multimii
A\ B.
Ioan Lungu, Vaslui
2. Se consider a triunghiul ABC cu [AB] [AC], m(
\
BAC) 90. Fie D (BC)
astfel nct m(
\
ADC) = 60

si fie punctul E astfel nct D (AE) si [AE] [CD].


S a se determine m(
\
EBD).
Mihai Gavrilut, Roman
121
3. Un pilot de avion parcurge o anumit a distant a cu avionul si efectueaz a urm a-
torul calcul: adun a numerele naturale nenule ce reprezint a distanta (n km) cu viteza
(n km/h) si cu timpul (n h) si obtine 6008. Determinati ct timp a durat zborul.
Cristian Laz ar, Iasi
Clasa a VII-a
1. S a se arate c a oricum am alege 51 de numere naturale distincte de la 1 la 100,
printre ele exist a dou a numere naturale distincte a si b astfel nct a | b.
2. Un casier de banc a distrat, pl atindu-i un cec lui Lucian Georges, a ncurcat
euro cu centii (1 Euro = 100 centi), dndu-i acestuia euro n loc de centi si centi n
loc de euro. Dup a ce si-a cump arat o acadea de 5 centi, Lucian Georges a descoperit
c a i-a mai r amas exact o sum a reprezentnd dublul sumei initiale de pe cec. Care
este suma scris a pe cec?
C at alin Budeanu, Iasi
1m
1m 3. 26 de pisici sunt nchise ntr-un labirint care
are forma din figura al aturat a (fiecare latur a are
lungimea de 1 m). Dac a dou a pisici se afl a la dis-
tant a mai mic a de 1, 5 m se vor zgria! Ar atati c a
oricum am aseza cele 26 de pisici n labirint, m acar
dou a se vor zgria.
Monica Nedelcu, Iasi
Clasa a VIII-a
1. Fie tetraedrul ABCD. Vrfului A i asociem num arul natural n, iar vrfurilor
B, C, D le asociem num arul 0. Numim "mutare" alegerea a dou a vrfuri oarecare si
m arirea numerelor asociate lor cu cte o unitate. S a se arate c a dup a un num ar finit
de "mut ari" putem face ca fiec arui vrf s a-i fie asociat acelasi num ar, dac a si numai
dac a n este num ar par.
Gheorghe Iurea, Iasi
2. Pentru a R consider am functia f
a
: R R, f
a
(x) = ax + 2 a, x R si
fie M
a
simetricul originii fat a de graficul functiei f
a
.
a) Ar atati c a graficele functiilor f
a
trec printr-un punct fix, oricare ar fi a R.
b) S a se arate c a, oricare ar fi a si b R, lungimea segmentului M
a
M
b
este mai
mic a sau egal a cu 2

5.
Gabriel Popa, Iasi
3. Un corp gol n form a de tetraedru regulat este asezat cu o fat a pe p amnt si
sufer a dou a r asturn ari instantanee, consecutive, pe alte dou a fete ale sale. O bil a nee-
lastic a, aflat a initial n vrful ce nu atinge p amntul, se misc a sub actiunea atractiei
p amntului naintea primei r asturn ari si pn a dup a ultima r asturnare, imediat dup a
fiecare r asturnare plecnd spre alt a fat a din punctul n care a ajuns.
a) Ar atati c a bila cade de fiecare dat a (nu se rostogoleste pe fete).
b) Calculati lungimea parcursului bilei, dac a latura tetraedrului are lungimea de
9

6
32
m.
Claudiu- Stefan Popa, Iasi
122
Concursul Traian Lalescu
Editia a V-a, Iasi, 2004
Not a. Fiecare subiect va fi notat cu cinci puncte. Timp de lucru: 2 ore.
1. Calculati: [62 + 8 (24 24 : 4 3)] : 5.
2. Aflati valoarea lui x din egalitatea: 1500 [(409 307) 4 + 315 : 15 x] : 3 =
= 1322.
3. Determinati cel mai mic num ar natural n astfel nct: n+3n+5n+7n+9n >
> 2004.
4. Cte numere de trei cifre au suma cifrelor mai mare sau egal a cu 25?
5. Pe tabl a s-a scris de dou azeci si trei de ori num arul 13 si de treisprezece
ori num arul 23. Cte numere trebuie sterse de pe tabl a pentru ca suma numerelor
r amase s a fie 464?
6. G asiti cel mai mic num ar natural care are suma cifrelor 102.
7. Multimea numerelor naturale diferite de zero se mparte n grupe astfel: (1),
(2, 3), (4, 5, 6), (7, 8, 9, 10), . . . . Cu ce num ar ncepe grupa cu num arul 51?
8. Suma anilor de nastere ai Raluc ai si Elenei este 3961. Stiind c a la data de 8
mai 1998 Raluca era de sase ori mai mare dect Elena, cti ani are ast azi Raluca?
9. De-a lungul unui gard sunt opt tufe de zmeur a. Num arul fructelor de pe dou a
tufe vecine difer a cu 1. Num arul fructelor de pe toate tufele poate fi:
a) 101; b) 213; c) 225; d) 229; e) alt num ar.
10. ntr-o clas a, elevii politicosi sunt de dou a ori mai multi dect cei nepoliticosi.
Num arul fetelor politicoase si al b aietilor nepoliticosi este de dou a ori mai mare dect
cel al b aietilor politicosi si al fetelor nepoliticoase, iar num arul fetelor politicoase este
egal cu num arul tuturor b aietilor. Cti b aieti sunt n clas a, dac a aceasta are ntre 20
si 30 de elevi?
11. mp aratul Rosu las a mostenire celor patru fii herghelia de cai astfel: cel mai
mare ia o treime din herghelie, al doilea ia trei optimi din rest, al treilea trei cincimi
din noul rest, iar mezinul a primit cei 10 cai r amasi. Cti cai are herghelia?
12. Pe distanta de 124 de metri s-au instalat 18 conducte de ap a. Stiind c a n
depozit se g aseau 15 conducte de 6 m si 10 conducte de 8 m, determinati cti metri
m asoar a mpreun a conductele care au mai r amas n depozit.
Solutiile problemelor enuntate la p. 108.
1. Numerele c autate sunt 11 si 1, 1, c aci 11 + 1, 1 = 11 1, 1.
2. Ursul se a a la Polul Nord. El are culoarea alb a.
123
Concursul Adolf Haimovici, editia a VIII-a
pentru liceele economice, industriale si agricole
Faza interjudetean a, Iasi, 8 - 9 mai, 2004
Not a. Toate subiectele sunt obligatorii. Timp de lucru: 2 ore.
Clasa a IX-a
1. a) Se d a graficul functiei f : R R, f(x) = x
2
+ ax + a, din figura 1. Ct
este a?
b) n figura 2 sunt reprezentate graficele a trei trinoame de gradul al doilea. Pot
s a fie acestea urm atoarele trinoame: ax
2
+bx+c, cx
2
+ax+b, bx
2
+cx+a? Aceeasi
ntrebare pentru figura 3.
O
x
y
Figura 1.
O
x
y
Figura 2.
O
x
y
Figura 3.
c) Se consider a trinoamele de gradul al doilea de forma x
2
+ px + q, p, q Z si
p +q = 30. Care dintre ele au r ad acini ntregi?
2. Se d a triunghiul ABC n care AB = 3, AC = 4 si BC = 5.
a) S a se calculeze aria triunghiului ABC.
b) S a se afle raza cercului nscris si raza cercului circumscris triunghiului.
c) S a se calculeze sinB, cos B, sin2B, cos 2B.
d) S a se arate c a cos nB Q, n N.
e) Argumentati c a cos nB 6= 0 si c a exist a o infinitate de numere naturale n cu
proprietatea c a cos nB > 0.
3. a) S a se arate c a (n + 1)
2
(n + 2)
2
(n + 3)
2
+ (n + 4)
2
= 4, n N.
b) S a se arate c a pentru orice k Z, exist a n N si o alegere convenabil a a
semnelor + si astfel nct k = 1
2
2
2
... n
2
.
Clasa a X-a
1. Fie z
1
, z
2
, z
3
numere complexe astfel nct |z
1
| = |z
2
| = |z
3
| = 2004 si
|z
1
+z
2
+z
3
| = 2004. Atunci (z
1
+z
2
)(z
1
+z
3
)(z
2
+z
3
) = 0.
2. Codul Morse, care a fost mult utilizat n comunicatii telegrafice, a utilizat
punctele si liniile pentru a codifica anumite caractere. De exemplu E se codific a
printr-un punct, T printr-o linie, zero prin 5 linii etc. Cte caractere se pot codifica
prin cel mult 5 semnale Morse (puncte sau linii)?
3. Se consider a polinoamele f = 8X
3
6X 1, g = 4X
3
3X.
a) S a se arate c a f nu are r ad acini rationale.
b) S a se afle ctul si restul mp artirii lui f la g.
124
c) Ar atati c a f

cos

9

= 0.
O
x
y
d) Este cos

9
irational? Argumentati.
e) Dac a h Q[X] si h

cos

9

= 0, s a se arate c a h
.
.
. f.
Clasa a XI-a
1. Se d a graficul functiei
f : R R, f(x) = ax
4
x
2
+bx +c.
S a se afle semnele numerelor a, b si c.
2. Fie dreptele d
1
si d
2
perpendiculare n A. Un punct F asezat n interiorul unuia
din unghiurile formate de cele dou a drepte, este situat la distantele 1 si respectiv 8
fat a de acestea. S a se determine lungimea minim a a unui segment [BC] (B d
1
,
C d
2
) care trece prin punctul F.
3. Fie multimile M =
_
_
_
_
_
a b c
c a b
b c a
_
_
| a, b, c N
_
_
_
si
K =

n N | n = a
3
+b
3
+c
3
3abc; a, b, c N

.
a) S a se arate c a dac a A, B M, atunci AB M.
b) Pentru orice m, n K, avem mn K.
c) Exist a o matrice E M cu proprietatea c a
_
_
a b c
c a b
b c a
_
_
= aI
3
+ bE + cE
2
,
a, b, c N?
d) Fie A M, nesingular a, cu d = det A. S a se arate c a
det

. . .

(A

. . .

| {z }
2004
= d
2
2004
si

. . .

(A

. . .

| {z }
2004
= d
2
2004
1
3
A.
a b c d e
a d
b
c e
d b
e
Clasa a XII-a
1. Pe multimea G = {a, b, c, d, e} operatia defineste
o structur a de grup. Completati tabla grupului G.
2. Fie polinoamele f = 8X
3
6X1 si g = 4X
3
3X.
a) S a se arate c a f nu are r ad acini rationale.
b) S a se calculeze ctul si restul mp artirii lui f la g.
c) Ar atati c a f

cos

9

= 0.
d) Este irational num arul cos

9
?
e) Dac a h Q[X] si h

cos

9

= 0, s a se arate c a h
.
.
. f.
3. a) Calculati aria subgraficului functiei f : [1, e] R, f (x) = lnx.
b) Dac a a, b, c sunt numere reale oarecare din intervalul [1, e] astfel nct a b c,
atunci (b a) lna + (c a) lnb + (c b) lnc < 2.
125
Olimpiada Balcanic a de Matematic a (juniori), 2004
Editia a VIII-a, Novi Sad (Serbia si Muntenegru)
A. Problemele de concurs - enunturi si solutii
1. Pentru x, y R, nu simultan nule, s a se demonstreze inegalitatea
x +y
x
2
xy +y
2

2

2
p
x
2
+y
2
.
2. Fie 4ABC isoscel (AC = BC), M mijlocul segmentului [AC], iar [CH]
n altimea din C. Cercul ce trece prin B, C si M intersecteaz a a doua oar a dreapta
CH n Q. S a se afle raza cercului circumscris 4ABC functie de m = CQ.
3. Fie x, y N astfel nct 3x + 4y si 4x + 3y s a fie ambele p atrate perfecte. S a
se arate c a numerele x si y sunt multipli de 7.
4. Se consider a un poligon convex cu n 4 vrfuri. Descompunem arbitrar
poligonul n triunghiuri ale c aror vrfuri sunt printre cele ale poligonului, astfel nct
oricare dou a triunghiuri s a nu aib a puncte interioare comune. Color am cu negru
triunghiurile ce au dou a laturi care sunt si laturi ale poligonului, cu rosu triunghiurile
ce au exact o latur a care este si latur a a poligonului, celelalte triunghiuri l asndu-le
albe. S a se demonstreze c a num arul triunghiurilor negre este cu 2 mai mare dect
num arul triunghiurilor albe.
1. Observ am c a cei doi numitori sunt ntotdeauna pozitivi. Dac a x + y < 0,
inegalitatea este evident a (si strict a). Dac a x +y > 0, se demonstreaz a imediat c a
x +y
p
2 (x
2
+y
2
), x
2
xy +y
2

x
2
+y
2
2
,
prin urmare
x +y
x
2
xy +y
2

p
2 (x
2
+y
2
)
x
2
+y
2
2

x +y
x
2
xy +y
2

2

2
p
x
2
+y
2
.
Egalitate se atinge pentru x = y > 0.
A
B H
Q
O
M
L
K
P
C
2. Fie P si O centrele cercurilor circumscrise tri-
unghiurilor BCM, respectiv ABC, K mijlocul lui
[CM], iar {L} = CH PK. Evident c a O CH si
cum OM si PK sunt perpendiculare pe AC, rezult a
c a OM k PK. Urmeaz a c a LK este linie mijlocie n
4CMO, deci CL = LO.
Deoarece OP BC, avem c a m(
[
LOP) = 90

m(
\
OCB). Apoi, m(
[
OLP) = m(
\
CLK) = 90


m(
[
OCA). ns a
\
OCB
[
OCA, deoarece n altimea
CH este bisectoare n 4ABC isoscel, deci
[
POL
[
PLO si PL = PO. Observ am c a
4PCQ este isoscel cu PC = PQ, prin urmare va rezulta c a CL = OQ.
n concluzie, CL = LO = OQ, deci R = OC =
2
3
CQ =
2
3
m.
126
3. Fie m
2
= 3x + 4y, n
2
= 4x + 3y; atunci m
2
+n
2
= 7 (x +y), deci m
2
+n
2
.
.
. 7.
Un p atrat perfect d a la mp artirea prin 7 unul din resturile 0, 1, 2, 4 si atunci, pentru
ca o sum a de p atrate perfecte s a se divid a cu 7, trebuie ca fiecare n parte s a se divid a
cu 7. ns a, dac a m
2
.
.
. 7, atunci m
2
.
.
. 7
2
si analog n
2
.
.
. 7
2
, adic a 7 (x +y) = m
2
+n
2
.
.
. 49,
deci x+y
.
.
. 7. Urmeaz a c a x = (4x + 3y) 3 (x +y)
.
.
. 7, y = (3x + 4y) 3 (x +y)
.
.
. 7.
4. Not am cu x, y, z num arul triunghiurilor colorate n negru, rosu, respectiv alb;
atunci x + y + z = n 2, acesta fiind num arul triunghiurilor din descompunerea
poligonului. Deoarece fiecare latur a a poligonului este latur a a exact unui triunghi
din descompunere, avem c a 2x + y = n. Combinnd cele dou a relatii, obtinem c a
x z = 2, ceea ce trebuia s a demonstr am.
B. Probleme aflate n atentia juriului - enunturi
1. Fie a, b, p, q N

cu p, q 3, iar a, b relativ prime si de parit ati diferite. S a


se arate c a num arul N = 2a
p
b 2ab
q
nu poate fi p atrat perfect.
2. G asiti toate numerele naturale A care se scriu cu patru cifre n baza 10, cu
proprietatea c a
1
3
A+ 2000 =
2
3
A. (Cu A am notat r asturnatul lui A).
3. G asiti toate numerele naturale n 3 cu proprietatea c a n divide (n 2)!.
4. Pentru a, b, c [1, +), demonstrati inegalitatea
(1 +abc)

1
a
+
1
b
+
1
c

3 +a +b +c.
5. Pentru x, y, z R, demonstrati inegalitatea
x
2
y
2
2x
2
+ 1
+
y
2
z
2
2y
2
+ 1
+
z
2
x
2
2z
2
+ 1
(x +y +z)
2
.
6. Dac a 0 <
a
b
< b < 2a, s a se arate c a
2ab a
2
7ab 3b
2
2a
2
+
2ab b
2
7ab 3a
2
2b
2
1 +
1
4

a
b

b
a

2
.
7. Dou a cercuri C
1
si C
2
sunt secante n A si B. Un cerc C cu centrul n A taie
C
1
n M si P, iar pe C
2
n N si Q, astfel nct N si Q sunt situate de o parte si de
alta a dreptei MP, iar AB > AM. S a se demonstreze c a
\
MBQ
\
NBP.
8. Fie E, F dou a puncte distincte n interiorul paralelogramului ABCD. Deter-
minati num arul maxim posibil de triunghiuri avnd aceeasi arie, cu vrfurile n trei
dintre punctele A, B, C, D, E, F.
9. Fie 4ABC nscris n cercul C. Cercurile C
1
, C
2
, C
3
sunt tangente interior
cercului C n punctele A
1
, B
1
, respectiv C
1
si tangente laturilor [BC], [CA], [AB] n
punctele A
2
, B
2
, respectiv C
2
, astfel nct A, A
1
sunt de o parte si de alta a lui BC
etc. Dreptele A
1
A
2
, B
1
B
2
si C
1
C
2
intersecteaz a a doua oar a cercul C n punctele A
0
,
B
0
, respectiv C
0
. Dac a {M} = BB
0
CC
0
, demonstrati c a m(
\
MAA
0
) = 90

.
10. Fie 4ABC cu m(
b
C) = 90

si punctele D [AC], E [BC]. Spre interiorul


triunghiului construim semicercurile C
1
, C
2
, C
3
, C
4
de diametre [AC], [BC], [CD],
127
respectiv [CE] si fie {C, K} = C
1
C
2
, {C, M} = C
3
C
4
, {C, L} = C
2
C
3
, {C, N} =
C
1
C
4
. Ar atati c a punctele K, L, M, N sunt conciclice.
11. Pe o tabl a dreptunghiular a cu m linii si n coloane, n fiecare c asut a se afl a
scris semnul "". Putem efectua urm atoarele operatii:
(i) schimbarea tuturor semnelor de pe o linie (din "+" n "", iar din "" n "+");
(ii) schimbarea tuturor semnelor de pe o coloan a.
a) Dac a m = n = 100, ar atati c a nu putem obtine 2004 semne "+" utiliznd de
un num ar finit de ori operatiile descrise;
b) Dac a m = 100, g asiti cea mai mic a valoare a lui n > 100 pentru care putem
obtine 2004 semne "+".
(continuarea tabelului din p. 113)
148. PRECUPANU Codrin Colegiul "C. Negruzzi", Iasi
149. DUREA Magdalena Scoala "T. Maiorescu", Iasi
150. BURGHELEA Diana Scoala "T. Maiorescu", Iasi
151. FARCA S Marius Scoala "T. Maiorescu", Iasi
152. BAGHIU Ciprian Liceul "D. Cantemir", Iasi
153. ION Elena Liceul "G. Ibr aileanu", Iasi
154. BUCESCU Dominic Scoala "I. Creang a", Iasi
155. PA SA Narcisa Liceul de art a, Iasi
156. SUFI TCHI Viorica Scoala gen. nr. 26, Iasi
157. POPA Gabriela Scoala gen. nr. 43, Iasi
158. RUSU Virginia Liceul "M. Costin", Iasi
159. D

ASC

ALESCU Diana Liceul "M. Eminescu", Iasi


160. GHERGHELA S Liliana Gr. sc. "Victoria", Iasi
161. FEREN T Olimpia Gr. sc. "Victoria", Iasi
162. LEONTIE S Rodica Liceul "Al. I. Cuza", Iasi
163. PANAINTE Ecaterina Bronia Scoala "Gh. I. Br atianu", Iasi
164. POPA Dumitru Scoala " St. Brs anescu", Iasi
165. C

ARBUNE Ioan Liceul "I. Neculce", Tg. Frumos


166. BRE SUG Constantin Liceul "I. Neculce", Tg. Frumos
167. C

AILEANU Sorin Liceul "I. Neculce", Tg. Frumos


168. DOCA Laurentia Liceul "I. Neculce", Tg. Frumos
169. BRNZIL

A Cristina Liceul "D. Cantemir", Iasi


170. PL

AE SU Dan Scoala normal a "V. Lupu", Iasi


171. CON TU Valentin Scoala "I. Cantacuzino", Pascani
172. OLENIUC Mariana Gr. sc. "D. Mangeron", Iasi
173. ROMIL

A Amalia - Patricia Scoala normal a "V. Lupu", Iasi


174. GR

ADINARU Daniela Gr. sc. "A. Saligny", Iasi


175. ARBONE Ion informatician
176. NECHITA Remus Liceul "M. Costin", Iasi
177. BUJOR Lorena Scoala "I. Teodoreanu", Iasi
(continuare la p. 165)
128
Cercul de matematic a Leonard Euler
organizat la Universitatea Humboldt, Berlin
Probleme pentru clasa a VIII-a
Holger STEPHAN
1
Not a. Rubrica "Corespondente" are ca scop informarea elevilor si profesorilor din
tara noastr a cu privire la activitatea de performant a din alte colturi ale lumii. Pentru
orice "corespondent a", solutiile problemelor vor publicate n num arul urm ator al revistei.
Materialul de fat a a fost obtinut prin str adania d-lui Dan Tiba, cercet ator principal I,
Institutul de Matematic a al Academiei Romne.
1. Patru numere adunate dou a cte dou a dau sumele 4, 7, 9, 14, 16, 19. Care
sunt cele patru numere?
2. Demonstrati c a prin "rotirea c atre dreapta" a unui num ar de 8 cifre, divizibil
cu 73, se obtine tot un num ar divizibil cu 73. (Se spune c a un num ar natural este
"rotit c atre dreapta", dac a ultima cifr a este mutat a n fata primei cifre; exemplu:
1234 4123.)
3. Aflati cifrele necunoscute x, y, z din egalitatea
20 058 473 11! = x00yz0055046400.
4. Un num ar x format din cinci cifre diferite si nenule este divizibil cu 9. Ar atati
c a suma tuturor numerelor de cinci cifre distincte ce se pot forma cu aceste cinci cifre
(inclusiv x) este divizibil a cu 2399976.
5. G asiti toate perechile de numere ntregi x si y care sunt solutii ale ecuatiei
diofantice 2x
2
+ 7xy + 3y
2
= 228.
6. G asiti toate perechile de numere ntregi x si y care sunt solutii ale ecuatiei
diofantice 2x
2
+ 3y
2
= 77.
7. Consider am num arul natural n, 1000 n < 5000. Form am num arul (de 12
sau 13 cifre) obtinut scriind n ordine cifrele lui 3n, 2n si respectiv n. Ar atati c a
acest num ar este divizibil cu 2
8
+ 1.
8. Sase numere prime 7 < p
1
< p
2
< p
3
< p
4
< p
5
< p
6
formeaz a un "sextet
de numere prime", dac a p
2
, p
3
si p
4
, p
5
sunt numere prime gemene (adic a p
3
p
2
=
= p
5
p
4
= 2), iar p
2
p
1
= p
4
p
3
= p
6
p
5
= 4. Demonstrati c a suma lor este
divizibil a cu 630.
9. Este posibil ca suma a sapte p atrate perfecte succesive s a fie un p atrat perfect?
1
Cercet ator dr., Institutul Weierstrass, Berlin (e-mail: stephan@wias-berlin.de)
129
Solutiile problemelor propuse n nr. 2 / 2003
Clasele primare
P.54. Calcula ti a si b dac a 46 a = 36 +a si b 3 = 17 b.
( Clasa I ) nv. Doinita Spnu, Iasi
Solutie. 46 a = 36 + a se scrie 36 + 10 a = 36 + a, de unde 10 a = a si
a + a = 10, deci a = 5. Analog, b 3 = 17 b se scrie b = 3 + 17 b, de unde
b +b = 20, deci b = 10.
P.55. n cte moduri pot fi aranjate n linie dreapt a 9 mingi ro sii si una galben a?
( Clasa I ) Georgiana Ciobanu, elev a, Iasi
Solutie. Toate mingile ocup a 10 locuri. Mingea galben a poate ocupa fiecare loc
din cele 10. Sunt 10 moduri.
P56. Cu cinci ani n urm a, suma vrstelor a trei copii era de 11 ani. Care va fi
suma vrstelor acelora si copii peste 6 ani?
( Clasa a II-a) nv. Rodica Rotaru, Brlad
Solutie. In prezent suma vrstelor copiilor este 11 + 3 5 = 26 ani, iar peste 6
ani aceasta va fi 26 + 3 6 = 44 ani.
P.57. n cte moduri pot fi mp ar ti ti 8 b aie ti n dou a echipe de cte 4 juc atori,
dac a Petru vrea s a fie n echip a cu Mihai si Dan, dar nu vrea s a fie cu Avram?
( Clasa a II-a) Adina Dohotaru, elev a, Iasi
Solutie. n gruparea Petru, Mihai, Dan mai trebuie un singur b aiat. Acesta
poate fi luat din restul b aietilor, cu exceptia lui Avram. Al patrulea elev din echip a
poate fi ales n patru moduri.
P. 58. S a se arate c a suma 1 + 4 + 7 +. . . + 100 mp ar tit a la 3 d a restul 1.
( Clasa a III-a) Alexandru - Gabriel Tudorache, elev, Iasi
Solutie. Suma se scrie 1 + (3 1 + 1) + (3 2 + 1) + . . . + (3 33 + 1). Restul
mp artirii la 3 este acelasi cu restul mp artirii sumei 1 + 1 + 1 +. . . + 1
| {z }
34 ori
la 3, care
este 1.
P.59. Fie a si b dou a numere consecutive. Suma acestor numere mpreun a cu
numerele ob tinute m arind cu 12 fiecare dintre vecinii lor este 939. Care sunt cele
dou a numere?
( Clasa a III-a) nv. Maria Racu, Iasi
a
a
a
a
a
11
11
a
13
13
1
939
1
1
Solutie. Consider am b = a + 1. Folosim metoda gu-
rativ a. Primul num ar este
[939 (11 + 13 + 1 + 1 + 11 + 1 + 13)] : 6 = 888 : 6 = 148.
Al doilea num ar este 148 + 1 = 149.
P.60. Din 16 bile, una este mai grea dect celelalte 15, care au mase egale. Care
este cel mai mic num ar de cnt ariri prin care se poate stabili bila mai grea?
( Clasa a III-a) Carmen Ciolacu, elev a, Iasi
Solutie. Asez am cte 8 bile pe fiecare taler. Dup a prima cnt arire se determin a
grupul de 8 bile care contine bila mai grea. Din acestea asez am cte 3 pe fiecare
taler. Dac a balanta este n echilibru, atunci bila mai grea se afl a n perechea r amas a.
130
Printr-o nou a cnt arire se afl a bila mai grea. Dac a balanta nu este n echilibru,
atunci bila mai grea se afl a ntr-o grupare de 3 bile. Aseznd cte o bil a pe fiecare
taler se determin a bila mai grea. Num arul minim de cnt ariri este 3.
P.61. Suma a dou a numere este un num ar de dou a cifre al c aror produs este 3.
Diferen ta dintre cele dou a numere este 7. Care sunt cele dou a numere?
( Clasa a IV-a) nv. Maria Racu, Iasi
Solutie. Suma celor dou a numere poate fi 13 sau 31. Cum diferenta numerelor
este 7, n primul caz numerele sunt 3 si 10, iar n al doilea numerele sunt 12 si 19.
P.62. Dou a ceasuri au nceput s a func tioneze la aceea si or a. Se constat a c a la
fiecare 30 minute (fa t a de ora exact a) unul r amne n urm a cu un minut iar cel alalt
avanseaz a cu un minut. La un moment dat orele indicate de aceste ceasuri sunt: 18
h 36 min si 19 h 24 min. La ce or a au nceput s a func tioneze?
( Clasa a IV-a) Felicia Amih aiesei, elev a, Iasi
Solutie. Cele dou a ceasuri se abat cu acelasi num ar de minute fat a de ora exact a,
unul prin lips a iar cel alalt prin adaos. n momentul citirii abaterea este (19 h 24 min
18 h 36 min) : 2 = 48 min : 2 = 24 min. Ceasurile au fost citite la ora 19 h 24 min
24 min = 19 h. Num arul de ore n care ceasurile au functionat este 24 : 2 = 12.
Ceasurile au nceput s a functioneze la ora 19 12 = 7.
P.63. Alege un num ar format din trei cifre. Scrie la dreapta lui un num ar format
din dou a cifre. Scoate din num arul format de 99 ori num arul format din trei cifre.
Din rezultat scoate diferen ta dintre num arul de trei cifre si num arul de dou a cifre si
scrie rezultatul. Eu ti ghicesc num arul format din dou a cifre. Cum se explic a acest
lucru?
( Clasa a IV-a) Prof. Petru Asaftei, Iasi
Solutie. Fie abc num arul de trei cifre si xy num arul de dou a cifre. Avem abcxy =
= abc00 + xy = 100 abc + xy, 100 abc + xy 99 abc = abc + xy. n continuare
folosim metoda figurativ a.
n urma efectu arii operatiilor indicate n problem a se obtine dublul num arului de
dou a cifre.
Clasa a V-a
V.41. Fie a num ar natural compus astfel nct dac a p | a, cu p prim, atunci
p + 1 | a. S a se arate c a 12 | a si s a se afle cel mai mare num ar a de trei cifre.
Ciprian Baghiu, Iasi
Solutie. Fie p un divizor prim al lui a, conform enuntului. Atunci p +1 | a, deci
p (p + 1) | a si, cum p (p + 1) este num ar par, deducem c a 2 | a. Din 2 | a rezult a
3 | a si apoi 4 | a; prin urmare 12 | a.
Cel mai mare num ar de trei cifre divizibil cu 12 este 996 = 2
2
3 83; cum 84 - 996,
deducem c a 996 nu este solutie. La fel 984 nu este solutie. Num arul 972 = 2
2
3
5
verific a cerintele problemei.
V.42. Se dau numerele xy, ab scrise n baza 10 astfel nct xy divide ab. S a se
131
arate c a x = y dac a si numai dac a a = b.
Ioan S ac aleanu, Hrl au
Solutie. Fie x = y. Atunci din xx = 11x | ab rezult a c a 11 | ab, deci ab {11, 22,
33, . . . , 99}, adic a a = b.
Fie a = b. Atunci xy | 11a si, cum a este cifr a nenul a, deducem c a xy se divide la
11. Ca mai sus, avem x = y.
V.43. S a se afle cifrele a si b stiind c a a b = cd si a
b
= dc.
Romanta Ghit a si Ioan Ghit a, Blaj
Solutie. Evident, a {2, 3, . . . , 9}. Dac a a = 2, atunci b 5 si cum 2 5 = 10 si
2
5
= 32, 2 6 = 12 si 2
6
= 64, iar 2
7
> 100 rezult a c a nu avem solutii. Dac a a = 3,
urmeaz a b 4 si din 3 4 = 12 si 3
4
= 81, iar 3
5
> 100 deducem c a nici n acest caz
nu avem solutii. Considernd si verificnd toate posibilit atile, obtinem solutia a = 9
si b = 2.
V.44. S a se afle x, y, z Q

+
pentru care x
n
= yz, y
n
= xz, z
n
= xy, cu n N.
N. N. Hrtan, Iasi
Solutie. Avem x
n+1
= y
n+1
= z
n+1
= xyz, deci, n mod necesar, x = y = z.
Conditiile initiale se reduc la x
n
= x
2
, prin urmare, x = y = z = 1 dac a n N \ {2}
si x = y = z Q

+
dac a n = 2.
V.45. Se dau sase urne, unele con tinnd bile. Fie opera tia: se aleg trei urne si
se pune cte o bil a n fiecare dintre ele.
a) Compozi tia urnelor fiind 0, 0, 4, 6, 6, 8, s a se indice o succesiune de opera tii n
urma c arora toate urnele s a con tin a acela si num ar de bile.
b) Compozi tia urnelor fiind 0, 1, 2, 3, 4, 4, s a se arate c a nu exist a o succesiune de
opera tii n urma c arora toate urnele s a con tin a acela si num ar de bile.
Gheorghe Iurea, Iasi
Solutie. a) O succesiune de operatii ce atinge scopul este (0, 0, 4, 6, 6, 8)
(1, 1, 5, 6, 6, 8) (2, 2, 6, 6, 6, 8) (3, 3, 7, 6, 6, 8) (4, 4, 8, 6, 6, 8) (5, 5, 8, 7, 6, 8)
(6, 6, 8, 8, 6, 8) (7, 7, 8, 8, 7, 8) (8, 8, 8, 8, 8, 8).
b) Presupunem c a dup a n operatii urnele contin acelasi num ar m de bile, rezult a
c a num arul total de bile din urne este 6m = 3n + 14, imposibil.
Clasa a VI-a
VI.41. Pe opt cartona se sunt nscrise cte unul din numerele 1, 2, 2
2
, 2
3
, 3,
3
2
, 3
3
, 3
4
. Dac a P (k) este probabilitatea ca, extr agnd dou a cartona se, numerele
ob tinute s a aib a n total k divizori distinc ti, s a se rezolve inecua tia P (k) 1/7.
Dumitru Dominic Bucescu, Iasi
Solutie. Considernd cele 28 de posibilitati de extragere a cartonaselor si num a-
rnd divizorii distincti ai numerelor extrase, obtinem: P (1) = 0; P (2) =
2
28
;
P (3) =
5
28
; P (4) =
8
28
; P (5) =
7
28
; P (6) =
3
28
; P (7) =
2
28
; P (8) =
1
28
si P (k) = 0,
k 9. Prin urmare, multimea solutiilor inecuatiei P (k)
1
7
este S = {3, 4, 5}.
VI.42. Fie x, y, z N pentru care 84x + 91y + 98z = 2002. S a se afle valoarea
maxim a a sumei x +y +z.
Adrian Zanoschi, Iasi
132
Solutie. Relatia din enunt este echivalent a cu 12x + 13y + 14z = 286 sau
12 (x +y +z) = 286 (y + 2z). De aici, rezult a c a x + y + z este maxim dac a
y + 2z este minim si 12 divide pe 286 (y + 2z). Obtinem x + y + z maxim
pentru y + 2z = 10. Valoarea maxim a a expresiei este 23 si se obtine pentru
(x, y, z){(18, 0, 5) ; (17, 2, 4) ; (16, 4, 3) ; (15, 6, 2) ; (14, 8, 1) ; (13, 10, 0)}.
Not a. D-l Titu Zvonaru, Com anesti (Bac au), stabileste valoarea minim a a
sumei x + y + z. ntr-adev ar, dac a x + y + z 19, atunci y + 2z 57 si am avea
12 (x +y +z) + y + 2z 12 19 + 57 = 285; prin urmare, x + y + z 20. Dac a
x+y +z = 20, atunci y +2z = 46. Pentru y < 15, z < 15, avem y +2z < 45. Pentru
16 y 20, avem c a z 4, deci y +2z 28. Pentru 16 z 20, avem y 4, deci
y + 2z 44. Rezult a c a valoarea minim a a sumei x +y +z este 21, care se obtine,
de exemplu, pentru x = 2, y = 4, z = 15.
VI.43. Fie {a
1
, a
2
, . . . , a
n
}Z

pentru care k{1, 2, . . . , n}, i, j {1, 2, . . . , n},


i 6= j, astfel nct a
k
= a
i
+a
j
. S a se arate c a n 6.
Petru Asaftei, Iasi
Solutie. Putem presupune c a a
1
< a
2
< < a
n
. Considernd num arul a
n
,
exist a i, j {1, 2, . . . , n}, i 6= j, astfel ca a
n
= a
i
+a
j
. Dac a a
i
< 0, a
j
< 0, atunci
a
n
< a
i
, imposibil. La fel, a
i
< 0, a
j
> 0 implic a a
n
< a
j
, imposibil. Deci a
i
si a
j
sunt pozitive si atunci multimea contine cel putin trei numere pozitive. Considernd
num arul a
1
deducem c a multimea contine cel putin trei numere negative, prin urmare
n 6.
Un exemplu de multime cu 6 elemente este {3, 2, 1, 1, 2, 3} iar una cu n 6
elemente este {3, 2, 1, 1, 2, 3, . . . , n 3}.
VI.44. Fie ABCD un paralelogram si MAB, NAD triunghiuri echilaterale con-
struite n exteriorul acestuia. Demonstra ti c a [MN] [BD] dac a si numai dac a
ND k MB.
Ciprian Baghiu, Iasi
M
B
A
N
C D
Solutie. Presupunem [MN] [BD]. Din con-
gruenta triunghiurilor BAD si MAN deducem c a
m(
\
BAD) = m(
\
MAN) =
360

120

2
= 120

, deci
m(
\
BAN) = 180

, adic a punctele B, A, N sunt coli-


niare. Cum
\
MBN
\
DNB, rezult a MB k ND.
Presupunem MB k ND. Ca urmare, m(
\
ABD)+
+m(
\
ADB) = 180

120

= 60

, deci m(
\
BAD) =
= 120

; totodat a m(
\
MAN) = 120

si atunci tri-
unghiurile MAN si BAD sunt congruente, de unde rezult a c a [BD] [MN].
VI.45. Fie E, F picioarele n al timilor din B si C ale triunghiului ascu ti-
tunghic ABC. Dac a P, N sunt mijloacele laturilor [AB], respectiv [AC], iar {Q} =
= PE FN, s a se arate c a m(
\
PQF) =

180

3 m(
b
A)

. (n leg atur a cu Q1086


din Parabola, nr. 3/2000)
Titu Zvonaru, Bucuresti
133
Q
E
C B
F
P
A
N
Solutie. Fie Q exterior triunghiului ABC. Din
4AEB dreptunghic n E, cu (EP) median a, deducem
c a triunghiul AEP este isoscel cu
[
AEP
[
PAE; deci
m(
\
QPF) = 2m(
b
A). La fel, din triunghiul dreptunghic
AFC cu mediana (FN) deducem
\
NFA
\
FAN. Din
4PQF rezult a c a m(
\
PQF) = 180

3m(
b
A). S a remar-
c am c a punctul Q este exterior dac a m(
b
A) < 60

. Cazul
n care Q este interior triunghiului ABC, ce corespunde situatiei m(
b
A) > 60

, se
trateaz a folosind aceleasi argumente. Dac a m(
b
A) = 60

, dreptele PE si FN sunt
paralele si putem considera c a m(
\
PQF) = 0

.
Clasa a VII-a
VII.41. Rezolva ti n N
2
ecua tia
a
b + 1
+
b
a + 1
= 1.
Alexandru Negrescu, elev, Botosani
Not a. Mai multe solutii ale acestei probleme sunt date n articolul prezent n
acest num ar la p. 109.
VII.42. S a se arate c a

a
2
+ 1

b
2
+ 1

c
2
+ 1

(|a| + |b|) (|b| + |c|) (|c| + |a|),


a, b, c R.
Dorin M arghidanu, Corabia
Solutie. Notnd |a| = x, |b| = y, |c| = z, x, y, z R
+
, inegalit atea este echiva-
lent a cu

x
2
+ 1

y
2
+ 1

z
2
+ 1

(x +y) (y +z) (z +x), x, y, z R


+
.Avem

x
2
+ 1

y
2
+ 1

=x
2
y
2
+x
2
+y
2
+1x
2
+y
2
+2xy=(x +y)
2
, deci
p
(x
2
+ 1) (y
2
+ 1)
x + y. La fel, au loc:
p
(x
2
+ 1) (z
2
+ 1) x + z si
p
(y
2
+ 1) (z
2
+ 1) y + z.
Prin nmultirea ultimelor trei relatii se obtine inegalitatea dorit a.
Egalitate se obtine pentru xy = 1, yz = 1, zx = 1, deci pentru x = y = z = 1 sau
|a| = |b| = |c| = 1.
VII.43. Pentru n N, not am cu s (n) num arul de reprezent ari distincte ale
lui n ca sum a de dou a numere naturale (n = a + b si n = b + a constituie aceea si
reprezentare). S a se arate c a:
a) s (m+n) = s (m) +s (n)
1
2
[1 + (1)
mn
]; b)
n
X
k=0
s (k) =
h
n
2
i

h
n + 1
2
i
.
Petru Minut, Iasi
Solutie. Ar at am c a s (n) =
h
n
2
i
+ 1. ntr-adev ar, pentru n par, n = 2k, avem:
n = 0 + n = 1 + (n 1) = . . . = k + k, deci s (n) = k + 1 =
h
n
2
i
+ 1. Pentru
n impar, n = 2k + 1, avem: n = 0 + n = 1 + (n 1) = . . . = k + (k + 1), deci
s (n) = k + 1 =
h
n
2
i
+ 1.
a) Relatia se demonstreaz a analiznd paritatea numerelor m si n.
b) Dac a n = 2k + 1, avem: s (0) + s (1) + . . . + s (n) = 1 + 1 + 2 + 2 + . . . +
+k +k = k (k + 1) =
h
n
2
i

h
n + 1
2
i
. Dac a n = 2k, avem s (0) +s (1) +. . . +s (n) =
= 2 (1 + 2 +. . . + (k 1)) +k = k
2
=
h
n
2
i

h
n + 1
2
i
.
134
VII.44. Fie [AB] diametru al cercului C de centru O; N, M C astfel nct
m(
\
AON) = 36

, iar [OM este bisectoare pentru


\
NOB. Dac a T este simetricul lui
O fa t a de MN, s a se arate c a proiec tia lui T pe AB este mijlocul lui [AO].
Valentina Blendea, Iasi
A O B
N
T
M
Solutie. n triunghiul TNO isoscel, m(
\
TNO) = 180

2m(
\
NOT) = 180


1
2
m(
\
NOB) = 108

. n triunghiul
isoscel OAN, m(
\
ANO) =
1
2
(180

36

) = 72

, deci
m(
\
TNO) + m(
\
ANO) = 180

, prin urmare T, N, A sunt


coliniare. Cum m(
[
TAO)=m(
[
TOA)=72

, triunghiul TAO
este isoscel si concluzia este imediat a.
VII.45. Fie 4ABC echilateral, iar P (BC). Not am cu D, E simetricele lui P
fa t a de AC, respectiv AB. S a se arate c a dreptele AP, BD si CE sunt concurente.
Constantin Cocea si Julieta Grigoras, Iasi
B P C
E
N
M
D
A
Solutie. Fie {M} = BD AC, {N} = CE AB;
AB = a, BP = x, PC = a x. Din asem anarea tri-
unghiurilor DMC si BMA g asim
CM
MA
=
CD
AB
=
a x
a
,
iar din asem anarea triunghiurilor BEN si ACN g asim
BE
AC
=
BN
AN
=
x
a
.
n triunghiul ABC avem
BP
PC

CM
MA

AN
NB
= 1; conform reciprocei teoremei lui
Ceva, dreptele AP, BM, CN sunt concurente.
Clasa a VIII-a
VIII.41. Fie f
1
, f
2
, f
3
func tii liniare ale c aror grafice sunt drepte concurente
dou a cte dou a. Cele trei drepte sunt concurente dac a si numai dac a exist a unic
R si exist a u 6= v R astfel ca
f
1
(u)
f
1
(v)
=
f
2
(u)
f
2
(v)
=
f
3
(u)
f
3
(v)
, cu f
i
(v) 6= , i {1, 2, 3} .
Claudiu- Stefan Popa, Iasi
Solutie. Fie f
i
: R R, f
i
(x) = a
i
x + b
i
, i = 1, 3. Graficele functiilor sunt
concurente dou a cte dou a dac a si numai dac a a
1
, a
2
, a
3
sunt distincte.
Prin proportii derivate relatia din enunt este echivalent a cu
b
1

a
1
=
b
2

a
2
=
=
b
3

a
3
, cu conventia c a, dac a unul dintre numerele a
i
este zero, atunci si b
i

este zero.
Graficele functiilor f
i
sunt concurente dac a si numai dac a exist a (, ), -unic,
astfel nct f
i
() = , i = 1, 3, adic a a
1
+b
1
= a
2
+b
2
= a
3
+b
3
= sau nc a
=
b
1

a
1
=
b
2

a
2
=
b
3

a
3
, ceea ce ncheie rezolvarea problemei.
VIII.42. Fie x, y, z (0, ). S a se arate c a
135

yz +

zx

yz +

zx +
p
x
2
+y
2
+z
2
yz zx
+

xy +

xz

xy +

xz +
p
x
2
+y
2
+z
2
xy xz
+
+

xy +

yz

xy +

yz +
p
x
2
+y
2
+z
2
xy yz
2.
Lucian Tutescu, Craiova
Solutie. Avem x
2
+y
2
+z
2
xy yz zx 0 deci x
2
+y
2
+z
2
yz zx xy
si atunci

yz +

zx

yz +

zx +
p
x
2
+y
2
+z
2
yz zx


yz +

zx

yz +

zx +

xy
. Sumnd
aceasta cu nc a dou a inegalit ati asem an atoare, obtinem concluzia problemei.
VIII.43. Dac a un triunghi dreptunghic are laturile numere naturale, iar suma
catetelor este p atrat perfect, atunci suma cuburilor catetelor este sum a de dou a p a-
trate perfecte.
Andrei Nedelcu, Iasi
Solutie. Fie (a, b, c) un triplet de numere naturale nenule care satisfac relatia
a
2
+ b
2
= c
2
. Urmeaz a c a toate trei sunt pare sau numai unul este par; deci
c b +a
2
,
c +b a
2
N

.
Avem a
2
+b
2
ab =

1
2
(c b +a)

2
+

1
2
(c +b a)

2
. Deoarece a +b = p
2
si
a
3
+b
3
=(a +b)

a
2
+b
2
ab

, rezult a c a a
3
+b
3
=
h
p
2
(c b +a)
i
2
+
h
p
2
(c +b a)
i
2
.
VIII.44. Pe laturile [AB], [CD], [BC], [AD], [AC] si [BD] ale tetraedru-
lui ABCD se iau respectiv punctele M, N, P, Q, R, S astfel ca
BP
BC
=
AQ
AD
,
AM
AB
=
DN
DC
,
AR
AC
=
DS
BD
. Not am cu V
1
, V
2
, V
3
, V
4
, V respectiv volumele tetraedrelor
AMRQ, BPMS, CPNR, DNQS si ABCD. S a se arate c a 2
12
V
1
V
2
V
3
V
4
V
4
.
Viorel Cornea si Dan Stefan Marinescu, Hunedoara
Solutie. Notnd cele trei rapoarte cu , , , avem:
V
1
V
= ,
V
2
V
= (1 )
(1 ),
V
3
V
= (1 ) (1 ) (1 ),
V
4
V
= (1 ) . Atunci
V
1
V
2
V
3
V
4
V
4
=
2
(1 )
2

2
(1 )
2

2
(1 )
2

1
4

1
4

1
4

2
=
1
2
12
,
unde am folosit inegalitatea x(1 x)
1
4
, x [0, 1].
VIII.45. Fie A
1
, A
2
, . . . , A
k
puncte pe un cerc C. S a se determine o condi tie
necesar a si suficient a pentru a putea nscrie n C un poligon regulat ce admite punctele
date ca vrfuri (nu neap arat consecutive).
Irina Mustat a, elev a, Iasi
Solutie. Presupunem c a P este un poligon regulat cu n laturi ce are punctele
A
1
, A
2
, . . . , A
k
ca vrfuri. Dac a ntre A
i
si A
i+1
sunt m
i
laturi ale lui P, cum
unghiul la centru dintre dou a vrfuri consecutive ale lui P este
2
n
, deducem c a
136
m(
\
A
i
OA
i+1
) = m
i

2
n
, deci
m(
\
A
i
OA
i+1
)
2
Q, i = 1, k (cu conventia A
k+1
= A
1
).
S a ar at am c a aceast a conditie este si sucient a. Fie
m(
\
A
i
OA
i+1
)
2
=
m
i
n
i
, (m
i
, n
i
)=1,
i = 1, k. Alegnd n multiplu comun al numerelor n
1
, n
2
, . . . , n
k
rezult a c a
A
1
B
2
. . . B
n
este poligon regulat care contine vrfurile A
1
, A
2
, . . . , A
k
(deoarece
n aceast a situatie m(
\
A
1
OA
2
) =
m
1
n
1
2 =
nm
1
n
1

2
n
si
nm
1
n
1
N, deci A
2
P etc.).
Clasa a IX-a
IX.41. Pentru n N, n 10, not am cu u
2
(n) num arul format din ultimele
dou a cifre ale lui n. S a se arate c a:
a) u
2

a
20k+p

= u
2
(a
p
), p {4, 5, . . . , 23}, k N, a {2, 3, 8};
b) u
2

a
10k+p

= u
2
(a
p
), p {2, 3, . . . , 11}, k N, a {4, 9};
c) u
2
(5
n
) = 25, n N;
d) u
2

6
5k+p

= u
2
(6
p
), p {2, 3, . . . , 6}, k N;
e) u
2

7
4k+p

= u
2
(7
p
), p {2, 3, 4, 5}, k N.
Ovidiu Pop, Satu Mare
Solutie. n rezolvarea problemei vom folosi faptul c a u
2
(a) = u
2
(b) dac a si
numai dac a a b se divide cu 100.
a) 2
20k+p
=

2
10

2k
2
p
= 1024
2k
2
p
= (1025 1)
2k
2
p
= (M25 + 1) 2
p
=
= M100 + 2
p
;
3
20k+p
= 9
10k
3
p
=

81
5

k
3
p
= 01
k
3
p
= (M100 + 1) 3
p
= M100 + 3
p
;
8
20k+p
= 2
60k+3p
= 1024
6k
2
3p
= (M25 1)
6k
8
p
= (M25 + 1)8
p
= M100+8
p
.
Egalit atile b) e) se demonstreaz a analog.
IX.42. Fie a, b, c R

astfel nct a +b +c =
1
a
+
1
b
+
1
c
. S a se afle numerele
a, b, c, dac a |abc| = 1. (enun t scurtat)
Marius Pachitariu, elev, Iasi
Solutie. Ecuatia |abc| = 1 este echivalent a cu a
2
b
2
c
2
= 1. Observ am c a
(1 ab) (1 bc) (1 ca) = 1 ab bc ca +abc (a +b +c) a
2
b
2
c
2
= 1 a
2
b
2
c
2
(s-a tinut seama de relatia din enunt). Ca urmare, suntem condusi la ecuatia
(1 ab) (1 bc) (1 ca) = 0. Dac a ab = 1, g asim c = 1 si, deci, tripletele
(x, 1/x, 1), x R

, sunt solutii ale problemei. Celelalte solutii se obtin din acestea


prin permut ari circulare.
IX.43. Fie func tia f : R R, iar a (1, ). Stiind c a
f

x
2
+ax a

f
2

1
x

f

1
x

+ 1, x (, 0) ,
s a se arate c a f nu este injectiv a.
Titu Zvonaru, Bucuresti
Solutie. Ecuatia x
2
+ ax a =
1
x
, x (, 0) este echivalent a cu (x 1)

x
2
+ax +x + 1

= 0 sau cu x
2
+ x(a + 1) + 1 = 0, x (, 0). Cum > 0,
S < 0, P > 0 ea are dou a r ad acini reale, distincte, negative x
1
, x
2
.
Pentru x = x
1
, relatia din enunt devine f

1
x
1

f
2

1
x
1

f

1
x
1

+ 1, deci
137

f

1
x1

2
0, de unde deducem f

1
x1

= 1. La fel f

1
x2

= 1. Prin urmare
f

1
x
1

= f

1
x
2

, x
1
6= x
2
; deci f nu este injectiv a.
IX.44. Dac a 4ABC este ascu titunghic, s a se g aseasc a maximul expresiei
E = sinA

cos A+ sinB

cos B + sinC

cos C.
Cezar Lupu, elev, si Tudorel Lupu, Constanta
Solutie. Putem presupune A B C, deci sinA sinB sinC si

cos A

cos B

cos C. Folosind inegalitatea lui Cebsev, vom avea


E
sinA+ sinB + sinC
3

cos A+

cos B +

cos C

.
Dar sinA+sinB+sinC
3

3
2
,

cos A+

cos B+

cos C
p
3(cos A+cos B+cos C)

r
3
3
2
=
3

2
. Deci E
3

6
4
. Cum pentru A = B = C = 60

obtinem E =
3

6
4
,
deducem c a maximul lui E este
3

6
4
.
IX.45. Demonstra ti c a 4ABC n care are loc egalitatea
X
h
a
h
b
m
c
m
a
m
b
m
c
+h
a
h
b
m
c
+m
a
m
b
i
c
= 1,
suma fiind ob tinut a prin permut ari circulare, iar nota tiile fiind cele uzuale, este
echilateral.
Iuliana Georgescu si Paul Georgescu, Iasi
Solutie. Avem h
a
i
a
m
a
si analoagele; deci suma din enunt este mai mic a
sau egal a cu S
0
=
X
h
a
h
b
m
c
m
a
m
b
m
c
+h
a
h
b
m
c
+m
a
m
b
h
c
. Notnd x =
h
a
m
a
, y =
h
b
m
b
,
z =
h
c
m
c
, avem 0 x, y, z 1 si S
0
=
X
xy
1 +xy +z
.
Cum 1 +xy x +y pentru x, y [0, 1] si analoagele, avem S
0

X
xy
x +y +z

X
x
x +y +z
= 1; egalitate are loc dac a si numai dac a x = y = z = 1, deci
h
a
= m
a
, h
b
= m
b
, h
c
= m
c
, adic a dac a si numai dac a triunghiul este echilateral.
Clasa a X-a
X.41. Prove the inequality
F
2
2n
F
n1
F
n

2n
n

, where the Fibonacci numbers F


n
are defined by F
0
= F
1
= 1, F
n+1
= F
n
+F
n1
, n 1.
Zdravko Starc, Vrac, Serbia and Montenegro
Solutie. Conform inegalit atii Cauchy-Schwarz, avem
"

n
0

2
+

n
1

2
+ +

n
n

2
#

F
2
0
+F
2
1
+ +F
2
n

n
0

F
0
+

n
1

F
1
+ +

n
n

F
n

2
.
Deoarece
n
P
k=0

n
k

2
=

2n
n

,
n
P
k=0
F
2
k
= F
n
F
n+1
,
n
P
k=0

n
k

F
k
= F
2n
(v. RecMat -
2/2002, p. 70), obtinem

2n
n

F
n
F
n+1
F
2
2n
. Egalitate apare pentru n = 1.
138
X.42. S a se rezolve ecua tia 2
[x]
+ 6
[x]
+ 7
[x]
= 3
[x]
+ 4
[x]
+ 8
[x]
.
Daniel Jinga, Pitesti
Solutie. Not am [x] = y, y Z. Avem de rezolvat n Z ecuatia 2
y
+ 6
y
+ 7
y
=
= 3
y
+4
y
+8
y
. Dac a y < 0 avem 7
y
> 8
y
si 2
y
+6
y
> 3
y
+4
y
(deoarece aceasta este
echivalent a cu (2
y
1) (3
y
2
y
) > 0, y < 0). Prin urmare nu avem solutii cu y < 0.
Se verific a usor faptul c a y = 0, y = 1, y = 2 sunt solutii.
Pentru n 3, n N, demonstr am c a 3
n
+4
n
+8
n
> 2
n
+6
n
+7
n
. Este suficient
s a ar at am prin inductie c a 8
n
+4
n
> 6
n
+7
n
, n 3, care este un exercitiu de rutin a.
Prin urmare y {0, 1, 2} si atunci x [0, 1) [1, 2) [2, 3). Multimea solutiilor
ecuatiei este intervalul [0, 3).
X.43. Fie f o func tie real a nenul a cu proprietatea c a
f (x +y xy) = f (x +y) f (x) f (y) , x, y R.
S a se calculeze f

2003
2002

.
Adrian Zanoschi, Iasi
Solutie. Rezolv am problema n mai multe etape.
1. Pentru x = y = 0, din relatia dat a g asim f (0) = 0.
2. Pentru y = x = 1, deducem f (1) + f (1) f (1) = 0, adic a f (1) = 0, sau
f (1) = 1. Dac a f (1) = 0, pentru y = 1 relatia dat a implic a f (x + 1) = 0,
x R, adic a f = 0, contrar ipotezei.
3. Ne ocup am de cazul f (1) = 1 si f (1) = a 6= 0. Pentru y = 1, g asim
f (x + 1) = af (x) +a, x R. (1)
Pentru y = 1 si x x + 1, obtinem f (2x + 1) = f (x + 1) + f (x) sau nc a
af (2x) +a = af (x) +a +f (x), deci
f (2x) =
a + 1
a
f (x) , x R. (2)
Din (1) si (2) pentru x = 1, deducem f (2) = a
2
+ a = a + 1, de unde a = 1.
Dac a a = 1, relatia (2) implic a f (2x) = 0, x R, fals. Prin urmare a = f (1) = 1.
4. Din (1), g asim f (x + 1) = f (x) +1, x R. Prin inductie, g asim f (x +k) =
= f (x) +k, x R si k Z. Pentru x = 0 deducem c a f (k) = k, k Z.
5. n relatia dat a nlocuim x cu
p
q
si y cu q (p, q Z, q 6= 0) si g asim f

p
q

=
p
q
,
adic a f (x) = x, x Q. Prin urmare, f

2003
2002

=
2003
2002
.
X.44. Urnele U
1
, U
2
, . . . , U
n
con tin fiecare cte a bile albe si b bile negre. Din
fiecare urn a se extrage cte o bil a care se depune ntr-o alt a urn a U. Din urna U
se scoate o bil a si se constat a c a este alb a. Care este compozi tia cea mai probabil a a
urnei U?
Petru Minut, Iasi
Solutie. Not am cu E
k
evenimentul constnd n faptul c a n U ar fi depuse k bile
albe (si n k bile negre). Avem P (E
k
) = C
k
n

a
a +b

b
a +b

nk
, k = 0, n. Fie
E evenimentul ca bila extrasa din U sa fie alb a. Conform formulei lui Bayes
139
P
E
(E
k
) =
P (E
k
) P
E
k
(E)
P
n
i=1
P (E
i
) P
E
i
(E)
=
kC
k
n
a
k
b
nk
P
n
i=1
iC
i
n
a
i
b
ni
=
kC
k
n
a
k
b
nk
na (a +b)
n1
.
Aceast a probabilitate este maxim a cnd f (k) = kC
k
n
a
k
b
nk
, k = 0, n este maxim a.
Observ am c a
f (k + 1)
f (k)
> 1 k <
na
a +b
si
f (k + 1)
f (k)
< 1 k >
na
a +b
. Dac a k
0
este punctul de maxim pentru f (k), avem f (k
0
+ 1) f (k
0
) si f (k
0
) f (k
0
1),
din care deducem k
0

na
a +b
,
na
a +b
+ 1

. Prin urmare, k
0
=
na
a +b
, dac a
na
a +b
N
si k
0
=

na
a +b

+ 1, dac a
na
a +b
/ N.
X.45. Se consider a triunghiul ascu titunghic ABC. S a se arate c a exist a un
triunghi A
0
B
0
C
0
astfel nct A
0
(BC), B
0
(AC), C
0
(AB), iar m(
\
AC
0
B
0
) =
= m(
\
BA
0
C
0
) = m(
\
CB
0
A
0
) = (0, 90]. Dac a n plus 4ABC este echilateral, s a se
calculeze lungimile laturilor 4A
0
B
0
C
0
n func tie de a = BC si . (n leg atur a cu o
problem a propus a la O. N. M., 2002)
Dan Popescu, Suceava
Solutie. Fie D proiectia punctului C pe AB, deci D (AB). Fie C
00
(DA) si
E (AC) nct m(
\
EC
00
A) = si A
00
(BC) nct m(
\
C
00
A
00
B) = . De asemenea,
fie A
1
si C
1
, A (BA
1
), C (BC
1
) si F (A
1
C
1
) cu A
1
C
1
k AC si m(
\
A
00
FC
1
) = .
n aceste conditii, (C
00
E (A
00
F = {B
00
}, (BB
00
(AC) = {B
0
}, iar omotetia de
centru B si raport , raport definit de

BB
0
=

BB
00
, transform a A
00
n A
0
(BC)
si C
00
n C
0
(AB). Prin urmare A
0
B
0
C
0
satisface enuntul.
n cazul n care triunghiul ABC este echilateral se deduce relativ usor c a 4A
0
B
0
C
0
este echilateral si (AB
0
) (CA
0
) (BC
0
). Cu teorema sinusurilor n 4AB
0
C
0
deducem B
0
C
0
=
a
cos +

3 sin
.
Clasa a XI-a
XI.41. Fie A
1
, A
2
, . . . , A
k
M
n
(Z) astfel nct
P
S
k
A
(1)
A
(2)
. . . A
(k)
= I
n
,
unde S
k
este mul timea permut arilor de ordin k. S a se arate c a n
.
.
. k!.
Vladimir Martinusi, Iasi
Solutie. Deoarece tr (AB) = tr (BA), A, B M
n
(C), inductiv se arat a c a
tr

A
(1)
A
(2)
A
(k)

= tr (A
1
A
2
A
k
), S
k
. Tinnd cont c a tr (A+B) =
= tr A+ tr B, A, B M
n
(C) deducem c a
tr

P
S
k
A
(1)
A
(k)
!
= tr (I
n
) tr

P
S
k
A
(1)
A
(k)
!
= n

P
S
k
tr

A
(1)
A
(k)

= n k! tr (A
1
A
2
A
k
) = n
si cum tr (A
1
A
2
A
k
) Z, concluzia problemei este imediat a.
XI.42. Prin punctele M
1
si M
2
ale unei elipse se duc normalele la elips a, care
intersecteaz a una din axele de simetrie ale acesteia n M
0
1
, respectiv M
0
2
. S a se
arate c a mediatoarea segmentului [M
1
M
2
] trece prin mijlocul lui [M
0
1
M
0
2
]. R amne
proprietatea adev arat a pentru hiperbol a sau pentru parabol a?
Gheorghe Costovici, Iasi
140
Solutie. Fie curba de ecuatie
x
2
a
2
+
y
2
b
2
= 1, = 1 si M
i
(x
i
, y
i
) pe curb a,
i = 1, 2. Normala n M
i
la curb a are ecuatia y y
i
=
a
2
b
2
y
i
x
i
(x x
i
), i = 1, 2, si
intersecteaz a una din axele simetrie, de exemplu Ox, n M
0
i

a
2
b
2
a
2
x
i
, 0

, i = 1, 2.
Mijlocul M
0
al segmentului [M
0
1
, M
0
2
] are coordonatele

a
2
b
2
2a
2
(x
1
+x
2
) , 0

. Me-
diatoarea segmentului M
1
M
2
are ecuatia y
y
1
+y
2
2
=
x
2
x
1
y
2
y
1

x
x
1
+x
2
2

.
Se veric a usor c a M
0
este situat pe mediatoarea segmentului [M
1
M
2
].
Prin calcul se verific a c a proprietatea r amne valabil a si pentru parabol a.
XI.43. Consider am sirul de func tii f
n
: (0, ) R, f
n
(x) = nx+lnx(n 1)
si fie x
n
solu tia unic a a ecua tiei f
n
(x) = 0. S a se calculeze limitele sirurilor (x
n
)
n1
si ((x
n
)
n
)
n1
.
Angela Tig aeru, Suceava
Solutie. Evident, functiile f
n
sunt strict cresc atoare si surjective, ceea ce asigur a
existenta si unicitatea solutiei x
n
.
Folosim n continuare inegalit atile
x 1
x
lnx x 1, x (0, ). Avem:
f
n

n
n + 1

=
1
n + 1
+ ln
n
n + 1

1
n + 1
+
n
n + 1
1 = 0 si
f
n

n + 1
n + 2

= ln
n + 1
n + 2
+
2
n + 2

n + 1
n + 2
1

n + 2
n + 1
+
2
n + 2
> 0, n N

.
Prin urmare, f
n

n
n + 1

f
n
(x
n
) f
n

n + 1
n + 2

si atunci
n
n + 1
x
n

n + 1
n + 2
,
n 1. G asim lim
n
x
n
= 1 si lim
n
x
n
n
=
1
e
.
XI.44. S a se determine func tiile continue f : (0, ) (0, ) pentru care
f (x) = f

2x
2
2x + 1

, x > 0.
Marian Urs arescu, Roman
Solutie. Fie x
0
(0, 1] fixat si sirul (x
n
)
n0
definit prin x
n+1
=
p
2x
2
n
2x
n
+ 1,
n N. Se verific a faptul c a x
n
(0, 1], n N, (x
n
)
n0
este cresc ator si lim
n
x
n
= 1.
Pentru x = x
n
, relatia din enunt devine f (x
n
) = f

p
2x
2
n
2x
n
+ 1

= f (x
n+1
),
n N, deci f (x
0
) = f (x
n
). Ca urmare, f (x
0
) = lim
n
f (x
n
). Rezult a c a f (x) =
= f (1), x (0, 1].
Fie x
0
> 1 si sirul definit prin x
n+1
=
1 +
p
2x
2
n
1
2
, n N. Se arat a c a
x
n
> 1, n N. (x
n
)
n0
este descresc ator si lim
n
x
n
= 1. Cum relatia de re-
curent a se poate scrie si n forma x
n
=
q
2x
2
n+1
2x
n+1
+ 1, obtinem f (x
n+1
) =
= f
q
2x
2
n+1
2x
n+1
+ 1

= f (x
n
), apoi f (x
0
) = f (x
n
) si f (x) = f (1), x > 1.
n concluzie, f (x) = f (1), x (0, ), deci f este functie constant a.
141
XI.45. Fie k N, k 2 si numerele reale pozitive a
1
, a
2
, . . . , a
k
, b
1
, b
2
, . . . , b
k
cu a
1
< a
2
< < a
k
. Definim x
n
=
n
p
b
1
a
n
1
+b
2
a
n
2
+. . . +b
k
a
n
k
.
a) S a se demonstreze c a lim
n
x
n
= a
k
;
b) S a se arate c a lim
n
n(x
n
a
k
) = a
k
lnb
k
;
c) Dac a b
k
= 1, are loc lim
n
n

a
k
a
k1

n
(x
n
a
k
) = a
k
b
k1
.
Marian Tetiva, Brlad
Solutie. a) Avema
k
n

b
k

n
p
a
n
1
b
1
+ +a
n
k
b
k
a
k
n

b
1
+b
2
+ +b
k
, n N.
Deducem c a lim
n
n
p
b
1
a
n
1
+b
2
a
n
2
+ +b
k
a
n
k
= a
k
.
b)
lim
n
n(x
n
a
k
) = a
k
lim
n
n

x
n
a
k
1

= a
k
lim
n
n
e
ln
x
n
a
k
1
ln
x
n
a
k
ln
x
n
a
k
= a
k
lim
n
ln

x
n
a
k

n
=
= a
k
lim
n
ln

b
1

a
1
a
k

n
+b
2

a
2
a
k

n
+ +b
k1

a
k1
a
k

n
+b
k

= a
k
lnb
k
.
c)
lim
n
n

a
k
a
k1

n
(x
n
a
k
) = lim
n
a
k
n

a
k
a
k1

n
e
ln
x
n
a
k
1
ln
x
n
a
k
ln
x
n
a
k
=
= a
k
lim
n

a
k
a
k1

n
ln

b
1

a
1
a
k

n
+b
2

a
2
a
k

n
+ +b
k1

a
k1
a
k

n
+ 1

=
= a
k
lim
n

a
k
a
k1

b
1

a
1
a
k

n
+b
2

a
2
a
k

n
+ +b
k1

a
k1
a
k

=
= a
k
lim
n

b
1

a
1
a
k1

n
+b
2

a
2
a
k1

n
+ +b
k1

= a
k
b
k1
.
Clasa a XII-a
XII.41. S a se calculeze
Z
(1 +x)

1 +x
2

. . .

1 +x
2
n1

x
2
n
dx, unde x [1, ),
n N

.
Oana Marangoci, student a, Iasi
Solutie. Pentru x (1, ), avem (1 +x)

1 +x
2

1 +x
2
n1

=
=
1 x
2
1 x

1 x
2
2
1 x
2
. . .
1 x
2
n
1 x
2
n1
=
1 x
2
n
1 x
= 1 +x +x
2
+ +x
2
n
1
si atunci
(1 +x)

1 +x
2

1 +x
2
n1

x
2
n
=
1
x
2
n
+
1
x
2
n
1
+ +
1
x
,
relatie care se verific a si pentru x = 1. Primitivele functiei sunt
1
(2
n
1) x
2
n
1
+
1
(2
n
2) x
2
n
2
+ +
1
x
+ lnx +C.
XII.42. Fie f :

,
5
4

R o func tie continu a pentru care


Z 5
4

f (x) sin2xdx =

4
.
142
S a se arate c a exist a c

,
5
4

astfel nct f (c) (1, 2).


Mihai Haivas, Iasi
Solutie. Avem
Z 5
4

sin2x
1
1
2
sin
2
2x
dx =
Z 5
4

(cos 2x)
0
1 + cos
2
2x
dx = arctg (cos 2x)

5
4

=

4
.
Z 5
4

f (x) sin2xdx =
Z 5
4

sin2x
1
1
2
sin
2
2x
dx
Z 5
4

f (x)
2
2 sin
2
x

sin2xdx = 0 .
Aplicnd teorema de medie exist a c

,
5
4

astfel nct

f (c)
2
2 sin
2
2c

sin2c = 0 f (c) =
2
2 sin
2
2c
.
Dar
2
2 sin
2
2c
(1, 2) pentru c

,
5
4

, adic a f (c) (1, 2).


XII.43. S a se arate c a
1
lna
3

Z
1
0
a
x
2
dx
arctg

lna

lna
, a > 1.
Petru R aducanu, Iasi
Solutie. Se arat a c a a
x
xlna + 1, x R si a > 1. Obtinem astfel
x
2
lna + 1 a
x
2
=
1
a
x
2

1
x
2
lna + 1
, x R, a > 1.
Integrnd ntre 0 si 1, obtinem inegalit atile cerute.
XII.44. S a se afle num arul r ad acinilor reale ale polinomului P Z[X] de grad
minim, care admite r ad acina
2
+, unde verific a ecua tia x
3
x + 1 = 0.
Laurentiu Modan, Bucuresti
Solutie. Polinomul n y care se cere, apare prin eliminarea lui x ntre ecuatiile
x
3
x + 1 = 0 si y = x
2
+ x. Deoarece x
2
= y x, g asim x(y x) x + 1 = 0 si
nlocuind din nou x
2
g asimx =
y 1
y
(evident y 6= 0). Deducemy =
(y 1)
2
y
2
+
y 1
y
si, n final, P (y) = y
3
2y
2
+3y 1. Folosind sirul lui Rolle, polinomul P (y) admite
o singur a r ad acin a real a y (0, 1).
XII.45. Fie S
5
. S a se arate c a
2
are puncte fixe dac a si numai dac a
3
are puncte fixe.
Paul Georgescu si Gabriel Popa, Iasi
Solutie. Folosind descompunerea unei permut ari n produs de cicli disjuncti,
se poate observa c a, dac a
k
, 1 k 5, are puncte xe, atunci contine n
descompunerea sa cicli de lungime l, cu l | k.
Presupunem c a
2
are puncte xe. Atunci contine n descompunerea sa cicli de
lungime 1 (puncte xe) sau 2. Dac a are puncte xe,
3
are de asemenea puncte
xe. Dac a contine n descompunerea sa cicli de lungime 2, f ar a a avea puncte xe,
atunci contine n descompunerea sa si un ciclu de lungime 3. Elementele acestui
ciclu vor toate puncte xe pentru
3
. Implicatia reciproc a se demonstreaz a analog.
143
Solutiile problemelor pentru preg atirea concursurilor
din nr. 2 / 2003
A. Nivel gimnazial
G46. Determina ti ultimele cinci cifre ale num arului
A = 7
2000
+ 7
2001
+ 7
2002
+ 7
2003
.
Viorel Cornea si Dan Stefan Marinescu, Hunedoara
Solutia I. Scriem A = 7
2000
400. Se constant a c a 7
20
= M1000+1. Ca urmare,
7
2000
= M1000 + 1 si A = (M1000 + 1) 400 = M100000 + 400, deci ultimele cinci
cifre ale lui A sunt 00400.
Solutia II (Irina Mustat a, elev a, Iasi). Conform teoremei lui Euler, a
(n)

1(modn) pentru (a, n) = 1. Considernd a = 7, n = 1000, avem (1000) = 400, deci


7
400
1(mod1000), adic a 7
2000
=

7
400

5
1(mod1000), de unde A = 7
2000
400
400(mod100000), deci num arul A se termin a cu 00400.
G47. Determina ti valorile parametrilor a, b Z pentru care solu tiile sistemului
x = a
y
y + 1
; y = b
x
x + 1
sunt n Z Z.
Temistocle Brsan, Iasi
Solutie. Fie M multimea perechilor (a, b) Z Z pentru care este ndeplinit a
cerinta problemei. Observ am c a (0, 0) ZZ este solutie a sistemului si c a aceasta
este unica solutie dac a a = 0 sau b = 0. Ca urmare, (a, 0) M, a Z si (0, b) M,
b Z.
Pe (R \ {1}) (R\ {1}) sistemul dat, dac a facem abstractie de solutia (0, 0),
este echivalent cu
(b + 1) x = ab 1, (a + 1) y = ab 1. (1)
I. a 6= 1, b 6= 1. Obtinem x =
ab 1
b + 1
, y =
ab 1
a + 1
. Atunci (x, y) Z Z
b + 1 | ab 1 si a + 1 | ab 1. Cum ab 1 = (a + 1) (b + 1) (a + 1) (b + 1),
deducem c a (x, y) Z Z b + 1 | a + 1 si a + 1 | b + 1 b + 1 = a + 1 sau
b + 1 = (a + 1). Prin urmare (a, a) M, a Z, a 6= 1 si (a, a 2) M,
Z, a 6= 1.
II. a = b = 1. Cum rezult a ab 1 = 0, sistemul (1) are o infinitate de solutii si
nu toate sunt n Z Z. Deci (1, 1) / M.
III. a = 1, b 6= 1 (analog a 6= 1, b = 1). Avem ab 1 6= 0, deci (1) nu
are solutii, iar sistemul dat are (0, 0) ca unic a solutie. Asadar, (a, 1) M, a Z,
a 6= 1 si (1, b) Z, b Z, b 6= 1.
n concluzie:
M = {(a, 0) , a Z} {(0, b) , b Z} {(a, 1) , a Z} {(1, b) , b Z}
{(a, a) , a Z} {(a, a 2) , a Z} \ {(1, 1)} .
G48. Fie A (0, ) o mul time care con tine
2002
2003
si avnd proprietatea c a,
dac a
a
b
A (a, b N

), atunci
a + 1
b
A si
a
2b
A. S a se arate c a A Q

+
.
Gheorghe Iurea, Iasi
144
Solutie. Consider am transform arile
a
b
A
a + 1
b
A (1)
si
a
b
A
a
2b
A. (2)
Avem:
2002
2003
A
(2)

1001
2003
A
(1)

1002
2003
A
(2)

501
2003
A
(1)

502
2003
A
(2)

251
2003
A
(1)

(2)

1
2003
A.
1
2003
A
(1)

2
2003
A
(1)

(1)

2003
2003
A, deci
1
1
A.
Fie
a
b
Q

+
. Cum
1
1
A
b
b
A
1
b
A
a
b
A, rezult a c a
a
b
A, adic a
Q

+
A.
G49. Fie x
1
, x
2
, . . . , x
n+1
R

+
astfel nct x
1
+ x
2
+ . . . + x
n+1
(n + 2) m,
iar x
2
1
+ x
2
2
+ . . . + x
2
n+1

n + 4
4
M
2
, unde m = minx
i
, M = max x
i
. S a se arate
c a exact n dintre numerele date sunt egale.
Eugen Jecan, Dej
Solutie. Dac a toate numerele ar fi egale, relatia a doua din enunt ar fi
(n + 1) M
2

n + 4
4
M
2
, imposibil. Exist a cel putin un num ar egal cu M, fie acesta
x
n+1
. Deducem c a (n + 2) m x
1
+ + x
n
+ M deci 2m M. De asemenea,
n + 4
4
M
2
x
2
1
+ + x
2
n
+ M
2
nm
2
+ M
2
sau M 2m. Rezult a M = 2m si
prima relatie devine x
1
+x
2
+ +x
n
+2m (n + 2) m, deci x
1
+x
2
+ +x
n
nm
sau nc a (x
1
m)+ +(x
n
m) 0. Cum x
1
m 0, x
2
m 0, . . . , x
n
m 0,
deducem c a x
1
= x
2
= = x
n
= m, valori care veric a si a doua conditie.
n concluzie exact n numere sunt egale cu m si unul egal cu M = 2m.
G50. Fie a N, a 3. S a se arate c a

an + 1

=

an + 2

= . . . =

an +a 1

, n N a {3, 4} .
Ovidiu Pop, Satu Mare
Solutie. Pentru n = 0 relatia din enunt devine

1

=

2

= =

a 1

,
deci

a 1

= 1, echivalent cu 2 a < 5, si cum a N, a 3, avem a {3, 4}.


Reciproc, ar at am c a, dac a a {3, 4}, au loc egalit atile din enunt. Fie a = 3;
trebuie ar atat c a

3n + 1

=

3n + 2

, n N.
Pentru un num ar n N exist a k N (k depinde de n) astfel nct k
2
3n +1 <
< (k + 1)
2
. Considernd num arul 3n+2 avem 3n+2 < (k + 1)
2
sau 3n+2 = (k + 1)
2
.
Cum 3n +2 nu este p atrat perfect, deducem c a k
2
3n +1 < 3n +2 < (k + 1)
2
, de
unde rezult a k

3n + 1 <

3n + 2 < k + 1, deci

3n + 1

=

3n + 2

.
Pentru a = 4, trebuie demonstrat c a

4n + 1

=

4n + 2

=

4n + 3

,
n N. Considernd k =

4n + 1

avem k
2
4n + 1 < (k + 1)
2
si cum 4n + 2,
4n+3 nu sunt p atrate perfecte pentru nici un n N, deducem k
2
4n+1 < 4n+2 <
< 4n + 3 < (k + 1)
2
si atunci

4n + 1

=

4n + 2

=

4n + 3

= k.
145
G51. Fie a, b, c

3
10
,

cu a +b +c = 1. S a se arate c a
2
3
a

a +bc +b

b +ca +c

c +ab <
3
4
.
Gabriel Dospinescu, elev, Onesti
Solutia. Avem

a +bc =
p
(a +b) (a +c)
a +b +a +c
2
=
1 +a
2
si atunci
X
a

a +bc
X
a
2
+a
2
=
1
2

1 +
X
a
2

. Cum a = 1 (b +c) < 1


6
10
=
2
5
etc., rezult a c a
1
2

1 +
X
a
2

<
1
2

1 + 3
4
25

=
1
2

37
25
<
3
4
.
Pentru inegalitatea din stnga avem: a + bc = (a +b) (a +c)

a +

bc

2
si
atunci
X
a

a +bc >
X
a

a +

bc

= 12
X
ab+
X
a

bc. R amne s a ar at am
c a
1
3
2
X
ab+
X
a

bc 0 sau, echivalent,
X
(a b)
2
3

X
c

2
. Este
suficient s a ar at am c a
(a b)
2
3
c

2
, echivalent a cu

a +

2
3c
sau cu a +b +2

ab 3 3a 3b. Ultima inegalitate se scrie 4a +4b +2

ab 3 si
este adev arat a n conditia a, b

3
10
,

.
G52. Se consider a o piramid a format a din p atrate 1 1,
avnd n trepte, pe treapta k existnd 2k 1 p atrate (n figur a,
n = 4). Afla ti num arul minim de dreptunghiuri, fiecare alc atuit
numai din c asu te ntregi, n care poate fi mp ar tit a tabla.
Adrian Zahariuc, elev, Bac au
Solutia I (a autorului). Color am c asutele tablei alternativ n alb si negru
(c asuta din vrf este neagr a). Cum modulul diferentei dintre num arul de c asute albe
si num arul de c asute negre dintr-un dreptunghi este cel mult 1 si cum sunt cu n
c asute negre mai multe dect albe, trebuie s a avem cel putin n dreptunghiuri. Acest
num ar poate fi obtinut t aind pe nivele. Asadar, r aspunsul este n.
Solutia II (Irina Mustat a, elev a, Iasi). Unim, ca n figu-
r a, vrfurile din stnga sus ale p atratelor marginale din stnga;
se observ a c a deasupra acestei drepte nu mai exist a vrfuri
ale piramidei. Oricare dou a p atrate marginale din stnga nu
pot apartine aceluiasi dreptunghi, deoarece ar nsemna c a acel
dreptunghi va avea coltul din stnga sus deasupra dreptei considerate; prin urmare,
num arul minim de dreptunghiuri este n, minim atins pentru mp artirea pe trepte.
Solutia III (Marius Pachitariu, elev, Iasi). Prin inductie complet a.
G53. Fie ABCD un p atrat de latur a 70. S a se arate c a exist a o mul time de
p atrate P
k
=

A
i
B
i
C
i
D
i
| A
i
B
i
= i, i = 1, k

care s a aib a suma ariilor egal a cu aria


p atratului dat. Putem acoperi p atratul ABCD cu elementele mul timii P
k
?
Petru Asaftei, Iasi
Solutie. Trebuie s a avem 1
2
+2
2
+ +k
2
= 70
2
, echivalent cu
k(k+1)(2k+1)
6
=
146
= 70
2
, de unde k = 24. Solutia este unic a deoarece k > 24 implic a 1
2
+2
2
+ +k
2
>
> 70
2
iar k < 24, 1
2
+ 2
2
+ +k
2
< 70
2
.
Ar at am c a multimea P
k
nu poate acoperi p atratul ABCD. Pentru aceasta, s a
observ am c a p atratele A
i
B
i
C
i
D
i
, pentru a acoperi P
k
, nu au puncte comune, exceptie
f acnd laturile (altfel aria acoperit a de acestea este mai mic a dect 1
2
+2
2
+ +24
2
=
= 70
2
); de asemenea, p atratele A
i
B
i
C
i
D
i
nu las a "spatii goale" ntre ele (altfel aria
acoperit a de ele plus ariile "spatiilor goale" este mai mare ca 1
2
+2
2
+ +24
2
= 70
2
).
Analiznd pozitia p atratului de latur a 1 n p atratul ABCD, observ am c a r amne
o suprafat a ce nu mai poate fi acoperit a (r amne o suprafat a dreptunghiular a de
laturi 1 si l 1 n care nu "ncape" nici unul din p atratele r amase, care au laturile
mai mari sau egale cu 2).
G54. S a se arate c a nu putem alege nici un punct n interiorul triunghiului
echilateral ABC de latur a l 10, care s a aib a distan tele la vrfuri numere prime
distincte.
Doru Buzac, Iasi
B
N
M
C
A
3
3
5
7
7
Solutie. Presupunem, prin absurd, c a exist a
M Int (ABC) astfel nct MA, MB, MC s a fie nu-
mere prime.
Cum MA, MB, MC < l si MA, MB, MC pot fi
laturile unui triunghi (teorema lui Pompeiu), deducem
c a AM, BM, CM {3, 5, 7}. Fie AM = 7, BM = 3,
CM = 5.
Fie N astfel nct BN = 3 si m(
\
MBN) = 60

. Evi-
dent, triunghiul BMN este echilateral. Din congruenta
triunghiurilor AMB si CNB deducem NC = 7. Atunci
cos
\
NMC =
MN
2
+MC
2
NC
2
2 MN MC
=
1
2
, deci m(
\
NMC) = 120

si m(
\
BMC) = 180

,
fals. Prin urmare, nu exist a M Int (ABC) cu proprietatea cerut a.
G55. Printr-un punct situat n interiorul unui tetraedru se duc planele paralele cu
fe tele tetraedrului. Dac a V
1
, V
2
, V
3
, V
4
sunt volumele tetraedrelor unic determinate
de aceste plane, iar V este volumul tetraedrului dat, s a se arate c a
27V 16 (V
1
+V
2
+V
3
+V
4
) .
Neculai Roman, Mircesti (Iasi)
Solutie. Not am cu x
i
distanta de la punctul considerat la fata tetraedrului
A
1
A
2
A
3
A
4
opus a vrfului A
i
si cu h
i
n altimea corespunz atoare acestei fete. Fie
a
i
=
3
p
V
i
/V , i = 1, 4. Avem a
i
=
h
i
x
i
h
i
= 1
x
i
h
i
, i = 1, 4, deci
4
P
i=1
a
i
=
= 4
4
P
i=1
x
i
h
i
= 4
4
P
i=1
S
i
x
i
S
i
h
i
= 4
1
V
4
P
i=1
V
i
= 4 1 = 3.
Relatia de demonstrat se scrie 27 16

a
3
1
+a
3
2
+a
3
3
+a
3
4

si decurge din faptul


c a
a
3
1
+a
3
2
+a
3
3
+a
3
4
4

a
1
+a
2
+a
3
+a
4
4

3
=
3
3
4
3
=
27
64
.
B. Nivel liceal
L46. Fie ABCD un patrulater inscriptibil. Bisectoarele unghiurilor
b
A si
b
B se
147
intersecteaz a ntr-un punct situat pe latura [CD]. S a se arate c a CD = AD +BC.
Mircea Becheanu, Bucuresti
A
B
C
D
E
P
Solutia I (Irina Mustat a, elev a, Iasi). Fie m(
b
A) = 2,
m(
b
B) = 2, iar P (CD) astfel nct AD = DP; avem c a
m(
\
APD) =
1
2

180

m(
b
D)

= . Deosebim dou a cazuri,


dup a cum E (DP) sau E (PC); ne plas am n prima
situatie. Deoarece
[
EPA
\
EBA, patrulaterul ABPE este
inscriptibil, deci m(
\
CPB) = . Atunci m(
\
CBP) = 180

m(
b
C) = , deci 4CPB este isoscel cu CP = CB, de unde concluzia.
Solutia II. Folosind teorema sinusurilor n triunghiurile ADE, BEC si AEB,
obtinem:
DE =
sin
A
2
2 cos
B
2
sin
A+B
2
AB, CE =
sin
B
2
2 cos
A
2
sin
A+B
2
AB,
AD =
sin

B
A
2

2 cos
B
2
sin
A+B
2
AB, BC =
sin

A
B
2

2 cos
A
2
sin
A+B
2
AB,
din care deducem
CD = DE +CE = AD +BC =
sinA+ sinB
4 cos
A
2
cos
B
2
sin
A+B
2
AB.
Not a. S-a mai primit solutie corect a de la Marius Pachitariu, elev, Iasi.
L47. Dac a un triunghi are p atratele laturilor n progresie aritmetic a, atunci
simetricul centrului de greutate fa t a de latura mijlocie se afl a pe cercul circumscris
triunghiului.
Gabriel Popa si Paul Georgescu, Iasi
Solutie. Fie a, b, c lungimile laturilor triunghiului ABC, c < a < b cu 2a
2
= b
2
+c
2
.
Simetricul cercului circumscris triunghiului ABC fat a de BC este cercul circum-
scris triunghiului BHC si atunci simetricul lui G fat a de BC este pe cercul circum-
scris dac a si numai dac a BHGC este patrulater inscriptibil, echivalent cu faptul c a
m(
\
BGC) = m(
\
BHC) = m(
b
A).
n triunghiul BGC se determin a cos
\
BGC =
a
2
2bc
(se foloseste relatia 2a
2
= b
2
+c
2
si formula medianei). Cum cos A =
a
2
2bc
, deducem c a avem cos
\
BGC = cos A, deci
m(
\
BGC) +m(
b
A) = , ceea ce ncheie solutia.
Not a. Solutii corecte au dat Irina Mustat a si Marius Pachitariu, elevi, Iasi.
L48. Fie R, r, R
1
raza cercului circumscris 4ABC, raza cercului nscris 4ABC,
respectiv raza cercului circumscris 4DEF determinat de picioarele bisectoarelor in-
terioare ale 4ABC. S a se arate c a R/2 R
1
r.
Marian Tetiva, Brlad
Solutie (Titu Zvonaru, Com anesti (Bac au)). Inegalitatea r R
1
este ade-
v arat a oricare ar fi punctele D (BC), E (CA), F (AB) (Liliana Niculescu -
148
O metod a de demonstrare a unor inegalit a ti geometrice, GM - 2-3/1993; Teorema 1,
p. 51). ntr-adev ar, fie Q centrul cercului circumscris 4DEF si d
1
, d
2
, d
3
distantele
de la Q la laturile BC, CA, AB. Cum fiecare dintre dreptele BC, CA si AB este
secant a sau tangent a cercului circumscris 4DEF, avem d
1
R
1
, d
2
R
1
, d
3
R
1
,
deci
pr = A
[ABC]
= A
[QBC]
+A
[QCA]
+A
[QAB]
=
ad
1
2
+
bd
2
2
+
cd
3
2

aR
1
2
+
bR
1
2
+
cR
1
2
= pR
1
,
adic a r R
1
.
Inegalitatea R
1
R/2 este adev arat a, dac a punctele D, E, F apartin segmentelor
determinate de picioarele n altimilor si mijloacele laturilor respective (asa cum se
ntmpl a cu picioarele bisectoarelor). n acest caz aceste puncte sunt n interiorul
cercului lui Euler al 4ABC, a c arui raz a este R/2, si rezult a R
1
R/2.
L49. ntr-un p atrat 10 10 se nscriu numerele 1, 2, 3, . . . , 100 n a sa fel nct
oricare dou a numere consecutive s a se afle n c asu te vecine. Demonstra ti c a exist a
o linie sau o coloan a ce con tine m acar dou a p atrate perfecte.
Adrian Zahariuc, elev, Bac au
Solutie. Observ am c a avem 10 p atrate perfecte dintre care 5 sunt pare. Pre-
supunem c a p atratele perfecte sunt situate pe linii si coloane diferite. Color am tabla
ca pe o tabl a de sah n alb si negru. Numerele pare vor fi situate pe c asute de aceasi
culoare, la fel numerele impare.
Fie (x
1
, y
1
), (x
2
, y
2
), . . . , (x
10
, y
10
) coordonatele c asutelor n care sunt situate
p atratele perfecte. Datorit a presupunerii c a p atratele perfecte sunt pe linii si coloane
diferite, numerele x
i
, i = 1, 10 snt diferite dou a cte dou a si {x
1
, x
2
, . . . , x
10
} =
= {1, 2, . . . , 10}; la fel pentru numerele y
i
, i = 1, 10. Obtinem (x
1
+y
1
)+(x
2
+y
2
) +
+ +(x
10
+y
10
) = 2 (1 + 2 + + 10) = num ar par si rezult a c a un num ar par de
perechi are suma par a. Cum perechile cu suma combinatiilor par a au aceasi culoare,
deducem c a exist a un num ar par de p atrate perfecte pare, absurd.
Not a. Solutie corect a a dat Marius Pachitariu, elev, Iasi.
L50. Fie (a
n
)
n1
o progresie aritmetic a avnd a
1
= 5, r = 2002. Pentru un
element b al progresiei, s a se arate c a b
m
apar tine progresiei dac a si numai dac a
60 | m1.
Mihai Piticari, C-lung Moldovenesc
Solutie. S a demonstr am mai nti urm atoarea
Lem a. Dac a a, m, n sunt numere naturale nenule astfel nct n | a
m
1 si m
este cel mai mic num ar cu aceast a proprietate, atunci n | a
k
1 dac a si numai dac a
m | k.
ntr-adev ar, dac a m | k, atunci k = sm si din n | a
m
1 | a
sm
1 rezult a
n | a
k
1. Presupunem n | a
k
1. Conform teoremei mp artirii cu rest, exist a
c si r numere naturale astfel nct k = mc + r, r < m. Din n | a
k
1, rezult a
n | a
mc+r
1 = a
r
(a
mc
1) + a
r
1 si deducem n | a
r
1. Cum r < m, pe baza
minimalit atii lui m rezult a r = 0, deci m | k.
Revenim la problema dat a. Evident, b
m
apartine progresiei dac a si numai dac a
2002 | b
m
b. Cum b = 5+2002p, rezult a c a 2002 este prim cu b si atunci b
m
apartine
progresiei dac a si numai dac a 2002 | b
m1
1. Din b
m1
1 = (5 + 2002p)
m1

149
5
m1
+5
m1
1, urmeaz a c a 2002 | b
m1
1 dac a si numai dac a 2002 | 5
m1
1.
Avem 2002 = 2 7 11 13 si 6, 10, 4 sunt minime cu propriet atile 7 | 5
6
1,
11 | 5
10
1, 13 | 5
4
1. Deci, pe baza lemei, 2002 | 5
m1
1 dac a si numai dac a
6 | m1, 10 | m1, 4 | m1, adic a dac a si numai dac a 60 | m1.
Not a. Solutie corect a a dat Marius Pachitariu, elev, Iasi.
L51. Fie A, B M
2
(R) dou a matrice care comut a si pentru care det

A
2
+B
2

<
< (det A+ det B)
2
. S a se arate c a xA+yB este matrice nesingular a, x, y R

.
C at alin Calistru, Iasi
Solutie. Se arat a cu usurint a c a
det (xA+yB) = x
2
det A+y
2
det B +xy [det (A+B) det Adet B] , x, y C,
prin urmare
det (A+iB) det (AiB) = (det Adet B)
2
+ [det (A+B) det Adet B]
2
.
Cum AB = BA, avem (A+iB) (AiB) = A
2
+B
2
si relatia anterioar a devine
det

A
2
+B
2

= (det Adet B)
2
+ (det (A+B) det B det A)
2
.
Conditia din enunt se rezum a la a ar ata c a x, y R

, avem det (xA+yB) 6= 0, deci


ecuatia x
2
det A +y
2
det B +xy [det (A+B) det Adet B] = 0 nu admite solutii
reale nebanale. ntr-adev ar, discriminantul acestei ecuatii este
= [det (A+B) det Adet B]
2
4 det Adet B = det

A
2
+B
2

(det Adet B)
2
4 det Adet B = det

A
2
+B
2

(det A+ det B)
2
< 0.
L52. Fie Q C[X] un polinom de grad m avnd r ad acinile distincte. S a se
determine cardinalul mul timii
E = {P C[X] | A M
n
(C) a. . Q(A) = O
n
si P (X) = det (XI
n
A)} .
Ovidiu Munteanu, Brasov
Solutie. Fie P E. Exist a A M
n
(C), Q(A) = O
n
si P (x) = det (xI
n
A).
Fie C o r ad acin a a lui P. Rezult a c a sistemul AX = X are si o solutie nebanal a,
notat a X
0
. Este usor de v azut c a Q(A) X
0
= Q() X
0
si prin urmare Q() = 0, deci
r ad acinile lui P sunt n numere din multimea {
1
,
2
. . .
m
}, unde
i
sunt r ad acinile
lui Q. Prin urmare, P este determinat de n numere, nu neap arat distincte,
1
,
2
,
. . . ,
n
din multimea {
1
,
2
, . . . ,
m
}.
Invers, dnd un polinom P care are ca r ad acini n numere
1
,
2
, . . . ,
n
ca
mai sus, fie A matricea care are pe diagonala principal a
1
,
2
, . . . ,
n
si 0 n rest.
Evident, det (xI
n
A) = (x
1
) (x
n
) = P (x), iar Q(A) este o matrice
care are pe diagonala principal a Q(
i
) = 0 si 0 n rest, deci Q(A) = O
n
. Rezult a c a
P E. Prin urmare, num arul de elemente al lui E este egal cu num arul de posibilit ati
de a alege n numere oarecare dintr-o multime cu m elemente, f ar a a conta ordinea,
adic a C
n
m
.
L53. Fie n 2 si (A, +, ) un inel comutativ cu n
2
elemente, care are cel mult
n 2 divizori ai lui zero. S a se arate c a A este corp.
Gabriel Dospinescu, elev, Onesti
Solutie. Presupunem c a A nu este corp. Fie T multimea divizorilor lui zero;
urmeaz a c a T 6= . Presupunem c a T are k elemente, 1 k n 2. Consider am
150
x T si a A

\T. Exist a d 6= 0 nct xd = dx = 0. Atunci (ax) d = d (ax) = 0, deci


ax = 0 sau ax T. Dac a ax = 0, avem a T, fals. Deci ax T si prin urmare putem
defini f : A

\T T, f (a) = ax. CumA

\T are n
2
k1 elemente, T are k elemente
si
n
2
k 1
k
> n, exist a elementele diferite a
1
, a
2
, . . . , a
n+1
A

\ T astfel nct
f (a
1
) = f (a
2
) = = f (a
n+1
). Deducem c a (a
n+1
a
i
) x = x(a
n+1
a
i
) = 0,
i = 1, n. Deci a
n+1
a
i
T pentru i = 1, n. Asadar T are cel putin n elemente,
contradictie.
L54. Fie f : R R o func tie cu derivata continu a pentru care f (x) 6= 0, x 6= 0.
S a se determine func tiile continue : R R care satisfac identitatea
f (x)
Z
y
0
(t) dt
1
a
(y)

= f (y)
Z
x
0
(t) dt
1
a
(x)

, x, y R,
unde a 6= 0 este o constant a dat a.
Adrian Corduneanu, Iasi
Solutie. Relatia dat a pote fi scris a sub forma
R
y
0
(t) dt
1
a
(y)
f (y)
=
R
x
0
(t) dt
1
a
(x)
f (x)
, x, y6=0.
Prin urmare,
R
x
0
(t) dt
1
a
(x)
f (x)
= c, c R, deci
Z
x
0
(t) dt
1
a
(x) = cf (x),
egalitatea avnd loc si pentru x = 0, n baza continuit atii. Pentru x = 0 rezult a
(0) = acf (0).
Prin derivare obtinem ecuatia (x)
1
a

0
(x) = cf
0
(x), ce are solutia
(x) = e
ax

k +
Z
x
0
e
at
(acf
0
(t)) dt

, unde k = (0) = acf (0). Asadar,


(x) = ace
ax

f (0) +
Z
x
0
e
at
f
0
(t) dt

, c R.
L55. Fie a (0, ) \ {1}. Definim sirul (x
n
)
n1
prin x
0
=
a 1
lna
; x
n
=
a
lna

n
lna
x
n1
, n 1. Ar ata ti c a sirul este convergent si calcula ti lim
n
x
n
si lim
n
nx
n
.
Gheorghe Iurea, Iasi
Solutie. Prin inductie matematic a se arat a c a x
n
=
Z
1
0
a
x
x
n
dx, n N

,
x
0
=
a 1
lna
. Pentru a (0, 1) avem x
n
a x
n
a
x
x
n
, x [0, 1]; integrnd,
obtinem
a
n + 1

Z
1
0
a
x
x
n
dx
1
n + 1
. Deci lim
n
x
n
= 0. Pentru a (1, ), din
x
n
x
n
a
x
x
n
a, x [0, 1], deducem
1
n + 1
x
n

a
n + 1
, n N. Ca urmare,
lim
n
x
n
= 0.
n concluzie, pentru orice a (0, ) \ {1}, lim
n
x
n
= 0. Din relatia de recurent a
rezult a lim
n
nx
n1
= a, de unde deducem c a lim
n
nx
n
= a.
151
Probleme propuse
1
Clasele primare
P.74. Descoper a regula de formare, apoi completeaz a sirurile urm atoare:
a) 1,2,3; 2,3,5; 3,,; 5,,.
b) 11,10,12; 13,12,14; 15,,; 17,,.
c) 2,6,4; 3,7,5; 4,8,6; 5,,; 6,,.
( Clasa I ) nv. Maria Racu, Iasi
P.75. R aspundeti la urm atoarele ntreb ari:
a) De cte suprafete este m arginit cubul?
b) Ce form a au fetele cuboidului?
c) Ce form a are un obiect care se aseam an a cu sfera?
( Clasa I ) Aliona Loghin, elev a, Iasi
P.76. Completati casetele din expresia 654321 cu semnele grafice "+"
sau "" pentru a obtine cel mai mic rezultat posibil.
( Clasa a II-a) nv. Gheorghe Toma, Muncelu de Sus (Iasi)
2
2
a
3
4 5
6
b c
4
P.77. Un corp este format din trei cuburi a, b, c ca n
figura al aturat a. Fiecare cub are fetele numerotate de la
1 la 6, iar suma numerelor de pe oricare dou a fete opuse
ale sale este 7. Stiind c a pe fetele lipite ale cuburilor a si
b este scris acelasi num ar si c a aceeasi proprietate o au si
cuburile b si c, s a se afle suma tuturor numerelor scrise pe fetele corpului care nu se
v ad.
( Clasa a II-a) Oxana Pascal, elev a, Iasi
P.78. a) Verific a egalit atile: 1 +3 +5 +7 = 4 4, 1 +3 +5 +7 +9 +11 = 6 6;
b) Scrie rezultatul la fel ca la punctul a) pentru 1 + 3 + 5 + 7 + + 19.
( Clasa a III-a) Andreea Surugiu, student a, Iasi
P.79. 7 elevi m anc a 7 inghetate n 6 minute. Cti elevi vor mnca 24 nghetate
n 36 minute?
( Clasa a III-a) Alexandru Tudorache, elev, Iasi
P.80. Dou a orase sunt legate printr-o linie de cale ferat a. La fiecare or a pleac a
un tren din fiecare oras c atre cel alalt. Toate trenurile merg cu aceeasi vitez a si fiecare
c al atorie de la un oras la altul dureaz a 6 ore. De cte ori fiecare tren, care parcurge
distanta dintre orase, se ntlneste cu trenuri care merg n sens opus?
( Clasa a IV-a) Alexandru Tudorache, elev, Iasi
P.81. S a se arate c a din fetele unui cub confectionat din carton putem construi,
f ar a resturi, fetele a sase cuburi.
( Clasa a IV-a) Petru Asaftei, Iasi
P.82. S a se afle cel mai mare num ar natural de forma abcd cu propriet atile:
a 6= d, b +c = 5 (a +d).
( Clasa a IV-a) Adrian Andronic, elev, Iasi
P.83. Mircea mpreun a cu fratele s au au un num ar de bomboane mai mic dect
30. Mircea are de 3 ori mai multe dect fratele s au. Aflati cte bomboane trebuie s a
1
Se primesc solutii pn a la data de 1 iunie 2005.
152
i dea Mircea fratelui s au pentru a r amne cu un num ar de dou a ori mai mare dect
al fratelui. Cte bomboane avea Mircea la nceput si cu cte a r amas?
( Clasa a IV-a) Inst. Tudor Tudorache, Craiova
Clasa a V-a
V.51. ntre oricare dou a numere naturale definim operatia a b = a
b
+a.
a) S a se rezolve ecuatia 2 (x + 1) = 34.
b) Este operatia dat a comutativ a?
Vasile Solcanu, Bogd anesti (Suceava)
V.52. Un dreptunghi se poate descompune n 1344 p atrate de arie 25 cm
2
. Aflati
perimetrul dreptunghiului dac a acesta este: a) maxim posibil; b) minim posibil.
Romanta Ghit a si Ioan Ghit a, Blaj
V.53. Determinati n N pentru care
3
2

1 +
1
3
+ +
1
3
n

5
4

1 +
1
5
+ +
1
5
n
=
2 + 6 + + 98
1 + 3 + + 17
.
Viorel Cornea, Hunedoara
V.54. S a se arate c a nu exist a numere rationale pozitive a, b, c astfel nct
a +b
ab
= 2
2003
,
b +c
bc
= 2
2004
si
c +a
ca
= 2
2005
.
Andrei - Sorin Cozma, elev, Iasi
V.55. Fie num arul rational N =
1
a
1
+
2
a
2
+
3
a
3
+ +
2004
a
2004
, unde a
i
N

,
i = 1, 2004. S a se arate c a exist a a
1
, a
2
, . . . , a
2004
astfel nct N =
1
2005
. Gene-
ralizare.
Petru Asaftei, Iasi
Clasa a VI-a
VI.51. Pentru efectuarea unei lucr ari, trei muncitori au fost retribuiti cu sume
de bani direct proportionale cu numerele 16, 14, 17. Unul dintre muncitori constat a
c a dac a sumele primite ar fi fost invers proportionale cu numerele 3, 4, 5, el ar fi
primit mai putin cu 1000000 lei. Aflati ce sum a de bani a primit fiecare muncitor.
Ion Visan, Craiova
VI.52. Determinati numerele ntregi n care pot fi scrise sub forma n =
1
a
+
2
b
+
3
c
,
cu a, b, c Z

.
Gheorghe Iurea, Iasi
VI.53. Se d a unghiul ascutit
d
xOy si punctele A, B (Ox, C, D (Oy astfel
nct A (OB), C (OD), AB 6= CD si t OA + s AB = t OC + s CD, cu
s, t R

. Atunci mediatoarele segmentelor [AB] si [CD] si bisectoarea lui


d
xOy sunt
trei drepte concurente dac a si numai dac a t = 2s.
Ioan S ac aleanu, Hrl au
VI.54. Fie 4ABC isoscel (AB = AC), N mijlocul lui [AC], iar D un punct pe
prelungirea lui [BC] astfel nct CD < BC. S a se arate c a ntre triunghiurile ABN
si NCD nu exist a nici o congruent a.
Romanta Ghit a si Ioan Ghit a, Blaj
153
VI.55. Fie punctele O, A
1
, A
2
, A
3
, . . . astfel nct OA
1
= OA
2
= OA
3
= =
= 1 cm, iar m(
\
A
1
OA
2
) = 1

, m(
\
A
2
OA
3
) = 2

, m(
\
A
3
OA
4
) = 3

etc. (toate unghi-


urile se consider a n sens orar). S a se arate c a exist a k 6= l astfel nct A
k
= A
l
.
Cristian Laz ar, Iasi
Clasa a VII-a
VII.51. Fie a, b N astfel nct n
2004
a se divide cu nb, pentru orice n N,
n 6= b. S a se arate c a a = b
2004
.
Alexandru Negrescu, elev, Botosani
VII.52. Fie a, b, c R cu a +b +c = 0; s a se arate c a

a
3
b
3

3
+

b
3
c
3

3
+

c
3
a
3

3
= 3(a b)(b c)(c a)

a
2
bc

b
2
ac

c
2
ab

.
Anca Tutescu, elev a, Craiova
VII.53. Determinati m, n, p Z astfel nct solutia inecuatiei |mx 1| n s a
fie [p, p +m+ 1].
Ciprian Baghiu, Iasi
VII.54. Se consider a unghiul
d
xOy de m asur a 10

si un segment [MN] de lungime


a. S a se construiasc a, folosind numai rigla si compasul, un triunghi dreptunghic
OAB, A (Ox, B (Oy, avnd o catet a de lungime a.
Florin As avoaie, elev, Iasi
VII.55. Fie ABCD patrulater convex, iar {O} = AC BD. Bisectoarele in-
terioare ale unghiurilor
\
AOB,
\
BOC,
\
COD,
\
DOA taie laturile (AB), (BC), (CD),
respectiv (DA) n M, N, P, respectiv Q. S a se arate c a dreptele MQ, NP si BD
sunt concurente sau paralele.
Constantin Cocea si Dumitru Neagu, Iasi
Clasa a VIII-a
VIII.51. Se consider a functiile f, g, h : R R definite prin f (x) = x, g (x) =
=
x

3
, h(x) = 3. Not am {A} = G
g
G
h
, {B} = G
f
G
h
, iar C si D sunt punctele de
intersectie ale dreptei x = 2 cu G
f
, respectiv G
g
. Determinati m asurile unghiurilor,
perimetrul si aria patrulaterului ABCD.
Dumitru - Dominic Bucescu, Iasi
VIII.52. Fie E(x, y) = 2004 2x
2
5y
2
+ 2xy + 6y, cu x, y N. Determinati
valoarea maxim a a lui E.
Gheorghe Iurea, Iasi
VIII.53. S a se arate c a pentru orice a, b, c R, are loc inegalitatea
a
4
+b
4
+c
4
+ 3a
2
b
2
+ 3a
2
c
2
+ 3b
2
c
2
2

a
3
b +ab
3
+a
3
c +ac
3
+b
3
c +bc
3

.
Marian Tetiva, Brlad
VIII.54. Pentru a, b, c (0, ), s a se demonstreze inegalitatea
a +
p
b
2
+c
2

p
a
2
+b
2
+c
2
<
2a

b
2
+c
2

a
2
+b
2
+c
2
< a +
p
b
2
+c
2
.
Radu Frunz a si Mircea Cosbuc, elevi, Iasi
VIII.55. O piramid a triunghiular a regulat a este tetraedru regulat dac a si numai
dac a unghiurile f acute de o fat a lateral a cu planul bazei, respectiv cu o alt a fat a
lateral a, sunt congruente.
Claudiu - Stefan Popa, Iasi
154
Clasa a IX-a
IX.51. Fie sirul (a
n
)
n1
definit prin: a
1
= 1 +2 3; a
2
= a
1
+4 +5 +6 7 8;
a
3
= a
2
+ 9 + 10 + 11 + 12 13 14 15 etc.
a) S a se determine semnele cu care apar 100 n a
100
, respectiv 91 n a
91
.
b) S a se afle formula termenului general al sirului.
Lidia Nicola, Craiova
IX.52. S a se determine functiile f, g : Z R cu propriet atile: f (0) = 2004,
f este par a, g este impar a si exist a a, b N astfel ncat f (x) = f

x
2
+x +a

,
g (x) = g

x
2
+x +b

, x Z.
D. M. B atinetu-Giurgiu, Bucuresti
IX.53. Exist a functii f : R R pentru care
|f (x +y +z +t) + cos x + cos y + cos z + cos t| < 4, x, y, z, t R?
Lucian Tutescu, Craiova
IX.54. Fie ABCD un p atrat de latur a a, iar T (AD) astfel nct m(
[
ABT) = .
Not am {S} = AC BT si fie R punctul n care perpendiculara n S pe BT inter-
secteaz a AB.
a) S a se arate c a 4RST este isoscel.
b) S a se exprime RS functie de a si .
Gheorghe Costovici, Iasi
IX.55. Fie ABC un triunghi cu c < b. Not am cu M si N mijloacele laturilor
[AB], respectiv [AC] si cu Dsi E punctele de tangent a a cercurilor nscris si respectiv
A- exnscris triunghiului cu latura [BC]. Ar atati c a
(i) ME si ND se intersecteaz a pe mediana din vrful A;
(ii) MD k NE a = 2 (b c);
(iii) dac a a 6= 2 (b c), atunci MD si NE se intersecteaz a pe prelungirea medianei
din A.
Temistocle Brsan, Iasi
Clasa a X-a
X.51. Fie OABC un tetraedru cu OA OB OC, circumscris unei sfere de
raz a r. Dac a R este raza cercului circumscris 4ABC, atunci
R
r

3 + 1
2
.
Cezar Lupu, elev, Constanta
X.52. Fie polinomul
P (X) =

1 +X +X
2

6n+1
=
12n+2
X
k=0
a
k
X
k
, n N.
S a se arate c a
2n
P
k=0
a
6k
=
2n
P
k=0
a
6k+2
.
C at alin Calistru, Iasi
X.53. Fie a, b, c (1, ) astfel nct a +b +c = 9. S a se arate c a
log
a

2b
3
+c
3

+ log
b

2c
3
+a
3

+ log
c

2a
3
+b
3

12.
Angela Tig aeru, Suceava
X.54. Definim multimile A
k
, k 1, prin
A
1
=
n
n
2003
| n = 1, 2, . . . , 10000
o
\ {1} ; A
k
= (A
k1
\ {a, b})

a
2 lg b

,
155
cu a, b A
k1
arbitrare, k 2. S a se determine A
9999
.
Marius Pachitariu, elev, Iasi
X.55. Fie a, b Z cu a
2
4b < 0, iar o solutie a ecuatiei x
2
+ ax + b = 0.
Definim functia f : Z Z Z, f (x, y) = x
2
axy + by
2
. Pentru orice pereche
(x, y) f
1
(1), s a se arate c a (x +y)
cardf
1
(1)
= 1.
Andrei Nedelcu, Iasi
Clasa a XI-a
XI.51. S a se calculeze determinantul unei matrice p atratice de ordinul patru care
are toti minorii de ordin trei egali.
Lucian - Georges L adunc a, Iasi
XI.52. Fie functia f : M
2
(R) [0, ), f (A) = det

A
2
+I
2

, A M
2
(R).
a) S a se arate c a f (A) = (det A1)
2
+ (tr A)
2
, A M
2
(R).
b) S a se demonstreze c a f este surjectiv a, dar nu este injectiv a.
Ovidiu Pop, Satu Mare
XI.53. Fie R; pentru n 3, definim

k
=

cos

+
2k
n

cos sin

+
2k
n

sin
cos

+
2(k+1)
n

cos sin

+
2(k+1)
n

sin

, k N

.
S a se calculeze limita sirului (a
n
)
n3
, a
n
=
n2
P
k=1
|
k
|.
Gheorghe Croitoru si Gabriel Popa, Iasi
XI.54. Fie k N

; s a se arate c a ecuatia x
n+k
x
n
x
n1
x 1 = 0 are
o singur a solutie pozitiv a, pe care o not am x
n
. S a se arate apoi c a sirul (x
n
)
n1
este
convergent; ce se poate spune despre limita sa?
Dumitru Mihalache si Marian Tetiva, Brlad
XI.55. Determinati toate functiile f : R R pentru care
f

x
2n+1
+x

x f
2n+1
(x) +f (x) , x R,
unde n N. (n leg atur a cu problema 2811 din Crux Mathematicorum, nr. 1/2003)
Titu Zvonaru, Com anesti
Clasa a XII-a
XII.51. S a se arate c a sirul (a
n
)
n1
, a
n
=
Z
1
1/n
sinx
x
p
dx, p < 2, este convergent.
Rodica Luca Tudorache, Iasi
XII.52. Fie f : [0, 1] R derivabil a, cu derivata continu a, astfel nct
f (x) +f
0
(x) = 0, x [0, 1]. S a se arate c a
Z
1
0
f (x) dx
(2e 5) f (1)
5
+
4
e
Z
2
1
xe
x
f (x 1)
(x
2
+ 1)
2
dx.
Mihail Bencze, Brasov
XII.53. Prove that
2 (e
x
1)
e
x
+ 1

Z
x
x

e
t
e
t
+ 1
dt x, x 0.
Zdravko Starc, Vrsac, Serbia and Montenegro
156
XII.54. S a se afle functiile continue u = u(t), solutii ale ecuatiei
u(t) = +
Z
t
0
b (s) u(s) ds +
Z
a
0
b (s) u(s) ds, 0 t a,
unde este constant a, iar b = b (t) este continu a pe [0, a].
Adrian Corduneanu, Iasi
XII.55. S a se arate c a pentru orice n N

, exist a monoizi care nu sunt grupuri


si care contin exact n elemente inversabile.
Paul Georgescu si Gabriel Popa, Iasi
Probleme pentru preg atirea concursurilor
A. Nivel gimnazial
G66. Se consider a multimea A = {1, n + 1, 2n + 1, . . . , mn + 1}, m, n N

,
m > n. S a se afle cte valori distincte poate lua suma a
1
+ a
2
+ + a
n
, unde
a
1
, a
2
, . . . , a
n
A.
Petru Asaftei, Iasi
G67. Fie b N, b 2. Spunem c a un num ar natural este decompozabil dac a se
poate scrie ca suma a dou a numere cu aceesi sum a a cifrelor n baza b. S a se arate
c a exist a o infinitate de numere care nu sunt decompozabile.
Adrian Zahariuc, elev, Bac au
G68. Fie N N

; s a se arate c a exist a n N astfel nct factorialul niciunui


num ar natural s a nu se termine cu n, n + 1, . . . , n +N zerouri.
Iuliana Georgescu, Iasi
G69. Fie E(x) = ax
2
+bx +c, a, b, c Q, x R. Dac a a +b +c Z, ar atati c a
exist a o innitate de numere ntregi n astfel nct E(n) s a e num ar ntreg.
Gheorghe Iurea, Iasi
G70. S a se arate c a ecuatia x
2
+y
2
+ 3x +y 707 = 0 nu are solutii n Q
2
.
Dan Popescu, Suceava
G71. Fie (m, n) N
2
\ {(0, 0)}. S a se demonstreze inegalitatea
a
(m+n) a
2
+mb
2
+nc
2
+
b
(m+n) b
2
+mc
2
+na
2
+
c
(m+n) c
2
+ma
2
+nb
2

1
2 (m+n)

1
a
+
1
b
+
1
c

, a, b, c (0, ) .
Titu Zvonaru, Com anesti
G72. Fie 4ABC circumscris cercului de centru I. Cercul de diametru [AI]
intersecteaz a bisectoarele unghiurilor
b
B si
b
C n M, respectiv N. S a se arate c a M si
N se afl a pe dreapta suport a liniei mijlocii paralele cu BC.
Doru Buzac, Iasi
G73. Fie ABCD un dreptunghi de centru O. Consider am N (AO), M mijlocul
lui [AD], {P} = MN CD, {E} = OP BC. S a se arate c a NE BC.
Andrei Nedelcu, Iasi
157
G74. Fie n puncte n spatiu astfel nct oricare patru s a formeze tetraedre de
volum cel mult 1. S a se arate c a exist a un tetraedru de volum cel mult 27 care s a
contin a n interior toate cele n puncte.
Tudor Chiril a, elev, Iasi
G75. Fie A
1
A
2
. . . A
n
un poligon regulat de latur a 1, n 4. Pe latura [A
1
A
2
]
se consider a punctul P
1
cu P
1
A
1
= a (0, 1). Din punctul P
1
se propag a o raz a de
lumin a care se reect a de laturile [A
2
A
3
], [A
3
A
4
], . . . , genernd pe laturi punctele
de incident a P
2
, P
3
, . . . (presupunnd c a raza de lumin a nu ajunge niciodat a ntr-un
vrf al poligonului) astfel nct m(
\
A
2
P
1
P
2
) =

n
,
2
n

. S a se ae valoarea
minim a a lui l pentru care P
l
si P
l+1
nu apartin la dou a laturi consecutive.
Irina Mustat a, elev a, Iasi
B. Nivel liceal
L66. Fie ABC un triunghi, D si D
a
punctele n care cercurile nscris si A-
exnscris sunt tangente la BC si E
b
, F
c
punctele n care cercurile B-exnscris si C-
exnscris sunt tangente la AC si respectiv AB. S a se arate c a punctele D, D
a
, E
b
,
F
c
sunt conciclice dac a si numai dac a AB = AC sau m(
b
A) = 90

.
Temistocle Brsan, Iasi
L67. Dreptele paralele t
1
si t
2
sunt tangente cercului C de centru O. Cercul C
1
de centru O
1
este tangent la t
1
si C, iar cercul C
2
de centru O
2
este tangent la t
2
, C
si C
1
; cele trei cercuri sunt exterioare unul celuilalt. S a se arate c a unghiul
\
O
1
OO
2
este ascutit si s a se afle valoarea maxim a a m asurii acestuia.
Neculai Roman, Mircesti (Iasi)
L68. a) Pentru x, y, z (0, ), s a se demonstreze inegalitatea
s
(x +y +z)

1
x
+
1
y
+
1
z

1 +
r
x
y
+
r
y
x
.
b) Folosind eventual a), s a se arate c a n orice triunghi, cu notatiile uzuale, are
loc inegalitatea
r
1 + 4
R
r
1 +
r
p a
p b
+
s
p b
p a
.
Marian Tetiva, Brlad
L69. Pentru ce numere naturale n 3, exist a n plan n puncte albastre si n
puncte rosii, oricare trei necoliniare, astfel nct n interiorul oric arui triunghi cu
vrfurile albastre s a existe cel putin un punct rosu, iar n interiorul oric arui triunghi
cu vrfurile rosii s a existe cel putin un punct albastru?
Adrian Zahariuc, elev, Bac au
L70. Fie k, p N

si un dreptunghi de dimensiuni 82k 2p, acoperit complet si


f ar a suprapuneri cu dreptunghiuri 7 5 si 6 4. S a se arate c a num arul p atr atelelor
(x, y), x par, y impar, ale dreptunghiului mare, care sunt colturi n dreptunghiuri
75, este egal cu num arul de dreptunghiuri 75. (Prin dreptunghiuri 75 ntelegem
dreptunghiuri cu lungimea egal a cu 7 si l atimea egal a cu 5.)
Marius Pachitariu, elev, Iasi
158
L71. Fie n N, n 2 fixat. S a se determine cea mai tare inegalitate de forma
n
X
k=1
q
a
2
k
+n
2
1 m
n
X
k=1
a
k
+M,
unde m, M nu depind de a
1
, a
2
, . . . , a
n
, valabil a pentru orice numere a
1
, a
2
, . . . ,
a
n
pozitive si cu produsul 1.
Gabriel Dospinescu, Bucuresti
L72. Fie a, b numere rationale, pozitive, distincte, astfel nct a
n
b
n
Z pentru
o infinitate de numere naturale n. S a se arate c a a si b sunt ntregi.
Gabriel Dospinescu, Bucuresti
L73. Fie k N

, k 3. S a se determine n N \ {0, 1} pentru care


r
a
1
+
q
a
2
+ +

a
k

n

a
1
a
2
. . . a
k
, a
1
, a
2
, . . . , a
k
[0, ).
Gabriel Popa si Paul Georgescu, Iasi
L74. Fie n N

, n 2 si a, b R, a < b. Dac a f : [a, b] R este continu a si


R
b
a
x
k
f (x) dx = 0 pentru 0 k n, atunci f are cel putin n + 1 zerouri distincte n
(a, b).
Andrei Nedelcu, Iasi
L75. S a se determine n N pentru care este adev arat a inegalitatea
cos <
1
8
p
1 +nsin
4

0,

2
i
.
C at alin Calistru, Iasi
Training problems for mathematical contests
Junior high school level
G66. Considering the set A = {1, n+1, 2n+1, . . . , mn+1}, m, nN

, m>n, nd
the number of distinct values taken by the sum a
1
+a
2
+. . .+a
n
, when a
1
, a
2
, . . . , a
n
A.
Petru Asaftei, Iasi
G67. Let b N, b 2. It is said that G N is decomposable if we can write G as
a sum of two numbers such that their expansions in the basis b have the same sum of
digits. Prove that there exist innitely many numbers which are not decomposable.
Adrian Zahariuc, high school student, Bac au
G68. Let N N

. Prove that there is n N such that no factorial ends in n,


n + 1, . . . , n +N zeros.
Iuliana Georgescu, Iasi
G69. Let E(x) = ax
2
+bx+c, a, b, c Q, x R. If a +b +c is an integer, prove
that there exist innitely many integers n such that E(n) is also an integer.
Gheorghe Iurea, Iasi
G70. Prove that the equation x
2
+y
2
+3x +y 707 = 0 has no solutions in Q
2
.
Dan Popescu, Iasi
159
G71. Let (m, n) N
2
\ {(0, 0)}. Prove that
a
(m+n) a
2
+mb
2
+nc
2
+
b
(m+n) b
2
+mc
2
+na
2
+
c
(m+n) c
2
+ma
2
+nb
2

1
2 (m+n)

1
a
+
1
b
+
1
c

, a, b, c (0, ) .
Titu Zvonaru, Com anesti
G72. Let I be the incenter of a triangle ABC. The circle with diameter [AI]
meets the bisectors of
b
B and
b
C in M, respectively N. Prove that M and N lie on
the line joining the midpoints of [AB] and [AC].
Doru Buzac, Iasi
G73. Let ABCD be a rectangle with center O. Let N (AO), let M be the
midpoint of [AD] and let {P} = MNCD, {E} = OP BC. Prove that NE BC.
Andrei Nedelcu, Iasi
G74. Let us consider n points such that any given four are the vertices of a
tethraedron with volume at most 1. Prove that there is a tethraedron with volume
at most 27 which contains all n points in its interior.
Tudor Chiril a, high school student, Iasi
G75. Let A
1
A
2
. . . A
n
be a regular n-gon with side 1, n 4. We consider P
1
on the side [A
1
A
2
] such that P
1
A
1
= a (0, 1). A ray of light is emitted from P
1
towards and is reected by the sides [A
2
A
3
], [A
3
A
4
], . . . , generating the incidence
points P
2
, . . . , P
n
(supposing that the ray never meets the vertices A
1
, A
2
, . . . , A
n
)
such that m(
\
A
2
P
1
P
2
) =

n
,
2
n

. Find the minimal value of l such that P


l
and
P
l+1
do not belong to adjacent sides.
Irina Mustat a, high school student, Iasi
High school level
L66. Let ABC be a given triangle and let D, D
a
be the points in which the
incircle, respectively the A-escribed circle are tangent to the side BC. Let also E
b
,
F
c
be the points in which the B-escribed and C-escribed circles are tangent to the
side AC, respectively to the side AB. Prove that D, D
a
, E
b
, F
c
are concyclic if and
only if AB = AC or m(
b
A) = 90

.
Temistocle Brsan, Iasi
L67. The parallel lines t
1
and t
2
are tangent to the circle C with center O, the
circle C
1
with center O
1
is tangent to t
1
and C and the circle C
2
with center O
2
is
tangent to t
2
, C and C
1
; C, C
1
and C
2
being exterior to each other. Prove that the
angle
\
O
1
OO
2
is acute and nd the minimum value of its measure.
Neculai Roman, Mircesti (Iasi)
L68. a) Prove that, for x, y, z (0, ),
s
(x +y +z)

1
x
+
1
y
+
1
z

1 +
r
x
y
+
r
y
x
.
160
b) Using a), prove that
r
1 + 4
R
r
1 +
r
p a
p b
+
s
p b
p a
for any given triangle with the usual notations.
Marian Tetiva, Brlad
L69. Find n N, n 3, such that there are n blue points and n red points in
the same plane, no three points being collinear, such that the interior of any triangle
with blue vertices contains at least one red point and the interior of any triangle with
red vertices contains at least one blue point.
Adrian Zahariuc, high school student, Bac au
L70. Let k, p N

and let a 82k 2p rectangle which is completely covered with


7 5 and 6 4 rectangles with no superpositions. Prove that the number of squares
(x, y) with side 1, x even, y odd, which are vertices of 7 5 rectangles equals the
total number of 7 5 rectangles. (By a 7 5 rectangle we mean a rectangle with
length 7 and height 5).
Marius Pachitariu, high school student, Iasi
L71. Let n N, n 2. Find the best constants m, M such that
n
X
k=1
q
a
2
k
+n
2
1 m
n
X
k=1
a
k
+M
for any a
1
, a
2
, . . . , a
n
> 0 satisfying a
1
a
2
. . . a
n
= 1.
Gabriel Dospinescu, Bucuresti
L72. Let a, b Q, a, b > 0, a 6= b such that a
n
b
n
Z for innitely many n N.
Prove that a, b Z.
Gabriel Dospinescu, Bucuresti
L73. Let k N

, k 3. Find n N\ {0, 1} such that


r
a
1
+
q
a
2
+ +

a
k

n

a
1
a
2
. . . a
k
, a
1
, a
2
, . . . , a
k
[0, ).
Gabriel Popa and Paul Georgescu, Iasi
L74. Let n N

, n 2 and a, b R, a < b. If f : [a, b] R is a continuous


function such that
R
b
a
x
k
f (x) dx = 0 for any k N, 0 k n, then f has at least
n + 1 distinct zeros in (a, b).
Andrei Nedelcu, Iasi
L75. Find n N such that
cos <
1
8
p
1 +nsin
4

0,

2
i
.
C at alin Calistru, Iasi
161
Pagina rezolvitorilor
BOTO SANI
Scoala nr. 7 "O. B ancil a". Clasa a IV-a. IFTODE Cozmin: P(54-59,61,63,64,
67,68,70).
Colegiul Na tional "A. T. Laurian". Clasa a IX-a. NEGRESCU Alexandru:
VII(43,47,48), VIII(47,49), IX(42,46,48), X.42, G46.
CRAIOVA
Scoala nr. 22 "M. Eliade". Clasa a IV-a (inst. VAN TU Angela). STANCIU
Ioan: P (64-73)
IA SI
Scoala nr. 3 "Al. Vlahu t a". Clasa a V-a. COJOCARU Ioana: P.71, V(47-
49),VI.46; DODU Corina: P.71, V(47-49),VI.46; IRIMIA Andreea: P.71, V(47-
49),VI.46; STIRBAN Ioana: P.71, V(47-49),VI.46; UNGURU George Claudiu: P.71,
V(47-49),VI.46.
Scoala nr. 7 "N. Tonitza". Clasa a II-a (nv. TUDOSE Elena). DOBRIN Diana -
Maria: P(64-68,70,72); LEONTE Anca: P(64-68,70,72); POSTIC

A Simona - Alexan-
dra: P(64-68,70,72); ROTARU Larisa-Maria: P(64-68,70,72); SAVIN R azvan: P(64-
68,70,72). Clasa a II-a (nv. MELINTE Rodica). BACIU Ciprian: P(64-69,71,72);
BRZU Constantin: P(64-69,71,72); BOTO SANU Bianca-Mihaela: P(64-69,71,72);
BUZDUGAN Petru-C at alin: P(64-69,71,72); CEUC

A D anut-Vasilic a: P(64-69,71,
72); CONSTANTINESCU Diana-Gabriela: P(64-69,71,72); CUCUTEANU Paul-
C at alin: P(64-69,71,72); GU SOVATE Diana- Stefana: P(64-69,71,72); LEOGAN
Larisa-Diana: P(64-69,71,72); MIRON Vlad- Stefan: P(64-69,71,72); MOTAN Geani-
na-Diana: P(64-69,71,72); ROTARIU Marian: P(64-69,71,72); SUCIUC Raluca:
P(64-69,71,72); TEIU-COSTIN Andra-Mihaela: P(64-69,71,72). Clasa a III-a
(nv. PA SANIUC Maria). ATASIEI Vl adut: P(64,65,67,68,72); P

AS

AROIU Bogdan:
P(66-68,70,71); SOLOMON Ana-Maria: (65,68-71); TINCU Andrei: P(64,65,68-
70,72); ZAMFIR Loredana-Cristiana: P(64,65,69-71). Clasa a III-a (nv. GEAM

AN
Gabriela). AD

ASC

ALI TEI IONU T: P(65,68-71); GHIARASIM Olivia: P(64,65,67,


68,72); MACOVEI Alina: P(64,65,68-70,72); NEGRESCU Vlad-Petru: P(64,65,68-
71); TROCIN Monica-Andreea: P(66,68-71).
Scoala nr. 13 "Alexandru cel Bun". Clasa a IV-a (inst. COJOCARIU Ana). CO-
BILI T

A C at alina-Elena: P(64-68,70); CURMEI Renata-Maria: P(64-68,70); DALAS


Radu: P(64-68,70); ILIE Laura: P(64-68,70); MAGDICI Magda-Otilia: P(64-68,70);
MARDARI Claudiu: P(64-68,70); LUCAN Mihaela-Alexandra; P(64-68,70); NECHI-
FOR Stefan-Marian: P(64-68,70); NICULAE-GRIGORESCU Andrei: P(64-68,70);
PRISACARU Carmen-Georgiana: P(64-68,70); PETREA Silvia: P(64-68,70); STE-
FAN Teodora-Ioana: P(64-68,70); TRL

AGEANU Ingrid-Maria: P(64-68,70).


Scoala nr. 17 "I. Creang a". Clasa a VIII-a. DUMITRIU Vlad: VI(46-48),VII.48,
VIII.46.
Scoala nr. 22 "B. P. Hasdeu". Clasa I (nv. STEFAN Liviu). D

ANIL

A Co-
drin P(55,64,65,67,71); IGNAT Andreea: P(54-56,64,65); NICOLA Delia-Corina:
P(55,64,65,67,71); PURICE Dumitru-Ciprian: P(64-67,71). Clasa a II-a (nv. DO-
162
HOTARU Liliana). TURCU Andrei-Daniel: P(64-67,71); Clasa a II-a (nv. TRZI-
ORU Iuliana). AD

ASC

ALI TEI Victor: P(64-69,71,72); APOSTOL Ana-Maria:


P(64-68,71,72); BALAN Andrei: P(64-68,71,72); BURUIAN

A C at alina: P(64-68,72);
CUBERSCHI Paul: P(64-67,70-72); E SANU Georgiana: P(64-68,71); GREIEROSU
Claudiu: P(64-68,72); GNDU Alexandra-Livia: P(64-66,68,71,72); L

AM

ATIC Ioa-
na: P(61,64-68,71); MOGA Alexandru: P(64-68,71,72); REBEGEA Andrada: P(64-
68,71); UNGUREANU Teofana: P(64-68,71,72). Clasa a II-a (nv. TUTU Laura).
AILENEI-OPREA Adriana: P(64-68); ANDRONICIUC Ana-Miruna: P(64-68); BR-
L

ADEANU Claus-Alex: P(64-66,68,71); BOARU Adrian: P(64-66,68,71); BURUIA-


N

A Sebastian-Andrei: P(64-68,71): BUHU Vlad: P(64-68,71); CEOBANU Andrei-


Nicolae: P(64-67,71); CHICHIR

AU Alexandra-Elena: P(64-68,71); COST

ACHESCU
Ivona: P(64-68,71); D

ANIL

A Alexandru: P(64-66,68,71); DIACONESCU Matei:


P(64-68,71); DOROHOI Ovidiu: P(64-68,71); GHERAN Ana-Maria: P(64-66,68,71);
GRIGORE Georgiana: P(64-68,71); GUR

AU Raluca-Claudia: P(64-68,71); HATES-


CU Iustina: P(64-68,71); HORBOVANU Bianca-Alexandra: P(64-68,71); N

ASTASE
Andrei-Ionut: P(64-68,71); ONOFREI Liviana Ana-Maria: P(64-68,71); RADU
Andrei: P(64-66,68,71); SIMIRAD Andrei: P(54,55,57-59,61,62,64-72); SMEREA
Alexandra-Arina: P(64-66,68,71). Clasa a V-a. PINTILIE Mina-Liviu: P(61,62,71-
73); PINTILIE Nicoleta-Livia: P(61,62,71-73); STERBULEAC Daniel: P(61-63,71-
73),V(42,43).
Scoala nr. 23 "T. Maiorescu". Clasa a IV-a (nv. CHIRIL

A Beatrice). TUDO-
RACHE Alexandru-Gabriel: P(64-73).
Scoala nr. 26 "G. Co sbuc". Clasa a II-a (nv. BUCATARIU Rica). IACOB
Robert-Ionut: P(54-57,64-67); IVANCIUC Dumitru-Florin: P(55,64-67); MOISA
Adrian-Bogdan: P(54-57,64-67); SANDU Ioana-Luiza: P(54-57,64-67); SCUTARU
Ionela-Cristina: P(54-57,64-67). Clasa a III-a (nv. RACU Maria). BULGARU
Ionela-Alexandra: P(64-68,70-72); BURLACU Stefan-Claudiu: P(64-68,70,72); C

A-
LIN Andreea-Georgiana: P(64-68,70-72); IFROS

A Adriana: P(64,66-72); IOJ

A Pe-
tru-Alexandru: P(64-70,72); MOISA Bogdan: P(64-68,70-72); PINTILIE R azvan-
Florin: P(64-68,70-72); R

AZLOG Ionut: P(64-68,70-72). Clasa a III-a (nv. GALIA


Paraschiva). ALUPEI Andra-M ad alina: P(64-67,68,70,71); CIOAB

A Oana-C at alina:
P(64-68,70,71); GHERC

A Marius-C at alin: P(64-68,70,71); HOMEA Liviu: P(64-


68,70,71); HUIDE S Gina: P(64-68,70,71); MANOLIU M ad alina: P(64-68,70,71);
MIH

AILESCU Laura; P(64-68,70,71); PISIC

A Alexandru: P(64-68,70,71); POPA


Florin: P(64-68,70,71); SCUTARU Constantin: P(64-68,70,71,73).
Colegiul Na tional "C. Negruzzi". Clasa a V-a. ANDRIESCU Gabriela: P(56,58,
59,61,63),V.42; TIBA Marius: P(71-73),V(46,47,49,50),VI.46.
Colegiul Na tional Ia si. Clasa a V-a. VLCU Maria Caterina: V(40,41,43-45).
Liceul "M. Eminescu". Clasa a VI-a. CIURARU Ionela: V(46,48), VI(47,48,50);
IPATE Cristina: V(46,48),VI(47,48,50). Clasa a IX-a. AVRAM Mircea: VII(41,43,
44,46,48). Clasa a X-a. COMAN Stefan: VIII(46,49), IX(46,47), X.47.
163
Premii acordate rezolvitorilor
Pentru aparitia de trei ori la rubrica "Pagina rezolvitorilor" redactia revistei
"Recrea tii matematice" acord a o diplom a si un premiu n c arti elevilor urm atori:
Scoala nr. 22 "M. Eliade", Craiova
STANCIU Ioan (cl. a IV-a): 2/2003 (9pb), 1/2004 (10pb), 2/2004 (10pb).
Scoala nr. 7 "N. Tonitza"
BACIU Ciprian (cl. a II-a): 2/2003 (5pb), 1/2004 (5pb), 2/2004 (8pb);
BRZU Constantin (cl. a II-a): 2/2003 (5pb), 1/2004 (5pb), 2/2004 (7pb);
BOTO SANU Bianca-Mihaela (cl. a II-a): 2/2003 (5pb), 1/2004 (5pb), 2/2004
(7pb);
BUZDUGAN Petru-C at alin (cl. a II-a): 2/2003 (5pb), 1/2004 (5pb), 2/2004
(7pb);
CEUC

A D anut-Vasilic a (cl. a II-a): 2/2003 (5pb), 1/2004 (5pb), 2/2004 (8pb)


CONSTANTINESCU Diana-Gabriela (cl. a II-a): 2/2003 (5pb), 1/2004 (5pb),
2/2004 (8pb);
CUCUTEANU Paul-C at alin (cl. a II-a): 2/2003 (5pb), 1/2004 (5pb), 2/2004
(8pb);
DOBRIN Diana-Maria (cl. a II-a): 2/2003 (5pb), 1/2004 (5pb), 2/2004 (7pb);
GU SOVATE Diana- Stefana (cl. a II-a): 2/2003 (5pb), 1/2004 (5pb), 2/2004
(8pb);
LEOGAN Larisa-Diana (cl. a II-a): 2/2003 (5pb), 1/2004 (5pb), 2/2004 (8pb);
LEONTE Anca (cl. a II-a): 2/2003 (5pb), 1/2004 (5pb), 2/2004 (7pb);
MIRON Vlad- Stefan (cl. a II-a): 2/2003 (5pb), 1/2004 (5pb), 2/2004 (8pb);
MOTAN Geanina-Diana (cl. a II-a): 2/2003 (5pb), 1/2004 (5pb), 2/2004 (8pb);
POSTIC

A Simona-Alexandra (cl. a II-a): 2/2003 (5pb), 1/2004 (5pb), 2/2004


(7pb);
ROTARIU Larisa-Maria (cl. a II-a): 2/2003 (5pb), 1/2004 (5pb), 2/2004 (7pb);
ROTARIU Marian (cl. a II-a): 2/2003 (5pb), 1/2004 (5pb), 2/2004 (8pb);
SUCIUC Raluca (cl. a II-a): 2/2003 (5pb), 1/2004 (5pb), 2/2004 (8pb);
TEIU-COSTIN Andrada-Mihaela (cl. a II-a): 2/2003 (5pb), 1/2004 (5pb),
2/2004 (8pb).
Scoala nr. 22 "B. P. Hasdeu"
AD

ASC

ALI TEI Victor (cl. a II-a): 2/2003 (5pb), 1/2004, (5pb), 2/2004 (9pb);
BALAN Andrei (cl. a II-a): 2/2003 (5pb), 1/2004 (5pb), 2/2004 (8pb);
BUHU Vlad (cl. a II-a): 2/2003 (5pb), 1/2004 (5pb), 2/2004 (6pb);
CHICHIR

AU Alexandra-Elena (cl. a II-a): 2/2003 (5pb), 1/2004 (6pb), 2/2004


(6pb);
CUBERSCHI Paul (cl. a II-a): 2/2003 (5pb), 1/2004 (6pb), 2/2004 (7pb);
E SANU Georgiana (cl. a II-a): 2/2003 (5pb), 1/2004 (5pb), 2/2004 (5pb);
GREIEROSU Claudiu (cl. a II-a): 2/2003 (5pb), 1/2004 (6pb), 2/2004 (6pb);
GUR

AU Raluca-Claudia (cl. a II-a): 2/2003 (5pb), 1/2004 (5pb), 2/2004 (6pb);


164
N

ASTASE Andrei-Ionut (cl. a II-a): 2/2003 (5pb), 1/2004 (6pb), 2/2004 (6pb);
HATESCU Iustina (cl. a II-a): 2/2003 (5pb), 1/2004 (5pb), 2/2004 (6pb);
L

AM

ATIC Ioana (cl. a II-a): 2/2003 (5pb), 1/2004 (6pb), 2/2004 (7pb);
REBEGEA Andrada (cl. a II-a): 2/2003 (5pb), 1/2004 (5pb), 2/2004 (6pb).
Scoala nr. 23 "Titu Maiorescu"
TUDORACHE Alexandru-Gabriel (cl. a IV-a): 2/2003 (10pb), 1/2004 (10pb),
2/2004 (10pb).
Scoala nr. 26 "G. Co sbuc"
ALUPEI Andra-M ad alina (cl. a III-a): 2/2003 (5pb), 1/2004 (5pb), 2/2004 (7pb);
BULGARU Ionela-Alexandra (cl. a III-a): 2/2003 (5pb), 1/2004 (5pb), 2/2004
(8 pb);
BURLACU Stefan-Claudiu (cl. a III-a): 1/2003 (5pb), 1/2004 (5pb), 2/2004 (8pb);
GHERC

A Marius-C at alin (cl. a III-a): 2/2003 (5pb), 1/2004 (5pb), 2/2004 (7pb);
HOMEA Liviu (cl. a III-a): 2/2003 (5pb), 1/2004 (5pb), 2/2004 (7pb);
HUIDE S Gina (cl. a III-a): 2/2003 (5pb), 1/2004 (5pb), 2/2004 (7pb);
IFROS

A Adriana (cl. a III-a): 2/2003 (5pb), 1/2004 (5pb), 2/2004 (8pb);


IOJ

A Petru-Alexandru (cl. a III-a): 2/2003 (5pb), 1/2004 (5pb), 2/2004 (8pb);


PISIC

A Alexandru (cl. a III-a): 2/2003 (5pb), 1/2004 (5pb), 2/2004 (7pb).


Colegiul Na tional "C. Negruzzi"
TIBA Marius (cl. V-a): 1/2003 (6pb), 2/2003 (7pb), 1/2004 (7pb).
Liceul "M. Eminescu"
CIURARU Ionela (cl. VI-a): 2/2003 (11pb), 1/2004 (5pb), 2/2004 (5pb).
(continuarea tabelului din p. 128)
178. PIFTOR Rositta Liceul economic nr. 2, Iasi
179. SPIRIDON Doina Liceul de art a, Iasi
180. MARTINU SI Vladimir Colegiul National "E. Racovit a", Iasi
181. TIMOHE TUMAC Gabriel Liceul "G. Ibra aileanu", Iasi
182. ANTON Florina Cristiana Colegiul National "E. Racovit a", Iasi
183. PRICOP Vasile Pascani
165
Premiile pe anul 2004 acordate
de FUNDA TIA CULTURAL

A "POIANA"
Fundatia Cultural a "Poiana" (director d-l Dan Tiba) acord a anual premii
elevilor - colaboratori ai revistei "Recrea tii matematice" care se disting prin calitatea
articolelor, notelor si problemelor originale publicate n paginile acesteia.
Redactia revistei decide ca pentru anul 2004 premiile oferite, n valoare de cte
1 000 000 lei, s a e atribuite urm atorilor elevi:
1. LUPU Cezar (Colegiul Na tional "Mircea cel B atrn", Constan ta)
Asupra unei inegalit ati conditionate (RecMat 1/2004, 27-28),
probleme propuse: IX.36 (1/2003), IX.44 (2/2003), IX.48, XI.46 (1/2004),
X.51 (2/2004).
2. NEGRESCU Alexandru (Colegiul Na tional "A. T. Laurian", Boto sani )
Asupra unei inegalit ati (RecMat 2/2004, 106-108),
probleme propuse: VI.38 (1/2003), VII.41 (2/2003), VIII.46 (1/2004),
VII.51 (2/2204).
Premiile se pot ridica direct de la redactie sau pot trimise prin mandat postal
la adresa elevului premiat.
IMPORTANT
n scopul unei leg aturi rapide cu redactia revistei, pot fi utilizate urm a-
toarele adrese e-mail: tbi@math.tuiasi.ro, popagabriel@go.com.
Pe aceast a cale colaboratorii pot purta cu redactia un dialog privitor la
materialele trimise acesteia, procurarea numerelor revistei etc.
La problemele de tip L se primesc solutii de la orice iubitor de matematici
elementare (indiferent de preocupare profesional a sau vrst a). Fiecare
dintre solutiile acestor probleme - ce sunt publicate n revist a dup a un
an - va fi urmat a de numele tuturor celor care au rezolvat-o.
Adres am cu insistent a rug amintea ca materialele trimise re-
vistei s a nu fie (s a nu fi fost) trimise si altor publicatii.
166
Revista RECREAII MATEMATICE apare de dou ori pe an (la
datele de 1 martie i 1 septembrie) i se adreseaz elevilor, profesorilor,
studenilor i tuturor celor pasionai de matematicile elementare.
n atenia tuturor colaboratorilor
Materialele trimise redaciei spre publicare (note i articole, chestiuni de
metodic, probleme propuse etc.) trebuie prezentate ngrijit, clar i concis; ele
trebuie s prezinte interes pentru un cerc ct mai larg de cititori. Se recomand
ca textele s nu depeasc patru pagini. Evident, ele trebuie s fie originale
i s nu fi aprut sau s fi fost trimise spre publicare altor reviste.
Problemele originale destinate rubricii Probleme propuse vor fi
redactate pe foi separate cte una pe fiecare foaie, cu enun i
demonstraie/rezolvare, fiind nsoite de numele autorului, coala i localitatea
unde lucreaz/nva.
Redacia revistei va decide asupra oportunitii publicrii materialelor
primite.
n atenia elevilor
Numele elevilor care vor trimite redaciei soluii corecte la exerciiile i
problemele din rubrica Probleme propuse vor fi menionate n Pagina
rezolvitorilor. Elevii vor ine seama de urmtoarele reguli:
1. Pot trimite soluii la minimum cinci probleme propuse n numrul
prezent i cel anterior al revistei; pe o foaie va fi redactat soluia unei
singure probleme.
2. Elevii din clasele VI-XII au dreptul s trimit soluii la problemele
propuse pentru clasa lor, pentru orice clas mai mare, din dou clase mai mici i
imediat anterioare. Elevii din clasa a V-a pot trimite soluii la problemele
propuse pentru clasele a IV-a, a V-a i orice clas mai mare, iar elevii claselor
I-IV pot trimite soluii la problemele propuse pentru oricare din clasele primare
i orice clas mai mare. Orice elev poate trimite soluii la problemele de
concurs (de tip G i L).
3. Vor fi menionate urmtoarele date personale: numele i prenumele,
clasa, coala i localitatea.
4. Plicul cu probleme rezolvate se va trimite prin pot (sau va fi adus
direct) la adresa Redaciei:

Prof. dr. Temistocle Brsan
Catedra de Matematic
Universitatea Tehnic Gh. Asachi Iai
Bulevardul Carol I nr. 11, 700506, Iai
E-mail: tbi@math.tuiasi.ro
CUPRINS

Ctre cititori .................................................................................................................. 85
ALEXANDRU MYLLER, ctitorul colii matematice ieene.................................. 87
HENRI POINCAR la 150 de ani de la naterea sa ........................................ 89
Trecerea planetei Venus prin faa Soarelui ............................................................... 91

ARTICOLE I NOTE
G. DOSPINESCU Cteva noi aplicaii ale unei idei consacrate ........................ 94
T. BRSAN Cteva proprieti ale medianelor....................................................... 99
C. - t. POPA O construcie geometric a mediilor (II)..................................... 102
F. POPOVICI O generalizare a teoremelor de baz ale calculului diferenial ........ 104

NOTA ELEVULUI
A. NEGRESCU Asupra unei inegaliti................................................................ 106
*** Asupra problemei VII.41 din RecMat 2/2003 ....................... 109

CHESTIUNI METODICE
D. MIHALACHE, M. TETIVA Asupra unei probleme de concurs.................... 111

CHESTIUNI COMPLEMENTARE MANUALELOR
M. CRCIUN Exponentul numrului natural a n produsul n!......................... 114

CONCURSURI I EXAMENE
Concursul Alexandru Myller ed. a II-a, Iai, 2004 ............................................. 116
Concursul Florica T. Cmpan, ed. a IV-a, 2004 ................................................. 121
Concursul Traian Lalescu, ed. a V-a, Iai, 2004................................................ 123
Concursul Adolf Haimovici, ed. a VIII-a, 2004 .................................................. 124
Olimpiada Balcanic de Matematic (juniori), ed. a VIII-a, 2004 ....................... 126

CORESPONDENE
H. STEPHAN Probleme pentru clasa a VIII-a..................................................... 129

PROBLEME I SOLUII
Soluiile problemelor propuse n nr. 2/2003............................................................. 130
Soluiile problemelor pentru pregtirea concursurilor din nr. 2/2003 .................... 144
Probleme propuse........................................................................................................ 152
Probleme pentru pregtirea concursurilor ................................................................. 157
Training problems for mathematical contests .......................................................... 159

Pagina rezolvitorilor.................................................................................................... 162

S-ar putea să vă placă și